Vous êtes sur la page 1sur 390
Student Solutions Manual for use with Prepared by NAEEM AHMED. LAURENTIAN UNIVERSITY MIHAI GHERASE SANTO D'AGOSTINO BROCK UNIVERSITY NELSON EDUGATION NELSON EDUCATION ie President ela higher Education. ‘ane walla Publisher Pou Fan Marketing Manager: {Bare Newel Mensing Developmental Eator Mark Greskovrae ‘CoPYRUGHT © 2014, 2 Helion Eeucation te Printed and bound in the United States of America Visa i isa for inae information contact Nation Education 1120 aichmount Road, Toronto, Ontario, MIK SGA. you can st ‘ur amet site at tpt nelion com Student Solutions Manual ‘by Macem Atmce, Mina: Gherse, and Sant D'Agostine {for use with Physics for Scents and Engineers: ‘An Interactive Approach by Robert Hawken faved lab, Fras Mansour, Morin Niner Bolotin and Peter Wiliams ConentProsueson Manager’ Cover Design: Shrstine ibe oh Truong ony Eto: over mage: Sem D'Agonting 83002 In Kramer Design director Ken Pipe ALLAIGHTS RESERVED. No par of SuNeAI9790-17.9672129 ‘ths wok covered bythe conyight—GEMAG-0.176672133 herein may be reproduces {farscred, a ined nay form oF ty any meant graphs eectrone or mechani including photocopying recordin opin, Web dstrbuton o intmation erage and retieval stem without the writen permit of ‘the pubis. For permision to use materi from this text or produc, ubrit “wun congage-compermisions Further questions about permisions con be emailed to ery effort hasbeen mace 0 ‘race ounersog of al copyrighted Tater ane to secure perm sion from eryrght holden. Inthe ‘vert af any question asing as {othe us of any materiale vl be pleaes to mae the neensry cerrectons in future printings. Table of Contents Preface. (Chapter 1: Introduction to Physics Chapter 2: Scalars and Vectors . (Chapter 3: Motion in One Dimension Chapter 4: Motion in Two and Three Dimensions. Chapter 5; Forces and Motion... Chapter 6: Energy... Chapter 7: Lincar Momentum, Collisions, and Systems of Particles. Chapter 8: Rotational Dynamios Chapter 9: Rolling Motion Chapter 10: Equilibrium and Elasticity... Chapter LL: Gravitation. Chapter 12: Fluids Chapter 13: Oscillations Chapter L4: Waves Chapter 15: Interference and Sound. Chapter 16: Temperature and the Zeroth Law of Thermodynamics Chapter 17: Heat, Work, and the First Law of Thermodynamics Chapter 18: Heat Engines and the Second Law of Thermodynamics Chapter 19: Electriv Charges and Forves Chapter 20: Electric Potential and Gauss’s Law. Chapter 21: Capacitance 7 93 108 119 141 137 i 180 187 196 206 213 223 i Chapter 22 Chapter 23 Chapter 24 Chapter 25: Chapter 26 Chapter 27 Chapter 28: Chapter 29 Chapter 30 Chapter 31 Chapter 32 Chapter 33 Chapter 34 Electric Current and Fundamentals of DC Circuits Magnetic Fields and Magnetic Forces... Electromagnetic Induction... Alternating Current Circuits Electromagnetic Waves and Maxwell's Equations Geometric Optics... Physical Optics... Relativity Fundamental Discoveries of Modem Physics Introduction to Quantum Mechanies Introduction to Solid-State Physies .... Introduction to Nuclear Physics Introduction to Particle Physics 229 246 268 288 298 208 364 376 Preface The Student Solutions Manual accompanying the textbook Physics for Scientists and Engineers An Interactive Approach was prepared was prepared to assist the student in mastering the skills required for an understanding of physics. ‘The selected questions have been chosen by the authors of your text to allow you to discover the range and depth of your understanding ‘This Student Solutions Manual contains the worked-out solutions to all odd-numbered exercises at the end of each textbook chapter. It has been independently checked for accuracy Nacem Syed Ahmed Laurentian University Mihai Gherase Santo D°Agostino Brock University Chapter 1—INTRODUCTION TO PHYSICS 9. ‘The graph does not state the difference between the measurement points represented by dots and vireles. ‘The y-axis should have the unit label (km/s) not KM. Sines both measurements, (distance and velocity) have associated uncertainties, the corresponding x and y error bars should be included. There is no indication on the graph as to what the two fitted lines (solid and dashed) represent and which data points (if any) were excluded for those fits. 11. The duration of 9192631770 periods of the radiation corresponding to the transition between the two hyperfine levels of the ground state of a caesium 133 atom at rest at 0 K. 13. The ampere is « measure of the amount of electric charge passing a point in an electric cireuit per unit time, with 6.241 » 10'* electrons (one coulomb) per second constituting one ampere. References include the Bureau international des pords et mesures (BIPM, the standards agency for SI units), the Guide for the Use of the International System of Umts (SI) published by the National Institute of Standards and Technology, and general reference books 21. (@)4 (b)2 @)4 @2(@)3 4 23. (a) 2.452x10° (by 5.92107 (12x10 (@asx10" 25.8 Copyright © 2014 by Nelson Education Ltd 27. (a) (c) 7.2m (or £3.6m) 29. Weknow that [U]=kg-m +s, [m]=kg, [g]=m-s7 and [A] = m =U ~meh 31. V~IR som? -kg-s +A 1=(A)m?*-kg-s 7A? 2 Copyright © 2014 by Nelson Education Ltd. 41 43. 45 rug =25 om? Let us assume that the space between the cells is negligible pv =N 75x10" cells Von About 57 000 players are registered with Hockey Canada, Fach team may have 18 players and 2 goalies. That makes a total of 57000/20 = 2850 teams or 5700 goalies. Thus, 5-10 thousand would be a reasonable estimate, The actual number may be on the low end, because sometimes goalies play for more than one team, so the total number of goalies may be less than the estimated number. 4 2 ay? Vigsaats = FP 11S x10 a Vs sente = 1000 x 1.1510 = PV pssnt =115%10" g=1.15x10" pg ‘Years 1990-1999: mean win pereentage is 0.513 with a SDOM of 0.020 Years 2000-2009: mean win percentage is 0.497 with a SDOM of 0.012 Although the teams in years 1990-1999 had a somewhat better win-loss record, the difference is within the range of the SDOMs, so these data do not prove any significant difference between the two sets of teams (@) Since the surface area of Earth is 51x10" m?, we receive about 20000 «1000 S1x10" 3.910" kg per year per m”, (b) 0.25% of the amount calculated in part (a) is 9.810" tonnes per year per m? Using p,, = 3000 kg/m! we get m= pV =3000«8.18x10™" 98x10? x10" 2.45x10" 245x210" ke, = 400 micrometiorites per year perm? (b)£=5.39x10 5 Copyright © 2014 by Nelson Education Ltd Chapter 2—SCALARS AND VECTORS 1. @ F=30N, 6-318" ) F=21N,F =-21N (e) P=(21-217) N A parametric equation for the line is xet 44 So a veetor parallel to the force veetor is (1,3) with magnitude gi sn by A.unit vector parallel to the foree is therefore given by 1 3 3.162 3.162 ‘There are two possible force vectors, because the force can point in two possible j=0316/-0.949} directions along the line, either in the direction of for in the opposite direction. That is, £8 N #8(0.316/ -0.9497 VG-[-2)) + (0-27 + (2-0) = 5.74 m 7 @) G-2.1D &) 42,1) (©) (-24,11,22) (d) (205.10) 9. (&) Vector magnitudes are invariant with respect to translations or rotations of coordinate systems, 11. Student 2: Consider a vector in each plane directed such that they both form a 45° angle with the line of intersection of the two orthogonal planes. ‘These two veetors are not perpendicular to each other. 20,0), 4, =(0,2,0), 4 =(0,0,-2), B =(-1,0,0), 5, =(0,3,0), B =(0,0,-2) 4 Copyright © 2014 by Nelson Education Ltd 15 17. 19, 25. 21. 29. Disagree, if more than one component equals 1, the magnitude will be greater than L VI1.00, & = 2.007 4.007 1 3.00%, || = 29.00 5|- Jo =-5.00)+ 4.008, |i,| = J4100 = 1.007 + 2.00}, Disagree, The magnitude of the resultant is not equal to the sum of the radius vectors of individual vectors. Similarly, the resultant angle is not equal to the sum of the individual vector angles. Mathematically. the head-to-tail rule is equivalent to addition of vectors in Cartesian form. ‘The associative property of vector addition, that is (RR) -H +(% 4%) proves the head-to-tail rule for more than two vectors, ViaPs +h ‘The scalar product of two vectors is defined as the product of the magnitudes of the two vectors and the cosine of the angle between them. Because the definition involves the product of three real numbers, and the product of real numbers is commutative, the scalar product of two vectors is commutative. ‘The veetor product of two vectors is not commutative because the direetion of the cross product is defined using the right-hand rule, and the direction is opposite if the two vectors are multiplied in opposite order. Fi = $008(30") = 4.33 N F, =Se08(—40°) =3.83 N F, = Sc0s(-60°) = 2.50 N >F 4,337 43.837 +2.50k) N 21+27+4k. Itis difficult to accurately draw the vectors head-to-tail in three dimensions by hand. —(10sin(20°) + 7 + 20sin(30°)) = -20.4 m 1 c08(20°) — 20c08(30 [08 67F = 219m 20.0c08(360°—15°) =19.3 N, A, = 20.0sin (360° -15°) = B, =15.0¢08(35°) =12.3 N, B, =15.0sin(35°) =8.60 N 14.3 N, 25.0c08(125° 25.0sin (125°) = 20.5 N Copyright © 2014 by Nelson Education Ltd (b) 4=(19.37-5.187) N =(12.37+8.60)) NG o 14374205) N A+ B+C=(1731 423.93) N |= vis +239 = 295N ww(F2) S41° 173 29.5, $4.1") 6 ©) (19.3 i-3.18j) N, 8 =(12.3/+8.60)) N,@=(-14.37+20.5]) N B= 24-38 + =(-12.6/-15.7j)N Vi2.8 415.7 =201N o=ta(82) s1.3° 126. =837 Ge Sa + Li 0.3597 -0.598).4.0.717% 837 (b) Any unit vector d that satisfies the condition (3'-sj+ 68 perpendicular to the given vector. This question thus has an infinite number of (i +374 &) 9 (©) There are an infinite number of unit vactors parallel to the plane. Any unit possible answers, such as vector fi = af + 4) + ok whose cocfficicnts satisfy the relation 3+ 26—4e =0 Tye, Saf is aunit vector parallel to the given plane; an example is 32 +7428), (d)_ The veetor (3,2,-4) is perpendicular to the plane; thus the following two unit vectors are perpendicular to the given plane For this we compare the x, y, and = coordinates. @)F =-V2, Copyright © 2014 by Nelson Education Ltd 37. Let the vector be ft = xi + yj. Since this is a unit vector, it must satisfy the relation xayel a Since it is perpendicular (o the given vector, we must have 3x-2y=0 Q) Solving (L) and (2) simultancously gives the required unit vectors as 2- 3: 2 3 + Si feed = Ve ss’ 4B =(10)(3)eos20" 47 ‘The line in parametric form is 2645 So a veetor parallel to the line is (1,—2) with magnitude given by JP +(27 =V5 Aunit vector parallel to the line is therefore given by mL 2 5s 5 ‘The projection of the given vector on the line is therefore Daf 2 3)) a |+(4)|- |= 5 ols) . [ 4) Ay B= 61-22} -146 direction |x 8 =(10)(5) sin(20") =17 pointing in +y or (@) The cross product gives —147 + 7 Hence the given equation becomes era AMS jt Ri NB] x0 =F =W2,F,=14 Copyright © 2014 by Nelson Education Ltd ) 49. (a) (b) The cross product gives 207-44 &. Hence the given equation becomes wi-4R E+ he + BP, =0 ‘The transformation matrix for translation and rotation is cos(60*) —sin(60*) 4 sin(60°) cos(60°) 0 oo4 ‘The position of any point in the new coordinate system is then given by x) feos(so°) —sin(60") 4)/x" y|=| sing) cos(6o°) 5 |] »” 1 o o aja From the above matrix we can see how the components of a veetor will transform. =" 608(60°)— y"sin(60°)+ 4 (60°) + y"eos(60°)+5 This fact can be proved by definition of the vector product —see Equation 2-26 and Figure 2-18, This fact can also be proved using the algebraic definition of cross product and the fact that the scalar product of two orthogonal vectors is zero. The proof below is for vector 4; the same can be done for veotor : G=1107-1406 => a= V0" + 140° =178 m/s* Copyright © 2014 by Nelson Education Ltd 65. 67. sf_110 “ cos =518 a (ress) 7 xoos"( = je 8 178.04 (©) As done in part (a) > G@=1107-140% @ a, =a-( 140k) = 1107 a, =%- 1107 =~ 140% G0] = 1107-1408 ‘The length of & in the figure below ean be obtained from the coordinates of its two end points B? = (cos B—cosee)’ + (sin #—siner) = B= 2-2eosacosP—2sinasinB (1) The cosine law gives B? =1+1-2c0s(B-a) ‘Substituting here value of 3? from (1) we get 1+1-2cos (fee) = 2- 2cosercos f—2sineesin Simplifying we get cos(f— a) = eoscreos f= sinasin fr Let us look at the triangle formed by one CI-C-Cl combination. We can cut this triangle in half by dropping a perpendicular to the base from the vertex. Let us consider one of these triangles and find angle «it subtends at the vertex. ‘Then we will find the required angle as O=2 If we assume each side to be of unit length, then the base of this triangle will be 0.5 units and height 0.707 units (by using the Pythagorean theorem), 4-1 0-707 Os =O =2a@ =109,5° Sa 34.7 Copyright © 2014 by Nelson Education Ltd u Chapter 3—MOTIO! /E DIMENSION ST 1. (a) The two objects have the same average velocity (b) Batr=30s; Aatr=90s (©) Yes, when the slopes of their position-time graphs are equal, at about no (b) yes, atr=0 (©) A. Maximum accelerations: A, about 100 my 5. @ B ®) B (© same magnitude for both (opposite signs) da (©) same acceleration at around f= 130 s 7. (a) yes, at the maxima and minima of the curve (b) highest at ¢= 200 s, lowest at ¢= 30s and = 140s % b 11. a Gsize of the stone does not matter—the one thrown at an angle reaches less height and therefore attains lower speed as it lands on ground) gt 1d 19. In the limit of infinitesimally small time change, average aes instantaneous acceleration, Aeration becomes equal to the 21, Ata point where the tangent to the position-time graph is parallel to the line segment joining the endpoints of the position-time graph on the time interval of interest, the average velocity for this time interval is equal to the instantaneous velocity 2 Copyright © 2014 by Nelson Education Ltd 23. 21. 4, 45 41. 49, a d Only momentarily. If you are moving in one direction but have acceleration in the opposite direction, you will have zero speed for just the instant when you come to a stop before starting to move in the direction of the acceleration. a d © (since the displacement is zero) (a) Total distance covered 100 +1300 (b) Displacement = 1100 ~ 1300 =~200m Total distance covered = 75~ 75+ 80 = 230 cm Displacement = 75~80—75 = 80 em [down] 21-20 Speed Alms Displacement = 21-20 = 1m [down] v= 2 = 0.1 m's [down] io Average Speed = 2 =13m's 12 09 1.3 mvs [down] Distance between Paris and New York is x = $800 km Given: y = 149597871 km 2ar _ (2)(149597871) E 365.25% 24 07x10" kav/h = 29.8 knvs. Assuming the light arrives almost instantly. (343)(3.00) = 1035 m= 1.04 km Copyright © 2014 by Nelson Education Ltd 31 57. 59. (@) Displacement = 10-3=5 km (b) Distance Covered = 104+5=15 km (©) OP Fy = 0.67 kon ’min = 40 kwh Giyspeed = 15. = 2 tannin =120 koh 75 By inspection: (@) 105m, 70m, 40m (b) 10 m’s* 149.6 «10° m. 3 M0610 98.7 5= 8.3 min ce 3.0x10° =46x10" m 46x10" © 30x10 =1.5x10! (oe) += 9.4610" m x _ 9.46 x10" 3.1Sx10" © 3.0010 .99 x10" years By inspection Speed = 800 m/s, F = -800 m/s ‘The distance covered before the stone hits the surface of the molasses ean be calculated from =y 2g 2x9.81 The time taken to cover this distance can be calculated from —% = ah wry 2-4 y= === ge OBI Copyright © 2014 by Nelson Education Ltd 61 65. ‘The deceleration in the molasses can be calculated from vy, -¥, =at, or 2g 2x85 =x=- =25.0m ‘The total distance covered by the stone is 21.66 + 25 = 46.66 m y+, =1T34+2=3.735 aRtH ‘The total time is ¢ 2, 46.66 =y 12.5 m/s 3B By inspection: @) -14.2m/s (b) -22.3 m/s =¥, _ 21377-7882.9 384.22 10.705 m/s* 21377? - 7882.9" 2x35.12 ) 5.62108 f= 1713.3 km We have (for simplicity we will leave out arrows to represent veotors) Ye =n Fat @ a 1 wyatt tage walt sat @) M> Substituting the expression for ¢ into (2) gives Simplifying this we get Yea ia = Pea At Copyright © 2014 by Nelson Education Ltd 15 67. The average acceleration is equal to the acceleration due to gravity; that is 9.81 m/s*. 69. Given: a=1.45g, v, =0, v, =100 km/h = 27.78 m/s 2778-0 _1 95, a 145x981” y 71. The ball’s average acceleration is equal to the acceleration due to gravity; that is 9.81 m/s" The ball’ time of flight is -32.0-320 81 "The time for the balls upward flight, which is the = 6.5238 same as the time for the downward flight, is half of the total flight time, 3.262 s ‘The distance travelled by the ball during the upward motion is 2g -2x9.81 And the total distance is Ay, = 252.192 = 104.38 m Hence the average speed is Ax, _ 104.3 Speed= = Pe 6S 6.0 m/s Since the displacement is zero, the average velocity is also zero. oO 7B. Given: vy, = 0, 1, =100 knvh = 27.78 mis =¥ _27.78-0 @ Sag 1S ms 27.78" -0 Ay SEI AO 334.2 ®) Qa 2x19 = © Given: 15 km/h =115.28 m/s 115.28- 27.78 a 11.29 +t =2.46+7.75 =10.28 18s 16 Copyright © 2014 by Nelson Education Ltd 75. (a) The speed of the ball should be 3 mvs on its way up as well. First we calculate the initial velocity of the ball. vi-v =2ey fe = 2gy = JF —(2)-9.81)(14) =16.84 mis Now we can determine the flight time of the ball when it first reaches the friend. Sys =¥ _ 3.0-16.83 SON oi. 9.81 (b) distance =144 2-3 4 3-9 <4 92m <2x981" 25981 dist: 1492 average speed = SSCS AP? od nis 2.02 Jo-(2)[ 48.2 9.81j(L6 0-38.08 46.29.81 = 0.084 s = 84 ms 79. v, =, bat=0+(14x9.81)(1.3) = 180 mis 81. (a) Velocity after first stage is ¥, = af, =(5x9.81)(40)=1962 mvs Velocity after second stage is ¥, =¥ + af, = 1962+ (3.5x9.81)(25) = 2820 mis b) Ay ht fad =(2820.375)(90) + 4(3.2%9.81)(90") Copyright © 2014 by Nelson Education Ltd 7 (©) Ay tag =1(s.0x9:81)(40 92410" m Ay athe J =(1962)(25)+ 4(3.5 «9.81)(25') = 8.978108 m = Ag = 3.81% 10° + 3.924 x10" + 5.978 x10" = 4.8010" m Ay tedar 2 = 30010" oe basxaaie > t=117s=1.94 min 85, Time to reach the highest point is t= —=3.5 s ‘This is also the time to return to the surface from the highest point. ‘The distance covered from the highest point to the suface is given by vet hat + Losi Ss} = 60.086 m is is also the distance from surface to the highest point. Hence y,,,, = 260.086 = 120.17 m Since the ball reaches back to the original position, the displacement is zero. = Jina, 120.17 average spee 172 mis fot ‘The average acceleration is equal to the acceleration due to gravity: that is a = 9.81 m/s* The displacement is zero, because the ball returns to its starting point 87. (a) The distance travelled by the ball to the highest point can be calculated from ost _ (9.8) Fem "19.62 If the distance traveled by the marble is y,,,.,, then 4.90se 18 Copyright © 2014 by Nelson Education Ltd 89, 91 During time ¢ the marble will travel a distance of 9057? Log ert = 58 Substituting the expression in the previous equation into equation (1) we get 4.905¢' +4,905¢ =15 = 1-1.2364 8 = y, =9.81X1.2364=12.13 m/s (b) For marble we have vp-w =2ey Sv) -0= 2x98] x16 =>, 7 21771 mis Yea _17T1- g 98L ‘The ball reaches the highest point in 1.24 s, after which it drops down, We need the velocity of the ball after f= 1.8053 —1.2364 = 0.5689 s. Vip =9-81 0.5689 = 5.58 m/s =v, =5.58-17.71 1.8053 5 12.13 m/s ¥y =135—90 = 45 kavh relative speed = 135-90 = 45 awh, Since the initial velocity of the dropped ball is zero, its velocity after dropping is given by y, = 2gh = VI9.62h For the ball thrown upwards, the height is 56 —h. Its velocity is given by y, =? —22(56— i) = fv? —19.62(56 Since y= =v) -19.62(56—f) =19.62h Sy =33.1ms Copyright © 2014 by Nelson Education Ltd 19 98. (@) Sinve the elevator is moving down with constant velocity, for the person in the elevator the frame of reference is inertial. Hence, the time for the apple to reach the maximum hight is given by vy 6 = ——=06ls © 981 Since the apple takes the same amount of time to reach back the thrower, the total time is t, = 25061-1228 (b) Foran observer on the ground, the velocity of the apple in the upward direction is 6-4-2 mss. ‘The maximum height reached by the apple with respect to ground is ate 4 ree rory| ‘The time taken by the apple to reach the maximum height is = 0.204 m (4)(0.204)=0.816 m Hence the maximum height reached by the apple is 0.2044 0,816 =1.02 m Yona Pp FY, ‘This shows that if the elevator was moving up, the apple will be caught before it reaches its maximum height. ‘The maximum height reached by the apple above elevator’ floor is given by (c) The time taken by the apple to reach the maximum height is The time taken by the apple to reach the maximum height is y_ 6 a ba Ons And the total time to return to the thrower is = 2x0.61=1.225 4, 95. Given: v, =360 knv/h = 100 mvs vented 10051 2-5 (6x9.81)1 2 =776m 20 Copyright © 2014 hy Nelson Education Ltd 16) Copyright © 2014 by Nelson Fiducation Ltd a 99, 22 © @ © @ ) Differentiating x(¢) =x, cos(ot + §) with respect to time, we get xao(e =xosin (at + 6) Differentiating Fea(t)= —a,osin (cat + g) with respect to time, we get cox(at+ 9) a(t) = x, 0" cos (et +p) But x, cos(@r +9) = x(0) = a(r)=-0'x(r) ‘The speed of the speedboat is yy = OO 13.89 mis 12x60 ‘When the speedboat reaches the dock, the ovean liner has moved a distance of 24 ¥ 20000 (12x60) = 4000 m 3600 ‘Therefore the distance between the ocean liner and the speedboat is 6000 m The time taken by the speedboat to move a distance s toward the ocean liner is given by s 15.89 If this is the time when the speedboat reaches the ocean liner, then for the ooean liner we have 6000 -s 20000/3600 — 9000-8 os “~ 20000/3600 13,89 => s=4285.7m 4285.7 13.89 ‘Therefore the speedboat will take 5.1 minutes to return to the 08.6 $= 5.1 min displacement = 10000 — 4285.8 = 5714.2 m = 5.7 km towards the dock distance = 10000 + 4285.8 = 14285.8 m = 14.3 km Copyright © 2014 hy Nelson Education Ltd 101 105: When the apple is dropped, it is moving up at 9 m/s. ‘The maximum height reached by the apple is given by 2 2g 2-981 Hence the maximum height of the apple above ground is = 47+ 4.13 =51.1m =4.13m The speed of the apple when it hits the ground is v=2gh =V2.981xS1.13 =31.7 ms ‘The speed of the ball thrown up is given by eS yr 2gh Since the ball just reaches the baloony, v,, = 0. = ¥,, = f0=2(-9.81)(30) = 24.26 m/s Therefore the final speed of the ball thrown down is Vy = 2v,, 48.5 m/s =v, = ep —2gh = (48.5 —2(9.81) (30) = 42.0 mis The distance travelled by the ball thrown up as it passes the other ball is given by se 1 nt ge = s=24.26r-49" (1) For the ball thrown down we have 30-s=42r+49F = Q) Adding equations (1) and (2) we get 24.2604 421 =30 = r=045s Hence the velocities of the two balls when they pass are given by vy + gt = 24.26 -9.81x0.45 =19.82 mis v, =¥, gf =42.0+9.81%0.45 = 46.44 m/s 9.824 46.44 = 66.3 mis ‘Time taken by volleyball to reach the highest point is given by =? 1276 g O81 ‘The vertical distance covered by the volleyball is nee 734m 2g” 2x9.81 Copyright © 2014 hy Nelson Education Ltd 2B ‘The time taken by the tennis ball to reach this height can be calculated from 1 tt 5 8 = 734 =251— 4.9051" =1=0318 Henec the tennis ball must be thrown after 1.22-0.3 =0.91s 107. The distance travelled by the lemon as it passes the bread is given by + Sel => s=301-4.905" (1) For the bread we have 2-s=71+4.9057 2) Adding equations (1) and (2) we get 301+ 74 = 25 => 1=0678 Hence the velocities of the lemon and the bread when they pass are given by y= ¥, + gt =30-9.81 x 0.67 = 23.37 m/s y, =¥, + gf = 749.81 0.67 =13.63 m/s > Yyg = 23.374 13.63 = 37 ms 109, (a) Less than 0.5 kin/s per kilometre of altitude at 120 km, about 2.9 kus per kilometre of altitude at 92 km. (b) Since the metcor fragment is slowing, the forve of atmospheric friction must be somewhat greater than the forve of Earth’s gravity at an altitude of 120 km, and much greater at an altitude of 90 km. (©) ‘The density of the atmosphere inereascs substantially as you descend in altitude from 120 to 85 km. LLL. (a) about 30 m/ (b) about 13 mis? () about 15 mis*, 14 m/s*, and 13 m, 24 Copyright © 2014 hy Nelson Education Ltd 113. Approximate shape shown below. Sine this aeceleration-time graph is based on values estimated from a small velocity graph, the resulting graph does not exactly match Figure 3-41 and does not show the details of the sudden changes in acceleration between the stages of the launch. 35 0 0 100 200300 400 $0960 Tine (9) Copyright © 2014 hy Nelson Education Ltd 25 Chapter 4—MOTION IN TWO AND THREE DIMENSIONS 15, b, When the boat is pointed straight across the river ota (2) nae 60 Since tan = = Yow = 5, When the boat is pointed south of the target point J-nse d (Since both marbles are dropped from the same height and the initial vertical velocity of the rolled marble is zero, Therefore, the vertical distance traveled by the two marbles during the time the first marble moves horizontally for 1 m is the same.) 4 Piston = 24{ 4) an( 2) 0 ascn( 4) 44m z z n 4d. The bullet is fired at an angle of d=tan'( 22) L167 100, ‘The time taken by the bullet to cover the horizontal distance is, 100 1100 cos(16.7° During this time the bullet covers the vertical distance of y= (1100sin(16.7%))(0.095)— 4.9(0.095)' = 29.98 m = 0.095 5 Copyright © 2014 hy Nelson Education Ltd 17. => The distance from the highest point of the apple is = 0.02 m =2 om And during this time the apple would have fallen a distance of hi =4,9(0.095) =0.04 m= 4 om Sine fi = f+ typival apple radius, therefore the bullet will hit the apple on its way down, a The vertical distance covered by the projectile fired upward is he vsin(30°)¢— 4.92 ‘The vertical distance by which the second projectile will drop is h=49e Hence, the vertical distance between the two projectiles is he ht = vsin(30°)e ‘The horizontal distance covered by the first projectile is x= ve0s(30°)r ‘The distance covered by the second projectile is sew Hence, the horizontal distance between the two projectiles is x'— x= vt[1—c0s(30")] = The horizontal distance between the two will increase with time, a Gince the ball will be travelling with the same horizontal speed) Both will hit the ground at the same time. tangential acceleration: tangent to the bow! and pointing in the direction of motion, radial acecleration: pointing towards the centre of the bowl 3 © speed = (©) average speed = = 2.0 ms (©) No, these average speeds apply for only part of the time elapsed. The magnitude of the average velocity depends on the displacement. Copyright © 2014 hy Nelson Education Ltd 7 27. (a) Given: x= reos(8t), y= rsin(8/) We differentiate these to get « and y components of velocity, = 8rsin( 8) = v, (0.2) =-8rsin(8 x02) =-8.0r Similarly yey, =8reos(8t) = v, (0.2)=8re0s(8x0.2) =-0.23r (e) Fora particle moving in a circle, x* + y* =r Position: y= v7 For velocity we have [-8rsin(8e)} +[8reos(8)} = 6477 2 oar For acceleration we first differentiate the velocity equations. . S[-trsin(s)] =-o4r c0s(8") J [reos(8)]= 64rsin (8) 29, ‘Trajectory of the particle is given below. Given: x = 8%, y = q or fh) ae 16) VY” 286r = v(3)=80 ms 28 Copyright © 2014 hy Nelson Education Ltd ‘To calculate acceleration we differentiate the velocity to get 0046 m/s* 2 75 128 175 228 31. (a) To caloulate the distance travelled on the ramp, we first calculate the acceleration. 23-0 =46 mis s75m 2a, 2x46 ‘The distance travelled on the eurve is given by _2ar_12in “4 3 hae =190.1m '7.5+190.1 = 248 m (b) For velocity we need to calculate displacement Let us have our coordinate system centred at the starting point of the ear. 7.5 +121.0 = 178.5 m 21.0 m 7 Ssin(37°) = 34.6 m ‘The x-component of the end point is The y-component of the end point is » ‘The z-component of the end point is Hence the displacement is D=y(I178. (121.0 Next we need to determine the time taken on the curve. First we caleulate acceleration. 2190.1 Copyright © 2014 hy Nelson Education Ltd 29 =>, Pha 5412721778 Hence the average velocity is given by D284 a 12.3 mis 37° up and 35.9° towards the end of the curve “117 0 ‘ © a =" =0.40 m/s? 37° up and 35.9° towards the end of the eurve 17 Wow = 2gh => vy, =¥2x9.81x0.7-7 =5.9 mis 35. vy =12m’s My mate + 2gh = SOI xOSIXT = 11.7 mis vy fe ty, ei 37. (a) We know that each time zone is 15° longitude apart Given: d = 2570 km 3-4. 2570 Ray 671 ‘The time zone difference between the two cities is either 1 hour or 2 hours. 7 mis = 0.4034 rad = 23.1 (b) Assuming that the time zone difference between the two eities is 2 hours, the plane must reach Libreville within 2 hours. 2570 => 1285 kaw/h 39. Given: v= 830 km/h = 230.6 m/s y v mom, te ee re tg = OS 417 i3x9.81 30 Copyright © 2014 hy Nelson Education Ltd 41. Given: 0 = 30° =0.524 rad. @=0 2.875 rad/s = 2875-0 _ 7 987 rads? 20 2x 0.524, Sg, Sar = 7.887% 0% = Pe 8 a at! rads 0.045 45. Given: 0 =180°-30° = 2.618 rad 12 3.0 rads = 2.0 rad/s 4 g. a a = 0.955 rad/s* ~2x261: cur = 0.955% 4.0= 3.82 mis* 4 =~ © s60ms! Sa? ta} = ¥3.82" + 16.0 = 16 mis* (3.82) 16.0 } @= tan" = 13.43° with respect to radial direction 47. ‘Tangential acecleration is given by 59 mist 2S 2(3x2ex0.72) Copyright © 2014 hy Nelson Education Ltd 49. Si. Required condition is: a, = a 0.59 = y, = 0.65 mis Time to achieve this velocity is 0.65 _ oe Since the horizontal speed of the acrobat is equal to the speed of the ear, she will land in the parade car. Given: v, =10 km/h = 2.78 mis Time to land back in the parade car is given by 278 9.8L S xvi =10%0.57=5.7m 0878 Given: v, +, = 2.6 mis 3 =(%, -¥, }(5x 60) ay 3a = (v, ty. (5x60) @) Adding (1) and (2) we get % +, = 300(2v,) But x, +x, = 500 83 m/s », =2.6-0.83=1.8 m/s Using the Pythagorean theorem, the horizontal distance between the archer and the watermelon is 90" — 30* = 84.9 m ‘The shooting angle is therefore given by = tan ( 23_)~ 22 4° 84.9) Copyright © 2014 hy Nelson Education Ltd 35. 57. ‘Time taken by the arrow to cover the horizontal distance is 849 160 cos(22.4°) ‘The vertical distane travelled by the arrow during this time is (160.c0s(22.4°))(0.57) — 4.90.57 In this time the watermelon covers a distance of 2m om ‘This corresponds to a height trom the surface of h, =30-1.6= 28.4 m Hence the distance between the arrow and and watermelon is h,-h, =33.2-284= 4.8m ‘The horizontal speed of teddy bear with respect to the cannon is y= 6005(37°) ‘Therefore the speed of the teddy bear with respect to the ground is ¥, = 6008379) +2.7=7.5 ms Given: v, = 34 km/h = 9.44 m/s ‘Therefore the distance travelled by the eagle in 1 s is x, £9.44 m Distance travelled by the eagle in time ¢ after the stone is thrown is x’ =9.4dt a During this time the stone must travel a distance of 944+ x’ 40,5 =v, cos(38°)t Using (1) this becomes 9447 49.904= 0.7880, (2) The vertical distance the stone must travel in time 7 is 2B=v,sinG8-4.97 GB) Solving (2) and (3) simultancously we get an imaginary result. Hence. the given seenario is not possible, Copyright © 2014 hy Nelson Education Ltd 59. Given: y, =39 km/h =18.8 m/s The distance travelled by the bird is x= 18.8(r+4) a The horizontal distance covered by the bullet must be x=(378e0s0)t @ The vertical distance covered by the bullet must be 25 = (378sin O)r—4.97* e Solving these simultancously gives 0=20.9° 61. ‘The time taken by the papaya to hit the ground is, Therefore the stones hit the papaya after 1.70.9 = 0.8 s ‘The vertical distance travelled by the papaya during this time is A, = 4.90.8 = 3.1m ‘This corresponds to a height of =14-3.1=109m For the stone thrown from a distance of 11 m, we have ye thni37 ms 08 10.94.4.9x0.87 7.5 ws 2 08 Sy, =175-9.8x08=9.7 mis 3.P 490? For the stone thrown from a distance of 16m, we have 6.8mis vy =18=200mi 08 10.94 4.9 x0.8° 17.5 mis 08 = vy =175-9.8x0.8=9.7 ms =v, =V20.0 49.7 =22.2 mis 34 Copyright © 2014 hy Nelson Education Ltd 65. (@) Given: y = 607? = 77+ 464120 For vertical velocity we differentiate this with respect to time. =v, =120¢-210 +4 At maximum height, vertical velocity is zero; that is 1201-20? 44=0 = raS75s = Yes 60(5.75)' —7(5.75)' + 4(5.75) +120 = 796 m (b) ‘The time taken by the missile to travel 120 m below the point of launch can be calculated from y= G01 — 71" +4r-+120 = 120 =1=251s The total time to travel is therefore fy) =5.7545.7542.51= 14.018 The horizontal distance covered during this time is 1400-+314.01= 1.44 km ‘The magnitude of the vertical velocity at this time is {:20(14.01)—21(14.01)' +4] =2.44 kms From the expression for horizontal displacement, we see that the horizontal speed is constant at 3 ms (©) Wesce that the acceleration in the x-direction is zero. Therefore, the total acceleration is the acceleration in the y-direction, which can be obtained by differentiating the expression for velocity in the y-direction. = a= 120-420 Since aeveleration varies linearly with time, it will have its maximum value right before the missile hits the ground, The component of velocity in the xy-plane is, y,, = 140005 (90° = 29°) = 140cos(61°) The x-component is then given by ¥, =140.cos(61°)e0s(40°) = 52.0 m/s Since there is no acceleration in the x-direction, the x-component is 2.0 Copyright © 2014 hy Nelson Education Ltd 35 67. 69, 36 Similarly, for the y-direction we have ¥, = 140 cos(61°) sin(40°) = 43.6 m/s => y= 43.60 ‘The initial velocity in the =-direction is y, = 140sin(61°) = 122.4 m/s ‘Since there is acceleration in the =-direction, we have 122r-4.91* (a) The maximum height reached above the tree house is 35sin(37°)) 2x98 Hence the maximum height from ground is /,,, = 4.0+ 22.6 = 26.6 m =22.6m (b) ‘The time needed to reach the maximum height is, 358in@7" 98 ‘The vertical velocity of the tomato right before it hits the ground is given by =215s Vy, = f2GTigg: = V2X9.8%266 = 22.8 m/s The time taken by the tomato to reach the ground from its maccimum height is The total time to reach the ground is f= 21542.33= 4485 The total horizontal distance covered is = (35e08(37°) (448) =125 m (©) Since there is no acceleration in the x-direction Seos(37°) = 27.95 mis v= 27.9 +22.8" =36.1 m/s (a) The distance between the two points at calculated from the haversine formula is 10494 km. The plane moves with a speed of 10494 = 954 kin's in vee Copyright © 2014 hy Nelson Education Ltd (b) Beijing is 12 time zones away from Toronto so the passenger will have to adjust the ‘watch by 12 hours. ‘The plane has swept an angle of 715° = 105°. ‘The distance covered by the plane is therefore given by s= ral Ran +600) = 2 2a(6371000 + 600) = 1.1677 x10" m = = 11677 km Since passengers had to adjust their watches by one hour, the plane must have taken 8 hours to get there. = y=2=1460 kwh 8 (a) F(t)=(2+0.Ssin(0.42))F4 (3-40.5sin(0.41)) 7 +2°% = F(O)= 2437 (b) at = ¥(r)=0.2c0s(0.41)i + 0.2c08(0.4e) 7 + 4th => ¥(0.2) = 0.2c08(0.4«0.2)F + 0.2008 (0.40.2) 7+ 40.26 = ¥(0.2) = 0.1997 + 0.1997 + 0.86 = |7(0.2)| = 0.199" + 0.199 40.8" = (©) #(0.2)=(2+0.5sin (0.4 x0.2))i +(3+0.5sin (0.4 x0.2))7+2(0.2)' & => F(0.2)= 2.047 + 3.047 + 0.086 85 m/s F(0.4)=(2+0.5sin (0.4 x0.4))i +(3+0.5sin (0.4x0.4)) 7 +2(0.4) E 08) + 3.08) + 0.32% Copyright © 2014 hy Nelson Education Ltd 37 38 (A) ¥(0.4) =0.2c05(0.4x 0.4)? + 0.2e08(0.4x0.4) 744% 0.46 => ¥(0.4) = 0.1977 + 0.1977 + 1.66 0.4)|= V0.197 + 0.197 +1.6° =1.61 mis - (0-4)-¥(0.2) _ 1.61 0.83 _ 3 87 még? 04-02 02 (@) S(0)=2F+37 (1:1) = (24 05sin(0.44.1))7+ (34 0.5sin(0.41.1))j +204) & 1 243.2174 242k => D= (221-2) +(3.21-3)' +(2.42-0) =2.44m (Q) Sinoe.x and y are sinusoidal functions of time and is a quadratic function of time, the particle follows a helical trajectory. Given: 0 =3 rov/s = 32 rad/s Position of A with respect to B is 2r-rcos ot yersinot asin ot rercos on Syero 88.87 (tangential acceleration is zero) Given: R(t) = 1426.5" h(t) = 658.8—0. 1847 +18.37 —3457 => R(60) = 1426.5" = 8.13 km (60) = 658.8 — 0.154 x 60" + 18,3 » 60? —345 x60 = 12.6 km For velocity we differentiate the given position funetions with respect to time. y,()=41.370" =v, (60) = 41.3728 = 235.7 mis y, (t)=-0.4620 +36.6r—345 = v, (60) =-0.452x 60" + 36.6 x 60-345 =187.8 m/s => y= V235.7? 4187.8 = 301 m/s Copyright © 2014 hy Nelson Education Ltd 81. For avceleration we differentiate the velocity functions with respect to time a,(t)=1.2e™ = a,(60)=1.22°%™" =6.8 mis? a, (t)=-0.9241+ 36.6 =a, (60)=-0.9244-+36.6 =-18.8 mis? = v= V6.8 418.87 = 20.0 mis? (@) Given: v, = 43 km/h= 11.94 m/s The horizontal velocity of the acom with respcet to the ground is Vig = 11.944 4oos(34°) = 15.26 m/s In 1 s the acom travels a distance of x, $15.26 <1 =15.26 m In 1s the car travels a distance of x, S194 x1=1194m Hence the acorn lands 15.26 11.94 =3.3 m ahead of the car 1.944 4cos 34° = 15.26 m/s = 4sin34° = 2.24 mvs v, = V15.267 + 2.24 =15 mvs () The acom makes an angle of 34° with respect to the horizontal after being () thrown. Therefore it will make the same angle with the horizontal right before landing. The acom’s velocity will also make the same angle with the horizontal right before landing. ‘The distance covered by Crunchalot before being smashed is, xaLde @ The horizontal distance covered by the ball during this time is 15-x=(Toos@)t —Q) ‘The time taken by the ball to reach its maximum height is Tain O 981 Since 1 = 21 Sra 1 43sin? @ Equations (1), (2) and (3) can be solved to find 6. = 8 =60.6° Copyright © 2014 hy Nelson Education Ltd 39 85. 40 (a) The particle is at x Oat = 20cos(0.41) = v, (12) =-20 x 0.4sin(0.4 x12) = 7.97 vs y= 20sin(0.40) =v, (12 20x 0.4c08(0.4% 12) = 0.67 m/s = v= 797 +007 (b) Since tangential velocity is constant, the average acceleration (12) = 20¢08(0.4x12) = 1.75 m y(12}= 20sin(0.4 x12) =-19.92 m ( 9.92 175 x(20) = 200s (0.4% 20) =-2.91 m y(20) = 20sin(0.4 20) = 19.79 m 19.79 n (SB) The displacement is given by .0 m/s tangent to the circle =, ) =-8498= 27502 wat he pie sess =4 98.37° wort. the positive a-axis D=(275.02-98.37)| |(20)= 61.66 m 180, sl $6 ams are The direct distance is d, = ‘The actual distance covered is d, = /300" + 244° = 386.7 km 4, 224.2 km/h The actual speed of the plane is: v, ‘The current bearing can be found from the cosine Law. 66.2 + 386.7’ — 2(366.2}(586.7)cos @ 1° sa ‘This can be used to determine the speed of the air. (224.2)(270) c0s(4.1°) = 49.1 km/h [224.2% +2 Copyright © 2014 hy Nelson Education Ltd 87. 89, ‘We will now use the sine law to caleulate the angle between the direct and the air velocity vectors. 386.7? =34? + 366.2 —2(34)(366.2)cos ff = pe12s° We ean now use the sine law to calculate the required bearing, 49.1 ‘sind sin(180°—125°) = 6=103° ‘The distance covered by the pheasant in time 1 is x= 28 The horizontal distance covered by the bullet in time ¢ is x=(332c0s)t = (332c0s0)1 = 28 = 0 =85.2° with respect to the pheasants dircetion of motion (a) By inspection: 0.36 km/s (b)_ By inspection: 0.42 km/s, 0.14 km/s, 2.0 km/s, 2.5 kas (©) The small inerease in the slop of the graph for the next 250 s indicates some radial acceleration, after that the curve is almost straight, so there is little or no radial acceleration after 1750s () Since the graph shows altitude versus time, the slop does not reflect any tangential component of the velocity. . ‘The horizontal distance the ball must cover to clear the tree is veos(47°yt = 40 ( ‘The vertical distance the ball must cover to clear the tree is vsin(47?y— 4.917 =12 2) Solving (1) and (2) simultaneously gives: v=234mis ‘There is only one answer to this question since angle, horizontal distance and vertical distance are fixed. Copyright © 2014 hy Nelson Education Ltd 4 Chapter 5—FORCES AND MOTION Loa 3b Gince acceleration is negative) 7. (since the friend does not move) 9. The 16 kg mass accelerates down while the 4 kg mass accelerates up. Md b (since the person being pulled is moving with constant speed) 15. d (since the net force is downward) Ino 19. a 21. (since the sled does not move, there is no net force acting on it) 23. b. F= mat 200-/ (f is the foree of friction acting on the lion) 120 x5 ~ 200-200 = 600 N 25. (@) F= mg = 60x9.81= 588 N 300+ 60)(20) . . ——___+— = 13600 N =13.6 KN = 13000 N=13 KN r 40 (0) P= Mg =1300%9.8= 12740 N=12.74 KN vertical 27. a (since no net foree is acting on the buckets) 20. © 31. a (draw free body diagrams for each bead) 42 Copyright © 2014 hy Nelson Education Ltd 41 a Gince the mass on table moves half of the distance covered by the hanging mass because of the movable pulley) b (a) For this we will use: # = 1473.09 = 4.93) +630) 1.87) = 4.64) b) 667 + 4.76) @ (b) F =14008(87°)i + 14 sin(57°) j = 7.67 + 11.77 F, =8c0s(236°)i + 8sin(236°) j = 4.57 - 6.67 Feh+h+k =6.2 4117 =F (e) B=(4-9)F (2414) j +(-5 +11)K = 57 4167 + 6k 2lsin(32°)cos(27°Yv + 21 sin(32°)sin(27°)j + Zleos(32°Vk 997+ S1FHITRE 7 sin{160°)cos(295°)i +17 sin(160°)sin(295°)j +17 cos(160° yk 251-5.3}—1606 F+ i =124)—0.27-1.88 @ =p vi -0=2 x14 x10 =v, =20.1 mvs Copyright © 2014 hy Nelson Education Ltd 8 43. Given: y, = 70 km/s =19.4 m/s =v 0-19.47 2a —2x36x9.81 0.54 m = 54,0 em 45. (a) Given: v = 110 km/h = 30.56 m/s (1300 + 71+ 79)(30.56 63.0 21.5 x10? N=21.5 kN (71)(30.56)" 63.0 () 050.N 1300 + 714 79)(30.56)" 63.0 © = 21.510? N=21.5 kN 47. (a) Balancing forces gives ma=mgsin® =>a=gsin0 down the slope + grain? (b) Balancing forces gives ma= nig sin O— ug cos = a= g(sin@— 10080 ) down the slope + at(sin@— 10080 ) (©) Balancing forces gives Tsind= = , Toosd = mg y - gtan toward the contre of the eirele Also, v = fer tand (d) Balancing forees gives F cos 0 —mg sin @ = ma _ Feos = mgsin? +(eatoms *) upward along the slope 44 Copyright © 2014 hy Nelson Education Ltd © a) ) aigeost Balancing forces gives Feos@— 44, (mg —Fsin8) = ma u, (mg —F sind) along the dircetion of motion Jn both cases the radial component of the acceleration is directed toward the centre of the circle, i) When the mass is at the top, balancing foress gives the minimum velocity roquired to maintain circular motion my’ (ii) Conservation of energy gives 2 ° E+ (mg)(2r)= ae Balancing forces gives 2" cos = mgsin 8 sve rgtand Acceleration is directed toward the centre of the curve Copyright © 2014 hy Nelson Education Ltd 45 49. (a) a 51 55. 46 (i) Balancing forces gives TY cos D+ 1 [anos mg cos |= mgsin O r F sre Acceleration directed toward centre of the curve. By Newton's aoad law, the magnitude of the net foree acting on you is F = m,a=81(10.1)=820N (b) F=m,a=2300(10.1)=23 kN (@) F=m,(g+a)=81(9.81+10.1)=16 kN (@)_ P= m,g =2300 «9.81 = 22.6 kN Balancing forces on the cat gives Fe mg+ma=2.1(98143.2 ve 3.2 2ax 2x15 Balancing forces gives 34 m/s? mesin@ = ma +) = sin (254) - 2.08 g 9.81 47-0 2av 2x31 Balancing forces gives =O =si = 0.36 mis* mg sin amg cos = ma _gsin—a _ 98Isin(37°)— 0.36 geos ORLeos(37P) = =071 Copyright © 2014 hy Nelson Education Ltd 57. (@) Balancing forces gives 59. 61 65. M,g-T=M,a T=Ma Solving these equations simultaneously gives MyM, M,+M (b) If lighter mass is hanging then, there =T=Mg or, -(FEpar-isen 2412 7, =2x98) 9.6N (@) Balancing forces on mass m, gives m,g-T= ma (b) Setting 7 = 0 in part (a), we obtain a= g. Thus, the system falls freely. and so the tension in the upper rope is also zero. F120 900 Nim x OL ay = 28 _ 10898) _ 9.082 m= 8.2 om & 1200 Force balance gives mg —kx= ma _12x9.81-123.0 - 03 Sk 272. Nim (a) ‘The no-slip condition gives my a= wing F = v= Jfugr = (0452 981x3 =3.6 mis Copyright © 2014 hy Nelson Education Ltd a 61. 69. 1 71. 48 (a) Balancing forces for the no-slip condition gives a BY cos(23°) = mg sin(23°) = v= Jer tan(23*) = 9.81 x 72tan(2 7.3 m/s = 62.3 km/h ) &, = Nsin(23°)=) (5 sn (23°) = mg tan(23°) = 72 «9.81 tan(23°) = 300 N, directed horizontally inwards towards the centre of the virele ‘To prevent slipping, we must have ma= mg sye oA = =051 Given: » = 67 kav =18.6 m/s my? _ 30* 18.6 =450N . Balancing forces gives Tos = mg Tsino= = v= Jertand = ,{(9.81)(6) 7 mis . Balancing the torque on the gymnast of mass m hanging with a rope of lengh J, we get 2" jeos 8 = mglsin® With m as the mass of the spider, balancing forces gives BE cos(30") =f sinc) « mint) = mgcos(60°) r F v= 470 ms Copyright © 2014 hy Nelson Education Ltd 79. (@) Force balance for both masses gives mg-T=ma T-mg=ma =o =a= 0.75 mis? mm, The velocity of mass m, is V2K0-75x4 = 2.46 mis Therefore, the velocity of mass m, is vy, =-2.46 mis (b) P= 1,(a+g)=12(0.754981) =127N 81. The angle swept by the marble is 83. ‘The horizontal foree experienced by the block is +m F wo Force balance gives F'eosO—mgsind=0 2) Solving (1) and (2) simultaneously gives, F=(m+M)gtan? meM 85. The horizontal force experienced by the block is 7 1 m+M o Forve balance gives F'cos0— u(F'sinO + mgcos0)—mgsinO=ma (2) Solving (1) and (2) simultaneously (m+Ar [a+ g(sind+ woost)] cos usin O Copyright © 2014 hy Nelson Education Ltd 49 87. We will consider that the hanging mass is moving down with aeceleration a and at the same time the mass on the table is moving to the left with the same acceleration, Balancing forees for the hanging mass gives, 3mg-T=3ma (1) Balancing forces for the mass on table gives =ma Substituting this in (1) gives 3 a-58 ‘Now, table must be moving with the same acceleration. 3 3 Fa(M 43m mam (+m) 2g 738(M +m) N 89. (a) In the absence of a force on B, mass C will be moving down with an acceleration of 2 =3.27 mis* downward m, +m, Therefore the force on B must be able to provide an acceleration of = 343.27 = 6.27 m im, +i; Ja"=19.6 N =F (b) (+ mz)a"=19.6N m,(3)=3.0N BK, F,=m,(3)= 6.0 N The force is different due to a difference in total mass, 91. @ N=Mgcosd (b) The normal force in this case is the sum of the nonnal components of centripetal force and the weight of the car. N=Mgcos 0+ F, sind Since car does not slide: Mgsin =F. cos? = N= Mg(cos 0+ tan) (©) N=Mgeos+F sind Since ear does not slide: Mgsin9 = 0s 0 + se(Mg cos + F, sin) mg =N-—"2 _ cos 0+ jssind 50 Copyright © 2014 hy Nelson Education Ltd 93, 95: 97. 101 Force balance gives T cos0+ 11(mg cos + sin 8) = mg sin @ (sin - 0080) “cos 8+ usin 8 mg Conservation of energy gives jm? = mgR—mg(0.2R) Am 2X08x9.8 = 15.68 ms? Lo prevent the block from slipping, the applicd force must have a component ‘equal to the gravitational pull on the block down the slope and opposite in direetion. F, =mgsin@=me R We first balance force on the hanging person. mg-T = ma, a Similarly, for the movable pulley we have Ma, =T @ Solving (1) and (2) simultaneously gives m 56 mtM "36478 acceleration of the sitting person is half that of the hanging person due to the movable pulley. 9.81 = 4.10 m/s* a7 = a, = 2.05 m/s* = \2a,y =V2%4.10 «19.0 = 12.48 m/s (a) The forve of friction is responsible for circular motion, mv? _ 69x11 r 34 54.6N Copyright © 2014 hy Nelson Education Ltd 31 103. Given: v= 110 kwh =30.56 m/s = 2230.56" 5 80 enis? 2x e120 Sinve the book is just starting to move:—ma= jung a _ 3.89 = --2-——-0.40 eee O81 2 a as 52 Copyright © 2014 hy Nelson Education Ltd Chapter 6—ENERGY TT 1. Yes, since the work depends only on the difference in energy at end points, T since work is being done on the spring to stretch it 5. Negative, since the spring applies the force in a direction opposite to the direction of motion 7. Yes, if the displacement is in a direction opposite to the applied force. 9. Can increase or decrease depending on the direction of motion and the applied force. 11. ‘The total work done by Earth’s gravity is zero, sinve the satellite is moving along the line of constant potential Zero, since the work done to stop the puck is exactly equal to the work done to hit the puck back. 15. Work done by gravity before the weight hits the scale is Wy, = mgh=(2)(9.8)(0.43) Velocity of the weight as it hits the scale is {(a(9.8)(043) =29 mi V(2)(9.8)(0.43) = 2.9 mis v=/2e8 ‘Total work done on the weight for the last 5 em is (2)(2.9) =-8.431 21 b 23. Since the child is moving at constant speed, we must have emg cos = mg sin => ston? Since the child is moving at constant speed, the work done by gravity must be equal and ‘opposite to the work done by friction. 25. b 2. Copyright © 2014 by Nelson Education Ltd 53 4. 34 35. @) W, - (ince the weight starts from rest and stops at the end) e mgh= (0.4)(9.81)(0.45) = () W =mgh=(0.4)(9.81)(0.45 (©) Woyg =AK =0 W =-20%0.9=-181 @ W aan Lo? Sey 36) (b) I = meh =(8)(9.81)(0.5)=39.21 (a) ‘The initial velocity required such that the sandbag reaches the 7 m height is, 10.4 m/s (b) W =-mgh=—(2)(9.81)(1.5)= 29.45 (a) Foree balance on the 16.8 kg mass gives Mg -T =Ma a) Similarly, force balance on the 4.8 kg mass gives T-mg-kk=ma (2) Solving (1) and (2)simultancously and substituting the given masses and distance travelled, gives a= $.44—0.021k ‘The velocity after moving a distance of 45 om is therefore y(2)(5.44—0.0214 )(0.45) v= 59-0016 vay Ras = (b). The speed is maximum when k = 0. Nyx = V49=0 = 2.2 més (©) v=V¥9— 0.01996 = 1.8 mis Copyright © 2014 by Nelson Education Ltd 45, 41. 49. (a) Forve balance gives F = , where = amg is the foree of friction = img =F P Sas mg (b) W= Fa (©) More work is required since the force has to overcome the gravitational pull as well. If the object still moves at constant speed, then force balance gives F=mgsind 2 W =mgdsind (d) In this ease, force balance gives F=mgsin0 = ma = F= mat mesind = W =md(a+ gsin®) More work is now required as compared to part (¢) (e) In this case, force balance gives F —mg sin — mg cos = ma = F = matmgsind+ png cos 0 = W =md(a+ gsinO+ sgc0s0) More work is now required as compared to part (6) 1,1 1 2 Wade dmg 0-4 a zm 5 3 (22)(4) 176 J We=Fd WN y 4.10 N d 13 1 = Feose = r-F-184 _y sco cos cos(42°) Since speed is constant, force balance gives F=f, where f= mg is the force of friction But F, =F cos(27*)cos( 41°) = 60 ¢0s(27°)eos(41°) = 40.3 N = pumg = 40.3 40.3 w= =0. ** Tp 981 Copyright © 2014 by Nelson Education Ltd 35 Sl. 53, (a) ‘The displacement veotor is d= (S—3)i+(4-2)j+ (341k = 27-674 4k Components of the force vector are F, = F eos(32*)cos(1 95°) = 170 cos(32°) cos(195%) = -139.26 N F, = Feos(32°)sin(195°) = 1 70cos(32")sin(195°) =—37.31 N F_ =F sin(32°) =170sinG2°) = 90.09 N Hence the work done is W =(-139.26; -37.31) + 90.09%) (2i-6j+48)= 31x10? J Since the particle starts from rest, the work done is (b) The displacement vector is 27-6] +48. The incline angle for the force of gravity can then be calculated from the direetion cosine. > yasrP ‘The incline angle is therefore 0 w = (139.267 -37, 31} +90.098 WW 37.7 = 32.3° 25-6} +48) + pmgcon(32.3 = W = 308,740.15 x1.1 9.81 0863: 307.10 W = AK =0 (since initial and final velocities are zero) 55. ‘The potential energy of the block gets converted into kinetic energy as it hits the spring. 56 ‘That kinetic energy compresses the spring and gets stored in the spring as potential energy 1 = tis? = mgh she? = me ‘The height h is given by A= 3sin(30°)=1.5 m 4 $40) =(10}(9.81)(1.5) = r= 0.93 m Copyright © 2014 by Nelson Education Ltd 57. (a) V = mgh= 279.81 x15sin(19°) = 1.3 KI (6) Woy, my’ = Rg cost a Energy conservation gives Linv? + jung cos(@R)= mght= me(R- Roos) 2) Here OR is the distance travelled before leaving the surface Solving (L) and (2) simultaneously gives Gc0s 8 + 2yu6)c0s 6 = 2 1 can be determined from this equation 95. Given: w = 62430 tb = 62430 x 4.45 = 38500 f= 17830.8 m U = mgh = 277813.5 x17830.8 = 4.95 x10" J 277813.5 N (a) Given: F = 223.450 Th = 223450 x4.45 = 208705 N Cruising speed can be obtained from ig =m? = mgh 3 igh = v= [gh = J2%9.81%17830.8 = 501.5 mis ‘The power is then given by P=By 23x10" W 208705 x 591.5 62 Copyright © 2014 by Nelson Education Ltd () © 277813 Give: TO mph e313 me, m= ZS aes kg Fav At maximum cruising speed we will have ma en? av @(313.4)° aa m 2R3R3 Here a depends on air density, drag coefficient and cross-sectional area. 3.50 m/s* W = mgh+ mah WF = 28348,3 17830,8(9.8 +3.Ser) 1x10" (9.843.5er) J Shs Copyright © 2014 by Nelson Education Ltd 63 Chapter 7—LINEAR MOMENTUM, COLLISIONS, AND SYSTEMS OF PARTICLES uw 1B. 15. 17. 19, 21 4. Since p = mv d, The final momentum is = mrsin0°V + mvreos@30} = Li + mj a, The total impulse is (10 cos 60") (0.1) (10c0s60")(0.1) =1000 N into the wall 0.001 0.001 1b. Conservation of linear momentum in the horizontal direction gives P, 605(60°) = p, cos(30°) ‘The centre of mass moves down as the pieces of the firecracker fall away from the needle. If the explosion is not symmetrical, the centre of mass also shifts to one side Yes, provided air resistance is negligible The speed of the ball just before impact is V2x98511 = 14.7 mis Momentum conservation gives 4 = 2eh my = FM+ my, 1 gAt=14,.7-9.80.02 = 14.5 mis ‘The speed is lower since the ball loses some momentum during impact. Copyright © 2014 by Nelson Education Ltd 4 45. . ‘The speed of the stone before hitting the ground is v= f2gh = ¥2x9.81x20 =19.8 m/s = p= 0.06*19.8=1.2 kg m/s ‘The acceleration of the athlete is given by 9-9 45. 0mis? + 020 = |F|=74x45=3.3kN (a) Given: R= 55 balls/min == alls/s, v=117 knv/h=32.5 m/s F = R= (0.088)(32.5) ) F L8N 55/60 Momentum conservation gives P= FAtt p, my, = mgAt+ my, = y, = gAr+0.99y, oo1 sare g But = yeh = aa 2S FORTAIO =0014 s= 14 ms Copyright © 2014 by Nelson Education Ltd 65 41. 49. 55, 66 away from the mass removed a * from the position of the removed mass orr ‘We will fix the origin of our coordinate system at the centre of the largest mass. =(0,L), 5, =(L.L). 5, =(L,0), 0,0) +2mr,, +3) +3, +4, =(0.52,0.32) m+ 2m m+ 4m vy, -22-2.9 mvs 63d Momentum conservation gives Ce 40 404210 First we will caloulate the time it took the balloon to reach its maximum height of 0.9 m. 2)=0.35 mis Since the vertical velocity is zero at maximum height, the initial vertical velocity is, vy, =\P2gh Also, v, = gt ‘The total time taken by the centre of mass to hit the ground is 2¢ => v= 2.3x2x043 = 1.98 m Momentum conservation in the x-direction gives my, = (nm, +m, vos O w ‘Momentum conservation in the y-direction gives im, +0, )vsin @ Solving (1) and (2) simultaneously gives t mm, ve lan Y +(my J = 4.1 mis Copyright © 2014 by Nelson Education Ltd 57. ‘The height reached by the pendulum is hh =0.92-0.92c08(27)= 0.1m ‘The speed needed to reach this height is determined by conservation of energy. bay mgh Syel4ms Conservation of momentum gives =(m, +m, )v (oases Sy Jos = 8636.0 m/s =8.7 kmis oon): 59, The fractional loss in kinetic energy is given by Applying momentum conservation in the direction we get my, cos = mv, cos 8" cos 0 ae vy c08” Substituting this in the above expression for fractional energy loss, we get 61 K=Ki4+k, Copyright © 2014 by Nelson Education Ltd or Energy conservation for the 3 kg mass gives diay = 2 His 2 e ( (0 8)=6.28 m/s VG Energy conservation before and ater collision gives i. x 1 Lit Lat bed 7m zm ty mN = WW? = 73.95 aw Momentum conservation gives my, = myf +m, =>y,-f 11.78 2) Solving (1) and (2) simultancously gives y, =9.0 m/s 65. We will assume that the collision is perfectly collinear and that the incoming ball is lighter than the stationary ball, Also, the incoming ball moves in the opposite direetion after collision with twice the speed of the second ball. Momentum conservation gives my, =— my, +m, Solving (1) and (2) simultaneously gives m, n= 5m, 67. Momentum conservation gives (Im +:m,v= mp! m, (2) a 68 Copyright © 2014 by Nelson Education Ltd Energy conservation gives 2 =(5) 100m = © (1.2}(660.5)= 792.5 N 71. For this question we will assume that the initial mass of 230 kg includes the mass of water. (@) The water flow rate is given by o-(8)(_)~933%107 ms (6 )Lino0 Q _9.33%10" ve <= 2.43 m/s A 0.035 Also, R= == 933 6 = 0.0986 m/s? = 9.9 om/s* M (b) The mass remaining after 4s is 2309.33 x 4 = 192.68 kg st 230 192.68, Copyright © 2014 by Nelson Education Ltd 69 (©) ‘the time taken by the squid to expel the last sixth of the water is given by 56 6x93 We can now apply standard rocket propulsion equation at the start and end of this last second. 10s oan 230 | 7 2.430) ws = 0.551 ms 6 Je oor 56 a7 ¥a_0.678-0.551 = 0,127 mis’ =13 om/s* t 10 73. We can apply the centre of mass formula, 7x10" sa- 100100 =7x10" km (this is the major axis) 5x10" 6 EP cos( a 48° 0.396 m=39.6 om 77. Conservation of momentum gives gig = MVS + MG 0.005 x 265 —0.005 «235 92 Since the watermelon will fall freely, its centre of mass will have 016 m/s =1.6 cm’s acecleration equal to g = 9.81 m/s*. 79. (a) Energy conservation gives 1 kt tiny 2 2400 17500 Im 70 Copyright © 2014 by Nelson Education Ltd 81 85 87. 89, (>) Momentum conservation gives (m, +m,)», 2400 2400 +7400 Jere 73 mis Momentum conservation gives my, =(m.#m,)», =y. (RS emy-s9m Momentum conservation when the backpack is thrown gives m, (m+n, + m,)y. % sy -(—§ _)20-0.192 ms 72465419 = y= 0.192x1.5 = 0.288 m= ‘When the other person catches the backpack, the canoe will move the 9 em same distance in the opposite diteotion. So, the total distance moved by the canoe will be 2 x 2° 58 om. Given: y, = 1400 kin’/h = 388.9 m/s Momentum conservation gives Ly =L0my, — my, (1113000}{ 4000) + (13000)(388.9} 1013000 => Av = 4438.9— 4000 = 440 m/s >, = 4438.9 mis ‘If we consider the astronaut to be a rigid body, then she must also rotate by 180° At the highest point there is no vertical component of velocity, s0 =y After the explosion, the lighter mass falls straight down. Therefore, according to conservation of momentum, we must have Copyright © 2014 by Nelson Education Ltd 1 a1 9s ‘The speed of the heavier mass is therefore given by Energy conservation gives Lome My = text a Momentum conservation gives m+ v @ Solving (1) and (2) simultaneously gives m+M) (2) Momentum conservation in the x-direction before the arrow emerges out of the plank: ives m m,v, eos(40°)" =(m, +m, )v, x90. 0° wy, = R112 900000840) 1 95 ay O1143.78 ‘The energy lost is then given by Smt Lm, om ye . il 22 2 SE 5(0-11)(90 )-z70 11+3,78)(1.95") = 440 J (b)_v, $1.95 m/s (as calculated above) (@)_ The mass of the ball is given by Geryin ~($202 Jive”) 3x10" kg Assuming no frictional losses, the time it will take the ball to reavh the bottom is given by 2h }2 x 7000 eV ost ‘The average velocity of the ball is 7000 378 3788 =185.2 m/s Copyright © 2014 by Nelson Education Ltd Momentum conservation gives mi, (13x10 )(1ss.2) 6x10" ‘The distance moved by Earth is then given by 1 =(4.0%1077)(37.8) =4.0x107 mis 2x10" m = 0.02 mm () ¥ =4%107 avs (as caloulated in part a) (6) Energy conservation gives (1.310!)(370.4) 6x10" 97. First we will determine the expression for density with respect to distance. Sty =H _ Aly r-0 EL pee j(Brrnhe ice v4} 18 99, We will write the equations in the comoving frame of the system. Momentum conservation gives m,=My, (I) Energy conservation gives 1 1 aloe lian my tomitck mmm +5 @Q 2 2 Copyright © 2014 by Nelson Education Ltd B Solving (1) and (2) simultancously gives op + oa =x") fee —<") _ fiaxsso(o0r 0.07 m( m+ A) 0.91(0.91 41.4) Hence the velocity of the lighter mass in the fixed coordinate system is vf =2.0-0.96=1.0 mis 0.96 m/s 101. The speed of the person just before he hits the platform is iar Energy conservation gives La mis 103. (2) Momentum conservation gives 4 im, +m, yw" @ Conservation of total energy gives 1 1 1 my? =k? + — (sn, +m, )v? (2) pt yt gm) ) Solving (1) and (2) simultaneously gives K(m, +m) ‘The maximum distance is therefore given by mam (am +m) Yas 6.0 |—21%03 _ 0.9526 m = 52.6 om 50.0(0.1+0.3) Copyright © 2014 by Nelson Education Ltd 105. (a) Momentum conservation gives 107, mv Mu = mv'+ Mil a Energy conservation gives Lint shane y = 0.48 mis m,+m, 324167 ‘The child’s acceleration relative to the wagon is HM. = fg =0.7 «981 4 =0.66 m 16 Copyright © 2014 by Nelson Education Ltd Chapter 8—ROTATIONAL DYNAMICS 1. spherical shell since its moment of inertia is larger All three quantities for the point on the rim are larger since the radius is larger. 5. The smaller disk has greater angular speed. Radial acceleration at the circumference of the smaller disk is smaller. The disks have the same tangential speed at their rims. 7. increase 9. shorter rod LL. The equation is still valid if other units (such as degrees) are used for all of the angular quantities, 13, @) No. (b) ‘The dot has radial acceleration but no tangential acceleration 15, negative, positive, equal 17, The moment of inertia of a ring about its axis is given by Tn =P? A thin cylindrical shell can be considered as a number of rings stacked on top of one another, Now, since moment of inertia depends on how mass is distributed about the axis of rotation, the number of rings in the stack will not change the moment of inertia, ‘That is, 19, Conservation of angular momentum gives (0 orn (o.09y( 23 | = (0:9}(0.09)' - (0.011)(0.09) | >v=1.8mm's 21. Tilting the axis of rotation changes the direction of the angular velocity vector, and hence changes the angular momentum vector 23. Tension is the same when there is no acceleration, 25. Yes, at the moment when the angular acceleration reverses the direction of rotation of the wheel. 27. Yes, as long as they are not pushing the car straight toward the tree Copyright © 2014 by Nelson Education Ltd 7 29. 8 No, torque is defined about a pivot, Force is a veetor quantity, so its magnitude is independent of the frame of reference 32nd parallels are the cireles at 32° north and south of the equator. S21 = rg, COS(32°) = (6371000) cos(32") = $.40 x 10° m ‘We know that the angular velocity on any point on Earth's surface is given by = 7.27 x10" rad’s er =(727%10")(5.40 «10° 193 m/s . sn (700022) Given: © = 7000 rev/min = ——_——— y= = 77 2.4510" rads T ~ i2x3600 (@) Given: @ = 227 = 12.57 rad We will use 0= one har with @, =0. 2x12.57 10 = a= cr, =(25.13}(0.11) = 2.76 mis* = 25.13 rads? This will also be the acceleration of a point on the gear's circumference. 2.76 =< =69 rad/s* 7, 0.08 Since every point on a disk has the same angular acceleration, this is the angular acceleration of the rear wheel (b) Since the initial velocity is zero, the velocity at time Fis given by veat=2.76x1.5= 4.14mi PAE 1285 mis? 7, 0.04 am fed $a = 2.76 428.5 = 428.5 mis? = 0.45 kis? (0) @,=2.8xm/s? (as calculated in part a) Copyright © 2014 by Nelson Education Ltd 41 (@)_ The linear aoceleration of the chain is equal to the tangential aveeleration of a point on the gear’s and pedal's circumference = 4,=2.8m/s* (as calculated in part a) ‘The angle swept in the first 5 seconds is given by ere 4 3a") _ 4 “125 >a ear -(4Jo => @, =0.40, Sa ‘The angular velocity at ¢=7s is given by a ‘(Je a, =0562 (1) 0, =O.40t= ‘The angular velocity at ¢=7 s is also given by alS)e2n f= of +200 ={ 0.40, era 23 > e' =0.16 +2010, 2) Solving (1) and (2) simultaneously gives 0, = 13.1 rad Do 05 radis “125~ = @, =(1.05)(8)=8.4 radis 1.3 rad’s? (@) Given: a = 23° =O4 rad in 1=0.758 We will fist calculate the initial angular velocity using the relation lie =ott+—ar 2 13x07 = 0.4=0.70, — => m, = 1.03 rad’s Copyright © 2014 by Nelson Education Ltd 1» ‘The total displacement can be calculated from 2x13 (0) 0, =1.0 radis (as ealeulated above) 43. @. Given: 7 = 0.42 m, @~3.5 rads? for 17.05, 0, = 0 =a, at 01 3.5:3.0-10.5 radis v= ro = 0.82 10.5 = 4.41 ws vo 44P = = 46.3 m's* vr 042 >4,= (0) ©=0, + at =043,5%7.0= 24.5 rads 0 =OA2x 24,5 = 10.29 m/s is constant after 1 =7.0 s, therefore the radial acceleration Since the velocit at ¢= 9.0 sis given by 2.97 -2.7K 4x2. 0))—(-2x2.7-0)j 4(2%2.9-0)8 = (10.87 + 5.4j-+5.88) Nm 45. F =(Asin(133°)e0s(90°))/ + (48in(133°)sin(90°)) j + (4 c08(133°)) rk 47. -5j 47k FF (145TH 6)) (14x 2— Tat 1} f+ (6x24 5x1I)E => # =(287+105} +678) Nm ai 670 cos(37°)i + 6T0sin(37°)} = 535.17 + 403.27 0)i-(0) 7+ (403.2% 2-0) #= (806.48) Nm 80 Copyright © 2014 by Nelson Education Ltd SL #xi= 3x 1- $x 5) (21-53) j +(-2 83x 3) = (287 4137-198 Px F] = 28" 413° +19 =3625 Nm = cost( —!)- 116° 36.28 29.9° = 180° — 29.9° = 150.1° 53. Length of the diagonal is Lavi +h7 =1.56 m ‘The perpendicular distance between a mass at the comer and the axis of rotation is therefore L 78m 2: = 1 = 0.4% 0,78? +0.4x 0,78" = 0.48 kg m? 55. The mass of each blade is i ma122=82 ose ‘The moment of inertia of a blade about an axis passing through the centre of its width and perpendicular to its plane is given by malt mat gf 27) ger ag! 12 im 2 ‘The moment of inertia of the blade about the fan's axis of rotation is then given by .0ST + 0.14% 0.11% = 0.058 kg m* ‘The moment of inertia of the eylinder is L=lame = 092017 9.605 bea? ‘The total moment of inertia is then given by =I, +31, =0.18 kgm? 57. ‘The moment of inertia of a thin disk perpendicular to the z-axis is given by di==y'dm L 2 But dm = ay? pdz Copyright © 2014 by Nelson Education Ltd 81 Using the Pythagorean theorem, y can be expressed as y* = R* - 2* 21 ¥](e- Integration yields 8mpR 15 AmpR* = r=2ure 5 But M 59, If.vis the distance moved by the masses, then energy conservation gives sm, gxsin. 0 = >{m +m, )v* 2, 2mersind m +m, Applying kinematic equation v? — mesin? = 2a, we get in this equation gives the required angular acceleration. 61. Given: v= 133 km/h = 36.9 mis Pag +1P =210.3m u sin @ = = 0.052 rrsind = (25a 9)(210.3)(0.052) = 5.1810" kg m‘s" 1 the case when the car passes the officer, we have 1.0 m and 9=90° (= mvrsing or Jos 9)(11.0)(1.0) = 5.18% 10" kg m’s 82 Copyright © 2014 by Nelson Education Ltd 63. Conservation of angular momentum gives 1,0, =10 ‘The angle between the velocity and radius veetors is given by 0 = 42° 490° =132° The moment of inertia of the pendulum with the putty is mit = 0.124 kgm? +m, (L4RY +m, (e- wee (.-$}» me [== +m, (L+R) +m, (-4) Substituting given values in this equation yields @=1,82 rad/s => y=(0.51+0.17)(1.82)= Energy conservation gives mehn= hie? = 4(F) hours 24) 24 ws zt) 2 051 = f= 0.067 m d= con tf 280.067) _ 56 go 0.68 65. (a) ‘The moment of inertia of the wheel with insect at the centre is given by Jpoaroass12( 22088) onsrsigmt ‘The moment of inertia of the wheel with insect atthe rim is given by 0.021 «0.45 Jso0a1.04 = 0.0638 kg m? Conservation of angular momentum gives lo! - lo ow 4(0.0875 10.30" 0.0638 <0.27°) = 2.610" Copyright © 2014 by Nelson Education Ltd 83 67. Pai, x¥, =(-2x4—5x4)7—-(-2x2-5%7) 7 +(4x2-4x7)k =F =-287 +39) -20k If I’ is perpendicular to the plane containing V, and ,, then we must have V, =0 28x24+39x4—-20x5=0 28 «7+ 39x44 20x2=0 Oa, xh = (5x 442x4)i-(5x742%2)j74+(7x4-4x2)F 2 P' =28)-20)4208 =F 69. (a) The moment of inertia of the tree about the apex of the ent is given by 4200 x 21° + 4200(10.5-1.1)' =5.25 10° kgm? Energy conservation gives 1 1 amv +— la? = mgh mob) =mun r sy — Pmgh _ P2x4200x9.810105 955 m+ 42004 525010 (10.51) () ‘The total kinetic energy of the centre of mass is K = mgh = 4200 «9.81x10.5 ‘The speed of a car having this much kinetic energy can be calculated from 210° J Hin = 43210" 2 =35.1 m/s =127 kavh 71. Radial acceleration is given by asrat .1x9.81 Se= 104 32 rad/s, 84 Copyright © 2014 by Nelson Education Ltd B 75 Given: ¢ = 2 = -19¢-2.57 at Integrating both sides gives fdo= f(i2r+2.50)ur > @, -@ =-6F -0.625° (I) For w, =~ this becomes 2 0.625"! +61 -31.5=0 =ta37Ts de For angular displacement we will integrate (1) on both sides with > foe Jaa- {lor + 0.6250" Jat => O= oft-2r -0.1251° => = (63)(3.77)- 23.77 -0.125x3.77 Conservation of angular momentum gives 10, +1,0,=1,041,0 Here cis the angular velocity of the disks after they've stopped slipping against cach other. _1120x5+910x8 34 rads 1120+ 910 The eneray lost due to friction is Egg = B,~ 8; hy dig + S18 | Lt +t10') 2 2 2 2 = i, = 411208 +910 x8) 4(1120%6.3474910.66.307 77. (a) ‘The moment of inertia of one of the bars perpendicular to the rotation axis is mit Since the bars are thin, the moment of inertia of the bar parallel to the rotation axis is i=mi Copyright © 2014 by Nelson Education Ltd 85 79. 81 86 ‘The total moment of inertia is (4 me (b)_ The parallel axis theorem can certainly be applied to cach perpendicular bar. ml? (©) A thin bar is treated as having its mass distributed in only one dimension. ‘The mass of a thin strip parallel to the axis of rotation is given by dm=W pds ‘The moment of inertia is = pf Pde Ifthe thickness is not negligible, the mass of a strip parallel to the axis of rotation is given by =W pdxdy The moment of inertia is i=}Pan=wp iG way) ree =i "Per str) 12 But WLT p=m ara R(r+r) @) v, = 40, =0.17%70=12 mis ) y= 90, =1,0, =1.2 m/s (©) %=12 m/s (since there's no slippage) Copyright © 2014 by Nelson Education Ltd 83 @ (©) Since the tangential velocity is constant, the total acceleration is equal @ ) 0, _0.17x70_ 052 a= to the radial avceleration. 119? a 1.19 == =27 mss, a= 5 052 x O17 3 ws? (12)(0.917) =14.9 kg m* (i) Hereh= 91m Energy conservation gives 1 = la? = mgh 2 igi sony 3.8 rad/s 149 = v= 2x0.913.79= 6.9 mis (ii) Here A= 27 =1.82 m Energy conservation gives Li 1 i? =mgh giet=me [2x12%9.81x1 82 = a |e SPT 149 => y=2x0.91x5,36=9.8 ms Here h= r+ rsin(40°) =1.49 m Energy conservation gives 4 rad/s 1 = 10" = mgh 3 igh x12K9: a pe EEO RED gy V 149) = v= 2x0.91 x 4.86 = 8.8 mvs Copyright © 2014 by Nelson Education Ltd 87 85. Tat tly th, = Feros059 (MLE sars03 }rmsscosteory => 1=38.58kgm? soxosn(o 4 ST) 2034 33+0.674017) yg #=—_— > 0 ms 394114 28 Energy conservation gives 1 = le? = ms 3 18m 87. Summing torques gives F (0.37) = f (0.02) + £(0.02) 2x110x0.02 037) =F 89. The equations of motion for the (wo masses are mg T=ma — (I) T,-mgsinO—umgeosO=ma (2) ‘Summing torques about the pulley’s axis gives Tr-tr ale la Ph-l=-> @) Solving (1), (2) and (3) simultaneously gives mg—m gain 0— pum,¢ cos 0 I m+ m+ r ‘The velocity after moving a distance d is therefore given by faa(m,@—m,gsin 0 — sumy¢ 00s) meme m+ ar 88 Copyright © 2014 by Nelson Education Ltd OL, @) Conservation of angular momentum gives 93, 98 1,0, +10, =lo+ la Here © is the angular velocity of the disks after they've stopped slipping against each other. omens Soest 4 0 rads ® x027 ) (° 0.135) 2 Jt 2 } (b) The energy lost duc to friction is Fin = EE, @ ©) 160.138"), ,,]_ 18.9 [/32%027°) (160. 4{ 7 pe | -]] | 2 2 2 Given: @,, = 6.0 rev/min = 0.63 rad/s > 0, 29-067 ra’ Conservation of angular momentum gives 1,0, =1,0, +130 q3 (2700 +28%3° or, = (28 x3*)(0.67)+(2700 +28 x3*)(00.63) = 0, = 0.69 rads Given: «4 = 0.8 rad/s Conservation of angular momentum gives Ton, = iy, = (2700)eo, = (2700 + 28x3")(0.8) => @, = 0.87 rad’s Integrating both sides we get feo = ferar Sona-2e-1) a For «= 0 and @, =35 rad’, this yields telds Copyright © 2014 by Nelson Education Ltd 89 To find the number of revolutions, we will integrate equation (1) with respect to time. de 2 on Fae -2(e"-1) > feo - a far-af(e =1)de (e" -2r-1) Fort =1.45, this gives 0 =36.35 rad 36.3 =e ‘This is equal to =5.8 revolutions, 97. (a) Equations of motion are ay 90. Copyright © 2014 by Nelson Education Ltd (>) 29. @) © ©) @ (e) ‘The linear velocity after the mass has descended 1.2 m is given by (Fah = (21 35x12 =1.80 mis = = tO 99 54 rads 0.08 Energy conservation gives Lie ae Smv? = mgh—— 1,08 ——T,093 3 igh 1, > I4 Negative, as the astronaut is being pushed toward the centre by the centripetal forve Angular speed will inerease since the moment of inertia will deorease such that their product remains constant to preserve angular momentum Conservation of angular momentum gives 1a, = 10, (om 15000178) sro |(43)-| BRIT om, j \ 2 4 >, = 13 rads Kinetic energy will decrease since some energy will be lost as the astronauts walk toward the edge of the platform Conservation of angular momentum gives 1,0, = 1,0, -(et Js) -( PIe% = 0, = 1.27 rad’s Copyright © 2014 by Nelson Education Ltd on ‘The work done is given by ame SW =467x10' F247 kT 5000 x17 sroar esau 27] Copyright © 2014 by Nelson Education Ltd Chapter 9—ROLLING MOTIO! SS 1. The ring will reach higher, as shown below. For the ring, conservation of mechanical energy gives, se] mgh= dmv a Lie = v Similarly, for the disk we have migra inst Le af =a ag 3. @) Fora uniform solid cylinder we have ka = Le? = marr 2 4 3 ‘Therefore, the solid cylinder will have greater rotational kinetic energy (b) For a uniform solid eylinder, conservation of energy gives Toy 1g 1g fm mgh= my? + See = Sm? + 2 a 2 2 igh oid For uniform solid sphere, conservation of energy gives a, Af 2mr (J Io my? + > x 3 2 10 =e= =o= 4h ‘Therefore the solid sphere will have greater angular speed, Copyright © 2014 by Nelson Education Ltd 98 94, ‘To prevent locking up of wheels that may cause skidding. Since the tires of the car are rotating without slipping, the friction is statio, For a solid uniform ring we have ge By maha Sm" + lo ari For a solid disk we have 1 yt sm a, Ms et ad my += 10? = mv + gq mgh= a 1 2 sre fe For a solid sphere we have mgh= Lm? + Lto* = Lin’ + pee ave tet Vo Hence the sphere will reach the bottom first. == 2mr? ‘The disks will reach the bottom of the hill at the same time since the final velocity is independent of the mass as shown below. For a disk: At the bottom of the ramp, conservation of energy gives ee Copyright © 2014 by Nelson Education Ltd 15, ‘The shells will reach the bottom of the hill at the same time since the final speed is, independent of mass and radius as shown in the solution to problem 13. 17. Itis not possible since the final speed of both objects is independent of their radii 19. 1:10 since the force of friction is proportional to mass 21. Friction and air resistance ean slow the wheel down. 23. opposite to the direction of motion 28. (@) vy =24 km/s? 610 mis” (6) After quarter period: speed = 15 m/s, a= G After half period: speed = 15 m/s, a = Ss = 610 mis® 27. (a) Given: y,, = 56 km/h= 15.56 mis v, =10 km/h = 2.78 mis = Mg =15.56-+2.78 = 18 ms () Since the tire is rolling as well as skidding, there is no point where the velocity is zero 29. (A) Giyy P= OSXT A= 5.68 m/s? 68 + 2.30 = 7.98 mis* >a= (b) 0, = @ +af=047.1%1.70 =12.07 rad/s Vio =P, = 9.66 mis vp =¥, +a =042.341,70=3.91 m/s 66 +3.91=-5.75 m/s 41.33 mis? u G= 2.307 + 41.337 > 2.30? +4133 =414 ms? 1f 4133 F @= tan '[ T=] =86.8" with expect to direction of motion 31. The apparent acceleration is due to change in frame of reference and not a real effect. Copyright © 2014 by Nelson Education Ltd 98 Summing torques about the central axis we get (eos r+ R= la With f = sume, this gives Bacoct ( net) 1F Balancing forces, we get mg-T=ma (1) Summing torques, we get rset =T=ma Solving (1) and (2) simultaneously gives But 7 = mg pant 2 37. Direotions are shown by the darker arrows Fj and fy sight, and 7% left 39. The equation of motion is given by mgsinG2°)— f'=ma (1) ‘Summing torques gives rl) =m @ 96 Copyright © 2014 by Nelson Education Ltd Solving (1) and (2) simultancously gives, Since the force of friction does not depend on radius, there will be no change in its magnitude when the radius is tripled keeping the mass the same AL. (a) The equation of motion is given by mgsinG?)—f=ma (1) Summing torques gives f= ta=(nr (2) = f=ma @ Solving (1) and (2) simultaneously gives sen a = 2.95 mis? 3x2.95 =3.8N (b) The equation of motion is given by mgsinG@7)-f=ma (1) Summing torques gives if T= (mr + me? YJ i = f=ma 2 Solving (1) and (2) simultaneously gives a= 80h 2.95 mais? = f-13%295=3.8N 43. IPF, is the force of friction between the stone and the log and 7, is the force of friction between the log and the surface, the equation of motion is given by F-2f,-2f, =(M42m)a a Summing torques about a log's axis of rotation gives FR+fR4+ f,R=ma @ Copyright © 2014 by Nelson Education Ltd oT Solving (1) and (2) simultaneously gives 3F Sm+ Me 45. (a) The equation of motion is 41. 98 mgsind—f=ma (1) If the cylinder could roll without slipping, the sum of the torques about its axis, of rotation would be given by Substituting this into (1) gives 2 g sino) =4.1 mis? = f=0.70N For rolling without slipping the maximum force of friction is given by Fase = Hamg = 0.57 N Since f > f,,,. the cylinder cannot roll without slipping, (b) frela (=). For rolling with slipping, f = 4#,mg =0.4N. 2\04) aed cadie? a=) (0.34)(0.03) (©) The energy lost duc to friction is equal to the work done by friction =e Jeans Summing torques about the axis of rotation gives fra=la sin(397) ‘The condition for rolling without slipping is: = (0.3mgcos8)(r)= CE 3 = a=0.75gc0s8 wo Copyright © 2014 by Nelson Education Ltd 49, 51 ‘The equation of motion is given by mgsin#—O.3mgcos@ = ma (2) Solving (1) and (2) simultancously gives tan = 1.05 = 0 =46.8 If fis the force of friction between the pad and the wheel rim, the equation of motion is, given by MgsinO-2f=Ma (1) Summing torques about the axis of rotation is “G 3 2 Substituting this into (1) gives la ae a= gsin(17°) = 2.86 m/s* = f=845N (a) Energy conservation gives mgh= mvs tre? <1 a? Af |e 2 ta 2 alo We 2(mgh)= 275 (b) Energy conservation gives Vtg byte den? mgh = —my? + — To" = —my? + ze tg 2 = Ky, = EA 205 2 Ky =Krgy = 201 Copyright © 2014 by Nelson Education Ltd 99 53 55 100. (a) Energy conservation gives igh = mvt + Lia? = Lm a A{ 20 |e ze as 3 = Kon 7 On the frictionless ramp, only translational kinetic energy contributes to mgh) = 49.0m 3 ‘movement up the ramp, = mgh’ = 49.0m = h'=5.0m (>). Yes, since the ball spins (not rolls) on its way up the ramp, (©) The ball keeps on spinning at the same angular velocity it had at the bottom of the ramp. K,, = 2, -[2}(490m)-19.6m — Uf 2m a 5 => © =160 rad/s (@) Energy conservation gives mgh= Ly? + Lye? = La? + 2 2 On the frictionless side, only translational kinetic energy contributes to movement up the frictionless surface. = nigh" S(meh) Sh T Energy is conserved. The reduction in height is due to the fact that the ball spins without rolling on its way up the frictionless surface Pa (b) At the highest point the translational kinetic energy will be zero Copyright © 2014 by Nelson Education Ltd (©) Atthe highest point the rotational kinetic energy will be the same as it ‘was at the bottom of the bowl that is c[BIe]oo (5) (@)_Aring of the same mass has a higher moment of inertia, so more of the total energy will go towards rotational motion as it moves down, Less translational cnergy means that the ring will move up to a lesser height on the right side as compared to the solid sphere. 57. (a) Energy conservation gives stig nt Are A dl ye a a mgh 3 Since tension can be considered working against rotation, we have aK = Th = mh 3 => 1 =069N (>) Energy conservation gives 3.07h 854 _ 109 rade? 0.06 Copyright © 2014 by Nelson Education Ltd 101 59. Applying the equation of motion to the eylinder gives -£=M,a oO Summing torques about the eylinder’s axis of rotation gives sa tae( SE) M,gsin 6, Q ) Applying the equation of motion to the mass on the table gives Ty Thy = ma @ Summing torques about the axis of rotation of the pulley on the left gives Fan tah AP ©) % Applying the equation of motion to the sphere gives 7, —M,gsin§ ~ f,=M,a ) Summing torques about the sphere's axis of rotation gives ml) o) Solving these equations simultancously gives the required acceleration gsin 8, -M,gsing 1.+2M, +m. ro 61. (a) The object will roll in the direction of the force. (b) If there is no force of friction, the oy about its axis of rotation gives der will spin about its axis. Summing torques lo2 Copyright © 2014 by Nelson Education Ltd 63. Energy conservation gives Dececapleras mgh = + my? + , = 7.76 mis Since the total energy possessed by the ball before collision is mgh, the conservation of mechanical energy gives mgh= jm + pane (assuming that the ball stops rolling after collision) = O.8y!? 44.50! = 67.42 @ Conservation of momentum gives my, = mv + Ml = 1G +9.0v) =12.42 Q) Solving (1) and (2) simultaneously gives =-6.8 m/s y 65. (a) The height covered by the centre of mass of the ball is given by h=(R-r)—(R=r)oos(72") = 0.69(R-r) Energy conservation gives fm? fe enmgi ey =0.69mg(8—r} ~ Bom? =0.69me(R-r) my? (R= * Therefore the total normal force is N=F.4mg=17.9"N = 81m N (>) The normal forve is independent of r and R. (c) N=17.9x0.67 =12.0 N. The normal force is independent of r and R Copyright © 2014 by Nelson Education Ltd 103 67. (@) Summing torques about the axis of rotation gives pone spe 2me a When the ball starts rolling, the foree of friction is fume = = am 5 ame 5x0.18 aan BOE si (>) Summing torques about the axis of rotation gives peta (7 le 2m?) =rame =| Je Hg = a= ES 2312 rads? 2r The equation of motion is, samg=—ma > a=-14ms? ‘Therefore, the kinematics equations for linear and rotational motion give vv bat = ro, = 90-1371 =o, =81.80-12.45/ a Also, 9, =@ +at 31.2t 2, Substituting this into (1) gives t=195 (©) ¥, re, =6.42 mis ak = bow? 1m 2 2 104 Copyright © 2014 by Nelson Education Ltd 69. ‘The equation of motion for the hanging mass is Mg-T,=Ma a Summing torques about the rotation axis of the pulley gives rT, rT, =a =r, rT, @ Summing torques about the cylinder’s axis of rotation gives RD, + Rf = Le, ( 5 (4) ma, > (3) Byte e) ‘The equation of motion for the eylinder is 1-f=ma, Solving this with (3) gives Only if the object is already rolling without slipping. Otherwise the object will slip because there is no frictional torque to change the angular specd. If the marble is a perfectly rigid sphere and the horizontal surface is totally frictionless, the marble will not lose any energy to the surface. However, air resistance will cause the marble to gradually come to a stop. Copyright © 2014 by Nelson Education Ltd 105 75. Equation 9-18 is Kalo 2 Here /, is the moment of inertia about point b on the surface. According to the parallel axis theorem, the moment of inertia about the central axis is given by 1,=,+m Substituting this into the above equation gives eel (ator )ot ahaa otne(- 2 This gives equation 9-17; that is rele! slim 77. (a) The force of friction on cach cylinder is given by f- 03s()e + 0.5{m + Xe Therefore. the maximum force that would not cause the plank to slip is fase =2[s9(EJevoss{me Je] -c Im+09M)¢ (b) (2m+Af a= (1.1m +0.9M)¢ _ (im +091) ~~ Qm+M) =a 79. Summing torques about the axis of rotation, we get Fre fra ani(2) 5 Ja > F+f= aM r The equation of motion is given by F-f=ma @ Solving (1) and (2) simultaneously, we get Since J > mr? f >0 and hence the foree of friction actually pushes the object forward 106 Copyright © 2014 by Nelson Education Ltd 81. (a) The sum of the translational and rotational kinetic energy of the marble must be equal to its potential energy. 1 ag lhize son tai h pi + slo" = me aif 2m? 25 (b) The height 4 is where the marble will be stable. If it enters above f, it will move down and if it enters below fy, it will move up. 83. (a) Balanoing forces on the mass M/ gives Mg-T, =Ma o Balancing forees on the mass m gives T,-mgsinO-f=ma (2) ‘Summing torques about the cylinders axis of rotation gives seems] 5 Summing torques about the pulley’s axis of rotation gives Tyr =], (2) ) E Solving equations (1), (2), (3) and (4) simultancously gi =O+f- o 2Mg - mgsin® Im 27 m/s* 1, 2M +22 Copyright © 2014 by Nelson Education Ltd 107 Chapter 10O—EQUILIBRIUM AND ELASTICITY 1, No, even when the object is at its highest point an unbalanced force (gravity) acts on it 3 5. 6 ib 9. No, an unbalanced centripetal force is acting on the object. 11. Yes. For example, the centre of gravity ofa V-shaped object lies outside the object. 13. no 15. Assuming origin at the centre of the star, we have » ommtm(ar) ar from the centre of the star m+m 17. No. since Young's modulus is inversely proportional to the surface area on which foree is applied 19. Steel is stronger since its yield strength and ultimate tensile strength are higher than those of hair. 21. compression 23. d 25. © 27. b 29. a 31. Balancing forces gives T=5x98=49N This must be equal to the force of friction on 12 kg mass on the verge of slipping. = f= (12x98) =49N = =0.42 108 Copyright © 2014 by Nelson Education Ltd 39. 4 |. Since compression foree is normal to the walls, therefore He =F 659.8 308.3 N This is the total compression force required. The force on cach wall is 308.3 therefore 08 = 2654.2 N. Balancing forces gives F,+ 8, =F, =14000 x98 =13.72 10" N Balancing torque with pivot at leit end gives F, (2.5) =F, (5.0) => F, = 6.86 x10" N => F = 13.72x10* — 6.86 x10" = 6.86 x10" N Balancing forces gives f=(2.7+4)9.8= 65.7 N Eere f is the force of friction) Now we have to detemmine the net nomnal force at the joint. Balancing torques about pivot at the base on right, we have N, (LOsin(8°)) = 4 «9.8% 0.80808") =, =139.5N Balancing torques about pivot at the base on left, we have N, (1.Gsin(8°)) = 2.7 9.8 D.8e0s(8") Since / = uN, = 4=0.49 Balancing torques about pivot on right end gives, Tg Sin(21°)(7) = 3009.8 3.5 = Tiy = 41019 N Balancing torques about pivot on right end gives Tog Sin BP?)(7) = 300 x9.8%3.5 > Tngy = 2442.6 N Balancing torques about the angle gives P(ViP 028) =759.8%0.24 = F=1648N Copyright © 2014 by Nelson Education Ltd 199 43. For simplicity, we will assume a one-dimensional rod of linear mass density u but the result will be equally valid for any uniform rod. ‘The centre of gravity is given by frsooure ar fre Mar soo erie few dr 120-(L+120)e >, ‘s 1-e 120-(L4120)e* a 1-e® 2 45. Distance between the masses is f= VL00? +2007 = 223.6 m (100 + 200 j)107mg = 49,37 498.67 110mg +107me 47. Balancing torque at the hinge, we get T (4sin(46°)) = mg (2} = 7 =1430.5 N 49. Since the erate is not moving, balancing forces gives 1 =4x9.8=392N 51. Balancing torques gives F(0.8-0.2—0 1)=me(Joa®=02") => F =8148N 53. First we will balance torques on hinges A and B. (2100)(9.8)(2.25)+¢. (I=C,(2) (700)(9. 8)(I)+C, (2.5)=c, (2) Q) Solving (1) and (2) simultaneously gives C, =26296.7 Nand C, =36300.8 N Lo. Copyright © 2014 by Nelson Education Ltd Balancing forces on the top bar gives A, = C, = 26296.7 N A+, =2100x9.8 >a 15720.8 N__ (negative sign shows that the direction chosen was wrong) Balancing forves on the bottom bar gives B,=C, = 26296.7N B,+C, =100x98 => B, =-29440.8 N_ (negative sign shows that the direction chosen was wrong) 55. Since the triangle ABC is isosceles, we have 2B=30° and 24=75° Length of the larger beam is therefore given by __ 6 © sin(7°) Considering only longer beam, summing torques about A gives mg cos(75° (2) F, (4—4c0s0}) =F, (2) 62m = 2F,-0.54F, = 8649.2 a Considering only shorter beam, summing torques about B gives mg cos(30°)(2)= F, cos(30°)(4)+ & (2) => 2F, 43.46, = 18671.5 @ Solving (1) and (2) simultaneously gives F,=S00L1N, and F, =2505.6N Summing forces in vertical direction gives N/4N,+F, =2mg = +N, #190544 @ Summing forces in horizontal direction gives KR ah = f= f = 5001.1 co) Summing torques about the hinge for the larger beam gives ae een F(2)+ mg eos(75 (2) , (4 4008(30°)) = 2f, -0.54N, = 8649.2 eo Copyright © 2014 by Nelson Education Ltd Mt Summing torques about the hinge for the smaller beam gives £, (2) + mg cos(30")(2) = N, e08(30°)(4) => 2f, -346N, =-18671.5 © Solving (3), 4), (5) and (6) simultancously gives |f]=1226.1N and |f|=3775.0N 57. (190 «10°)(0.13)(25 x10*) ~ (9.8}(4.0) =m 575.3 kg 59. We know that the maximum stress on a beam is given by i be _ 0.1(0.1)" 8.33%10% kg m* Here / = = 12 12 0.05 m M ue = 2.3(mg) 2.3(mg)(0.05) 833x105 = m=184.8kg = 25x10% = 61. 2000 - a(o21y) 3.0 =>Y =2.9x10° Pa=2.9 MPa os 63. (a) 12 Copyright © 2014 by Nelson Education Ltd 65. 67. 69. roa, _#(0.03)'(450%10°) (b) = = 3x10" ky to) m= 3 8 (0) Da 2rm 2 [ME ap fEdA1O 29.8 9 04 m= 4.0 om ra. {7(1000%10") 00 MPa. ‘The yield strength of femur is approximately given by 0, ‘The average diameter of a female femur bone is D = 25.4 mm. F__ 82x98 ——— = 1.6 «10° Pa = 1.6 MPa A” 7 (0.0127) = =0016 The bottom block can withstand a force of 25010°)(0.3*03) =2.25 x10" N ‘This is equivalent to a mass of M =“ =230x10° ke g ‘The mass of one block is m= (0.30.3 x 0.3)(2700) = 72.9 kg Therefore the number of blocks that can be stacked is N= 531550 m (a) Change in length before breaking is given by cet, _ (30010°}(2.0) Yy 7x10 Force needed to break the column is 1 = (30010) (:70.248") = 5.663107 N AL= 0.0082 m. W = FAL =(5.66 x10" )(0.0082) = 4.6410" J Velocity after dropping for 20 m is given by 2(9.8}(20) =392 m’s? Copyright © 2014 by Nelson Education Ltd 13 n 78 71. 4 And the kinetio energy is K= dmv =196m I ‘This must be equal to the work done caleulatd in part (a). => 196m = 4.64 10° m = 2367.3 kg, Balancing torques at the hinge gives (900)(9.8){1.85 cos(11°)) + ( 200}(9.8){3.0cos(1 1°) = (3.7 sin(41°)) = N=8976.3N For reaction forces, we balance horizontal and vertical forces. R, = Nc0s(30°)= 7773.7 N R, + Nsin(30°}= (900 + 200}(9.8) = R, =62919N Since there is surface is frictionless, the sphere will not rotate. It will stabilize when the line of rope passes through its centre. = == ~0.167 raa= oR 0" Foree balance on the mass on table gives LD, = amg = (0.35)(3.0)(9.8) = 10.29 N Torque balance on pulley gives TR=T2R 0, =5.15N Forve balance on hanging mass gives = m=053ke Balancing torques at the hinge gives 7 (1A3sin(85°)) = (760) (9.8)(1.7 eos(30%}+ (220}(2.8}(2.98c0s(30")) => =14683.6 N Balancing forces gives R= Tcos(85°~ 30°) = 8422.2 N sin (85° 30°) —(760-+ 2209.8 = 2424.1 N Copyright © 2014 by Nelson Education Ltd 29. 83 85, Balancing torques about the centre of the sphere gives F(d-r)= fr a Since f= 44N and N = fam Q Solving (1) and (2) simultaneously, we get d=r(l+y) Balancing torques about the central axis gives £,(0.5)+ F, (0.5)=120(0.2) (Here subscripts I and 2 refer to inclined and horzontal surfaces respectively.) S ht hrs ao A = a4, = (0.2)(120)(sin(32")) =12.72 N fy = Hng =39.24t, Substituting these in (1) gives 209 Balancing torques about the point where the bar is attached to the string gives Mg cos(42 (5) Balancing forces gives M,N =Tsin(21°) @ Teos(2l°)+N=mg @) Balancing torques about the point of contact with floor gives fg cos 42" (5) Co) Solving (1), @), @) and (4) simultaneously gives 4, = 0.29 Neos(42°(L)+ wNsin42°(L} 1) Tsin(42(L) (a) Balancing horizontal forces gives F = f= ung =306.5N (b) ‘The clockwise torque about the front pedestal is = F(L1) The counter clockwise torque about the front pedestal is 1, = mg(0.38) = 342.6 N Since +, <7,, the refrigerator will not tip first. 137.2 N Copyright © 2014 by Nelson Education Ltd M5 87. 89, lis (©) The torques about the front pedestal must be balanced in this case. F(y)= mg (0.38) y= 118 m=11180m Balancing torques about the front edge of the shelf gives " F(14sin(30°)) = (15)(9. a => F=284N (@) F=13GPa 0) 238x107 20x10? 18x 105 Force (N) 10x 10> osx 1 0.0% 10> 0 001 02 0.03 Elongation (m) (©) Since a and F=kAL ( ser a3 = (13x10 JA = 0.021 Nin (ny = $(0.021(0.01%0.11) =1.26x10" 7 @ (©) Maximum strain on silk is AL _UIS _150x10° _ 9 99 LY 13x10 Energy needed for this is Basia 0.021)(0.88 0.2) =3.29x10 Copyright © 2014 by Nelson Education Ltd 95 my? =3.29x104 = v= 0.115 m/s = 11.5 cm/s Yes, this speed is praotival (160-20) 10° . i hs ¥ == 23.3. GP ponte (0.8—0.2)x0.01 “ (oe "2 23.3 x10°) ——~_+ =1,22 Nin L 0.015 Weight will be distributed equally in all four bars. Each bar will therefore support amass of = 1204 $500 1155 49 Stress in each bar is given by mg _1155x9.8 44x10 P= 1.44 MPa A O05 Strain in cach bar is given by 1.44 x10" Y 00x10" 2x10 Kinctio energy of the bullet is 1 1 fi Lot = L(o.075)(390)' = 570.41 ins? =3(0.0078)(390) The point at which the steel plate will reach its shear strength is give by Al, _ O.TSUFS. _ 0.75 360010" _ 9 497 L ¥ 100x110" = AL = 0.027% 0,04 = 0.00108 m ‘The elastic constant is given by A Energy needed to break the thread is 1 SH (an) (2.5x10" a )(0.00108)" = 1.46 x10%d? J 2 This should be equal to the bullet's kinetic energy. = 1.46 x10%d? = 570.4 => d=0.02 m=2.0 om Copyright © 2014 by Nelson Education Ltd 7 97. (@) Velocity of athlete just before hitting ground is v= J2gh ‘The momentum is therefore given by p, = m/2gh Since the final momentum is zero, the change in momentum is Ap = m/2gh. This is equal to the impulse /t. 1 faa 1 ioox10* xx 0.017 sha {O_O sm 2gt\ mm 2x9.8%220 x10" 70 b) HL _ mbsf2gh Ya WA =1.6«10% m 99. Longitudinal compression: ” = (200 10° )(70.015') =1.4 x10" N Longitudinal tension: = (135% 10") (0.0154) =9.5 x10" N 1314108) (70.015*) = 9.3.10" N ) ig =(The10°)(r0.018°)= 5010" Transverse compression: 46x10" N Transverse tension: H,, = (65 x10°)(70.015° 101. (@) ‘The ultimate tensile strength inoreascs as the steel passes through subsequent eycles From the graph we see that the UTS of the steel was about 850 MPa. The UTS increased to about 860 MPa after first pass and to about 1140 MPa after four passes. (b) The pressing process increases the ultimate tensile strength of the steel, and hence inereases the energy required to break the steel ls Copyright © 2014 by Nelson Education Ltd Chapter 11L—GRAVITATION Lob Weight is a force and should be measured in units such as newtons, 5b Acceleration due to gravity is given by GM. R Because the density of the two planets is the same, it follows that M, M, %, _oM Re > ex a i x a ou eR is the universal gravitational constant Copyright © 2014 by Nelson Education Ltd 19 11. The satellite speeds up as it Loses altitude duc to atmospheric drag. ‘The magnitude of the satellite’s kinetic energy is half of the magnitude of the satellite’s potential energy, but the potential energy is negative and the kinetic energy is positive. Thus, when atmospheric drag causes the satellite’s total energy to reduce by 1 unit, the kinetic energy actually increases by 1 unit and the potential energy decreases by 2 units 13. b Ped Tag 2 P'0(4a}? 27 =8T 15. ©, Because the star has no component of velocity towards ot away from us, there is no redshift or blueshift Ina 19. p= 2.3108 ly = (2.3 10°}(3.00 « 10% «3.16107 )= 2.210% m = (10°)(1.989:-10") = 1.989 10" ky 1, (1.989 10" )(50) = 6.672» 10 - (2.25107) =14510" N 84 10" —6.378 <10" =3.776%10" m 104 «10° m. 21. Nearest point: 1, = Farthest point: r, = 3.8410" + 6.378 x10" Mm F=G = (6.674 x10" )(7348<10”) 2.21K10% Nikg As a percentage of the force per kg at the centre of the earth, the difference between the forces at the nearest point and farthest point is 1. GM = (3 ne } GM 120 Copyright © 2014 by Nelson Education Ltd 25, 21. (3.84x10'Y | —-—__| = it, = Sar p= 42500) (4800) =2.513 x10" ke GML OM, iggy, (S6TDAIO™ )ast9n00") ae (500) = 9.801 m/ First we will calculate the mass of the planet. Gin _ my > = 767 x10" ke (6672110 ){7.67-10") > 8.00 m (8000«10") _(6.672x10"")(7.67 x10") : = 20.5 mi? (s000%10°)) Copyright © 2014 by Nelson Education Ltd nt 29. ‘The work done against gravity is equal to the change in potential energy: GM GMm R 6 ~avn{ 2-2) R we R, — 3.40108 W =(6.672x10-" \(6.42x10" (2si( W = 202x108 J “vu BU yt ax 2.410 mis = 2.4 kmis (4:10*)(1.989%10") = 7.956310" ky = 3.8.x 24 x 3600 x 3.00 x10* = 9.072 x10" m an 5 GM, +M,) Since M,, >> M, 4a An GM, (6.672% 10" )(7.956 10") = 236x10' s=75y (9.072 x10" 2na _27(9.07x10") “TY” 236x10" 122 Copyright © 2014 by Nelson Education Ltd GN 6.672 x10" (7.3510 aad, (6972x107 )(735110") sy r 1737 x10" 39. @ 4, May FM, With origin of the coordinate axes at the centre of the Sun, we have 7, = 0 0+ (5.972 x10 }(1.496 x10") - P= 4.492 «10° m = 449 km from the centre 1.98910" + 5.97210" 6.955 «10° m By _ 4.492 x10° Tuy 0.955 x10" a0 Far pa($n)fiosst0'y (980) 9.2000" kg (6.67210 )(9.12 x10") (10.sx10*) Since the object is dropped, the initial velocity is zero. = 55x10? mis? ge Copyright © 2014 by Nelson Education Ltd 123 _ aa? Ga, Photos (Oi +My.) >My -M, oP a ‘The orbital period of Phobos is 7” = 0.319 days = 27,561.6 s Using the value 6.4185 10” kg for the mass of Mars, and the value 9.400 <10% m for the semi-major axis of Phobos, we have 4" (6.672 10" )(27561.6)" This result is inconsistent with the mass obtained in the solution to part (a), and is a sign that the assumptions in parts (a) and (b) are inconsistent. The accepted value for the mass of Phobos is 1.072 10" kg, which is close to the result of part (a) (9.4x10°) 6.4185 110" = 5.11210" ke = Mites 43. (a) The moment of inertia of a point mass a distance r from axis of rotation is given by Lame ‘Thus, the total moment of inertia of the two masses is 1=2(0.73)(09) = 118 kgm? () 7 anf « 4m (1.18) (420) = 2.6510" Nem = (6ren10 OO ants N (8) r=rF =(0.9)(1.45x10 31x10? Nem (©) 5 =-18 =\-= (21319197) gap x na x 265x107 GM. 45. R gk? (1-8)(4300- 10°) 2 (800 Yoo 10% bg >M = 6.67210" 124 Copyright © 2014 by Nelson Education Ltd 41. 49. 31 (b) Escape speed is given by Baar _ [(2)(6.672x10)(5,0x10") Vr V @0000 9x10" m/s 4300 x10" 3.9 kin/s To be gravitationally bound to Sun, the kinetic energy of the object must be less than the magnitude of the gravitational potential nergy: that is 1 GMm mm < 2(6.67210 )(L 98910") 1.4910" =v <42.2 km/s = 42.210" mis ‘The total angle subtended by Sun is 0.53°. The radius of the Sun can be estimated from half of this angle (see figure) and the Sun-arth mean distance D. R= Dtan( 0.265" } = (1,50 x10" ) tan 0.265" ) = 6.938 x10" m Hence the density is M___1.989x10" 7 + = 1400 kg/m? re 4n(69x8%10') Tarth Sun For an object in circular orbit about another object, the centripetal force is provided by, and therefore equal to, the gravitational foree; that is my _ GMin ror Copyright © 2014 by Nelson Education Ltd 125 57. 126 |. (a) For an object in circular orbit about another object, the centripetal foree is provided by, and therefore equal to, the gravitational force; that is my? _ GMin r # [oar _ [[esr10 aan0") Vr 55000 3x10" m/s 2ar _(2n)(55000) ——* = 0.0047 ¢= 4.7 ms v 73x10 ) The simulation shows increased numbers of asteroids that have orbital radii and periods similar to Jupiter, but travel approximately 60° ahcad of or behind Jupiter. These locations are two of the Lagrange points (L4 and L). (a) According to Kepler’s third law Assuming the masses of both planets to be much smaller than the mass of the star, we get ram... 4a” This can be used to calculate the ratio of semi-major axes for both planet. ai Given: 7, = 10 days and 7, = 25 days >a, ) Momentum balance for the planet with the 10-day period is TAY = Maes Vany Similarly for the sccond planet we have MY, = Moy Var @ But we saw earlier that = (2. a, (A, Hence the planet with a period of 10 days is slightly more massive. 59. Suppose x is the distance from the centre of Moon where Moon’s gravitational force balances Earth’s gravitational force. The force balance for an object of mass m at that distance gives (Here r is the distance between Earth and Moon) 3.8410" ie & 97x10" 7.3410" This is the minimum distance the abject has to travel to reach the point at which it could fall into Earth, The minimum initial speed needed to take the object to this point is calculated as follows: sm? OM R, 3.83x107 m Copyright © 2014 by Nelson Education Ltd 7 3 knvs 61. We know that force is equal to the negative gradient of potential, that is y gam 4 2 ar, (6.955 x10") =5.22 10" m Since the mass of the Sun was concentrated in a shell extremely far away from. its contre, its potential energy ean be assumed to be zero. uz0 ‘The potential energy of the Sun after contracting is given by vn Henee, the energy available to be radiated away by the Sun after contracting is GM ty 1p [72x10 }f.999x10°) =5.1x10" J 3210 * _ 51x10" 3.85% 1 ) « 3x10" $= 4.2%107 y= 42 million years 128 Copyright © 2014 by Nelson Education Ltd 65. (a) According to Kepler's third law — HM 9, OAT aed [CA + Mae) ee [ea (6.67210 " (1.8 x10" +4.5 <10") s TO ag = a=1.5x10" m Assuming spherical orbit, the veloc: 2a(1.sx10") 12243600 ‘Momentum balance gives y of the planet is Qn 9.010" mis MaarVar =M stout pont Steg 4.5x10" Me vos = gage MIO =22.5 ms > Meer () The spectral shift due to velocity is given by 7 eA __gs G(M gp + M4 jane) =F G (Mogae + M pane yt 4a? [eer 67210 (1.810% +4.5x10")]/ = 0=1.5x10" m > aA 396,78100 — 0.00003 nm. 0.10" = 4, = (396.78100 - 0.00003) to (396.7810 + 0.00003) nm Or 2,, = 396,78097 to 396,78103 nm Copyright © 2014 by Nelson Education Ltd 129 67. (@) We know that the eseape velocity is inversely proportional to the square root of distance. Let us assume a comet in an orbit very close to the eentre of the Sun, say r= 510" m. If this comet has a velocity higher than the escape velocity, it can break out of Sun’s gravitational pull and enter Earth’s atmosphere. The escape velocity is given by paw _ P(6o7~ 10" \(1989 10") Oe = 72.910? mis = 73 kwv's $x 10" (>) ‘The mean radius of Halley's comet is approximately = 4000 m with a mcan density of =600 kg/m? =M (600)* (4000) =1.6 x10" kg @ KE = My 1 (1.6 x10")(73000)' = 4.2710? J ‘This is equivalent to 42710 )(239x10"" 2x10* tonnes of TNT. 130 Copyright © 2014 by Nelson Education Ltd Chapter 12—FLUIDS Ww 17, (©) Density is inversely proportional to volume. (@) The net force at the bottom of the eylinder is proportional to the area of the bottom. ABC. The density corresponds to the slope of the graphed line. (a), When the iron cube is in the boat, the volume of water displaced has the same weight as the weight of the cube. However, when the cube is dropped in the water, the volume of water displaced is equal to the volume of the cube. The first volume of water is greater than the second volume of water. Therefore the water level will fall (©) The magnitude of the buoyant foree on a floating object is equal to the weight of the object. In other words, the floating object displaces an amount of water equal to its ‘weight. When the ice cube melts, the amount of extra water in the glass is exactly the same as the amount of Water displaced, so the Water level remains the same. (b) The scale measures the weight of the container and its contents. If instead of placing the ball in the container, an amount of water with weight equal to the ball’s weight were plaved in the container, the seale would read the same in each vase, Yes, the pressure at the bottom depends on the height of the water, not on the surface area at the bottom. (a) The tension on the string is given by r Mg-F, T=ple-ple Given: p, = 4p =T=pl Bh, 4 Number of water molecules in 1 em’ is (1.0) =3.344x10" molecules Assume each molecule to be at the centre of a cube of length x. The volume of cach of these cubes is given by ve. 2.99.10 om? 3346 x10" x=3.bd0° em Since the molecules are at the centre of these eubes, therefore the distance between the molecules is also 3.110 em = 0.31 nm, Copyright © 2014 by Nelson Education Ltd iat 2 25. 27. 132 (7001281) _ 65670 Nima? = 69 ENim? 0.010 @) p = atic? kg/m’? (b) Sar“ n(1.20x10") () W=mg = pig >W = (4.14% 10" )(1.0«10}(9.81) = 4.110" N It would be impossible for one person to lift such a heavy weight. The volume of the solid portion of the shell is A) te(0 15° 0.147) = 0.00264 m’ ‘Therefore, the mass of the solid portion is M = p¥’ = (6000)(0.00264) = 15.86 kp ‘The average density of the shell, including the hollow portion (assuming the hollow is evacuated), is given by Mo 15.86 fee Anois 3 3 Pow 1120 kg/m’* Since the average density is greater than the density of water, the shell will not float in water The force on the window from inside the submarine is (101325)(7 0.2") the pressure on the window from outside the submarine is P= pgh+ P, =(1030)(9.81)(45) + 101325 = 556018.5 Pa Fy 12732.9 N Therefore, the force on the window from outside the submarine is Fig, = PA=(356018.5)(x 0.2) = 69871.3 N Hence the net force on the window is BE Figg ~My = 9871.3 -12732.9 T1384 N=57 kN P= pgi=(1000)(9.81)(0.9~0.3)=5.9 kPa Copyright © 2014 by Nelson Education Ltd (2000x9.81) 7222287 _ 359 x02 31. (@) See the solution to Problem 8. 33. When empty, the volume of the ship submerged in water is given by ¥,, =(A)(9) m? If hand 4 are the height and arca of the base respectively, then Vy Pay Maw Ahp, But ¥, =4h 94 May a4, hp, M, 2x10° oe = * 917.86 m? 9p, (9){1020) Now, with oil the ship will sink according to S Py Alig If d is the depth to which the ship sinks, then = 4. 210° +4x10° (217.86)(1020) 35. ‘The apparent weight of wood in aleohol is given by =H, =(0.48)(9.81)-0.46 = 4.25 N But , = 2,27 >. Ps 4.25 Pg (790/981) Hence the density of wood is given by M_ 0.48 Vo 3.4810 = 48104 m? Pp = 880 kg/m’ Copyright © 2014 by Nelson Education Ltd Il 37. ‘The volume of the cork is 10 p 033 ‘The buoyant force on the cork is given by p.gV =(1000)(9.81)(30.3x10% ) =0.297 N 30.3%10 m? k, The weight of the cork is W = mg =(0.01)(9.81) = 0.0981 N Hence the tension in the string is T =F, -1W =0.297-0.0981=0.20N 39. The total mass when the beaker is half full is st m= sons 0) = 600 Og When the beaker is half full itis neutrally buoyant, that is Therefore the volume of the glass is Y, =¥, 500 = 140 om? And the density of the glass is At (ay Vib a Phas 700 = =0.69 %y Prue 1020 (b) For the boat not to sink, we must have Fs _ Prost ey a Pea = Poa § Pte Mass of the boat is (700}(5.5) = 3850 kg If there are NV survivors, then the total mass of the boat is m=3850+65N _, 3850465 35 NS27 £1020 134 Copyright © 2014 by Nelson Education Ltd 45. 41. 49. (©) Yes, some of the weight of the survivors would be offset by the buoyant forve corresponding to the volume of water displaced by their legs. The avceleration depends on the net force acting on the sphere, which is equal to the apparent weight of the sphere when fully submerged. 0 mB —PBV = Poa — pel But =ma=pya” Posh — pv i FBS )a =6.1 mis? 27 me-F, = pai 3g ee The aceelerat will decrease as the speed of the ball increases, due to drag forces from the ‘water. Although drag is the dominant factor, the density of the water also inercascs slightly with depth, so the buoyant force will also increase slightly with depth, which will also ast to slightly decrease the acceleration as the ball’s depth increases. Foree balance gives Prono SV = 98 + Phos BV m 280 Pans Poe 280.85 ave = 650 m? ‘The volume of the lake in terms of area and height is given by Vedh Differentiating with respect to time gives dv _ dh a de. aah lav a Ad Lav A dt = Ane Ane 1 . * 5 ——_ (2.5 x10" = 1,910") (6 «3600) =1.3 m 1.0010" M (@) Flow rate is given by AY _ 500x10 x OS (b) Since Q= vA Copyright © 2014 by Nelson Education Ltd 135 SL 136 ©) @ 17 om? @) ) P, =(5)(101325)+ F(1000)(11 4? — 45.6*) = 468.1 «10° Pa But negative pressures make no sense, so this problem is ill-posed Specifically, the pressure in the large part of the pipe is insufficient in reality to force water through the pipe at the given speed in the large part of the pipe. (a) Assuming height does not change, Bemoulli's principle gives 1 R+— py 2 Loe Pip 2 2° ou YL in the above equation we get Using Ay, a raR-toni a 02. pen Lamon.) ) -1]-tosa Copyright © 2014 by Nelson Education Ltd. 55. 37. 59. (). The percentage decrease in the blood pressure is PP 3.610? 13-410? BAB, 19g _13.6x10? =13.4200 R 13.4x10" x100=1.4% x10? m/s (@) Given: 4,v, 4107 3.18 m/s 70.02 Also, since 4.4, dv, _ 4x10" A ae According to Bemoullis principle AY, 3.18107 mvs 1 Lan Ape t sen At pent se >R-P, pal. n)+50(% —¥) =>R-P =(1000)(9:81)(12=7)+ 5 (1000)/ 1st isso) | AL x10" Pa .7x10" Pa =47 kPa (b) For maximum height, we will assume that the water barely reaches the top. In this case the Bemoulli’s equation becomes Bt PPL + Paw Rak 54ix10! pg (1000)(9.81) Bernoulli's equation indicates that the pressure along a flow tube increases when the speed of the fluid is reduced, Conservation of mass gives A Ay, oy e4y According to Bernoulli theorem Hs Li ay PEt TON E+ PM. ten Copyright © 2014 by Nelson Education Ltd. 137 1, 1 fA), = pent spr =n oe, +4o( 4) ¥ 2h +2pe(s1 OL. (a) Pressure in radial direction is given by 138 () © lie p=tpv Using v= awe get pahpr'a® 2 Differentiating with respect to r gives prey reo > dps prolar Integrating both sides we act fo- for sate Rx >P,-P, =pa' P= pe" sn -Bs pate ety Consider a particle of mass m in the rotating frame of reference of the fluid. In this frame the fluid is in static equilibrium. For simplicity we will look at the particle in xy frame only and then extend the result to the third dimension. ‘The centripetal force acting on the particle is in x-direction and is given by Fr, = mo! x 1 gravitational force is in y-direction and is given by F,=mg The angle the resultant force makes with horizontal is given by Copyright © 2014 by Nelson Education Ltd. Integrating both sides we get att 40 2g ‘This is the equation of a parabola. In three dimensions, this will form a paraboloid. 63. Given: 4=10% m? 013 x10" m? The pressure difference is given by ap _ HQ 8(107)(5)(5 x10") 28x10 Pa =6.3 kPa But Pane ~ Faeane = Pr Sh 2p,gh Pow >y 0) Peyrone = Paes evn Plame “Pae) BUt Prague ~Peane = AP oye PAA Par Copyright © 2014 by Nelson Education Ltd. 139 67. Pressure due to flow is given by . Where @ is the flow rate through area 4, therefore 2.0107 or ) = 20264.2 Pa Sinee F = PA = F = (20264.2)(70.01°) = 6.37 N Total torque due to water flowing out of both ends is given by Vf = (2(0.15)(6.37)=19 Nom 69. (b) As the elevator accelerates up, the effective downward force on the block increases to m(g + a) and therefore the block sinks deeper into water 140 Copyright © 2014 by Nelson Education Ltd. Chapter 13—OSCILLATIONS SS The total distance travelled in one eycle by a simple harmonic oscillator is four times the amplitude, or 44. The oscillator reaches maximum positive amplitude of 4 and maximum negative amplitude of 4. Since the oscillator returns to the same position after one eyele, the net displacement after one cycle is zero. For a mass-spring system, the period 7° is independent of the amplitude A because the period depends only on the angular frequency of the motion. This is evident by examining the equation of motion: x0) The period of a sinusoidal function is Acostent + #) ra2# o oof k which is independent of the amplitude of oscillations If the mass of the spring in a mass-spring osvillator were not ignored, the period would be greater, because Im 2a k If more mass is oscillating, the formula for period indicates a greater period, ‘The period of the pendulum is greater if the mass of the string is not negligible, because then the pendulum is no longer a “simple” pendulum. The formula for the period of a physical pendulum includes the mass of the pendulum, and greater masses mean greater periods. (a) When the elevator accelerates upward the effective value of gin the elevator increases Since 7 = 22, \e for a pendulum, the period will decrease. 8 The period of the swing decreases when the child stands up. If we approximate the standing fe ee child as a rod of length Z, then the new period is 7 ™ = 2a | =» which is te fee 3Ve *D than the period when the child sits and can be approximated as a point mass. Copyright © 2014 by Nelson Education Ltd. la 13, (¢) ‘The period of a physical pendulum in air is greater than in vacuum because air resistance slows the pendulum. 15. All of (a), (b), and (c) will not result in simple harmonic motion. For simple harmonic motion, the force must be restoring and a linear function of position. 17. (d) Atthe maximum amplitude of oscillation, the total energy F, is equal to the potential energy U, F, =—K4* . Ifthe amplitude is doubled without “kicking” the oscillator, 2 then the new cnergy is If the amplitude is doubled, the total energy increases by a factor of 4. 19. Using the information given at t= 0 we can solve for the amplitude. (@)_x(f)=Acos(er) x(0) = 40.3m = A cos(0) +0.3m ‘Thus, the amplitude is 0.3 m. (6) a= £ m S0Nim _ 55 sad’s 25 rads) -(0.3 m)= 0.47 m’s ©) dy, = 0°A= (VES rad's) (0.3 m)=0.75 mis? (a) Bither x(1)= (0.3 m)cos(1.61) or 2(f) =(-0.3 m)cos(1.6r) are suitable position functions, depending on which side of the equilibrium position the oscillator is located at = 0. 142 Copyright © 2014 by Nelson Education Ltd. au Because the mass is pulled to the left and then released, we can take the phase angle to be x radians. Equivalently, we can write the position function as mt) where 4 is a positive constant. Differentiating, we obtain cos(or) ve oAsin( et) (a) Substituting the given information into the position and velocity equations, we obtain 0.10 =—Ae0s(0.30) 0.857 = co-Asin( 0.30) Dividing the two previous equations, we obtain 8.57=—otan(0.3<) Using software, one ean solve the previous equation to obtain = 7.67 rad) and therefore the frequeney is e e128! f on 8 () x(9 =~A00s(0.300) 0,10 = —4e0s(0.3(7.67)} (©) The phase constant can be taken to be zero because of the form of the starting formula for the position function. ) x) —(0.15 m)cos(7.7¢) Because the petiod is 2.0 s, the angular frequency is 2m _ 2s rT 2 (a) An equation for the position of the oscillator is of the form = Acos(a1 +9) Copyright © 2014 by Nelson Education Ltd. 143 Because the position at ¢= 0 is x= 0, then 0= Acos(¢) and therefore ¢ 4 (You can obtain an equivalent equation for the position by choosing other values of ¢.) Thus, an equation for the position of the oscillator is x vor(ar+ 4) To determine the value of 4, substitute x =-0.4 and f= 0.5 into the position function to obtain / 0.4 = Acos| 0.501 \ 2 0.4= Acos(n)=—A 0.4 ‘Thus an equation for the position of the oscillator is outs at 5) (b) Differentiating the position function in part (a), we obtain 0.4m sin (=) and a=—0.4x* oa( 2142) Thus, the maximum velocity is 0.47=1m's and the maximum acceleration is 0.4m? =4mis* (©) The total energy of the oscillator is equal to the maximum kinetic energy, which is 0087? T= 0.081 Copyright © 2014 by Nelson Education Ltd. 25. No. Applying the given information to a position funetion x(0) = Acos(ot +8) , we get 2(0) = Acos() a & & 4 oe BIA RR AS 2) Equations (1) and (2) are not independent, so there is no way to use only these two equations to solve for the valucs 4 and ¢ . given the valucs of x(0) and x(T'/2) 27. (@) The lower the angular frequeney (the factor of 1), the higher the period; therefore, the periods, from high to low, are 32> 4>1>5 (6) The maximum speed is @A , so the maximum speeds from high to low are 5 > 1 > 4> 2>3. (©) The phase angles, from high to low, are 2 > 4 (4) The maximum acceleration is «” 4, so the maximum accelerations from high to low are S>1>4>2>3, 29. Considering only the rotational motion of the Earth, the period of a point on the equator is I day. The angular speed is 1 revolution per day, which is 2arad __ 2nrad TW 97y10- radia day 24%3600s 43200 31. Because the period is rar? k the rank of the periods, from small to large, is D< A“ B - shah = (0.03) gtk) E fee @ 47. @) (b) Copyright © 2014 by Nelson Education Ltd. 149 49, U+K =B and U=K 0K Lag? =1(Lee) 2 2 SL Thus. the periods are in the same rank order as the lengths: A, D, B, C, E, F 33, (a) T= 20 |&.100- 2002/2? _ 200.6 s z 981 (b) T= 27 [2 160 = 2002 | 10 _ _ 290.9, 972g (0.9972)(9.81) Yes, the true difference is 0.3 s and can be measured with a stop watch accurate to 0.1 s. 150 Copyright © 2014 by Nelson Education Ltd. (>) Ss_,_5s 5 T Th 36008 3600 ‘The positive sign is because we wish to increase the period HW, M4 1 _o.299, LT 3600 | 360 ‘Thus, the length of the pendulum must be increased by 0.28%. 37.1, = For + mi é T=29 | a sme ml? T=20 | fESE ae fc 2 wae fh 2R Ve. Sel saul "| el SE T=2 Ve SE 42k *SE Copyright © 2014 by Nelson Education Ltd. Ist 59. (a) (>) () qd) © © 61. @) ) 152 Reading from the graph, the amplitude is 0.6 m and the period is 3 s If'we use a cosine function to mode! the position funetion, then the phase constant is 075s, = xla= 38 2 ‘The phase at ¢= 1s is 7 FFAs AE a 2 6 ‘The phase at ¢= 2s is 2 242 -Ue 2 6 a =0, seoo{ 21+) Differentiating the formula for the position function, we obtain v()=-0.4rsin) ‘Thus, wl) ~oabesin( 2 2 vw) -0 rial 7) (1) = 0.63 m/s Differentiating the formula for the velocity funetion, we obtain 080° a(t) = oon %4()+2) 3 Thus. a(2)= a(2)=— a(2) = 2.3 m/s? Reading from the graph, the amplitude of the oseillation is 0.2 m, and the period is 3 s The equilibrium position is 1 m. Copyright © 2014 by Nelson Education Ltd. 0 (c) Reading from the graph, the phase constant is @ © © 8 xn els 38 x) = Acos(or+ g) +1 xO) ~o2se( r + 2) +1 se 0.2004{2 w= al a() =- a() = a() a(]) = 0.88 m/s? 3,12 Because the springs are identical, the total energies are in the same rank order as the amplitudes; therefore, the order of total energies is 1, 3, 2. aah Oscillator 2 has the lowest kinetic energy, as its speed is zero. It’s not possible to rank the other two, because we don’t know their masses Copyright © 2014 by Nelson Education Ltd. 153 65. (a) &*" =0.95 5 ba-2 n(0.95 2 lo95) b= 0.0064 ke/s ) et = Fm.s) 2m 2 in(ts) 2(0.25) Jo.on6ay(z 85} =158 t r Z, r (©) No, because the amplitude decreases according to a decreasing exponential funetion, not a linear function, 67, For A: We"! =10e"* = 6.1 em For B: 20") = 206" = 7.4 em For C: 30e%°) = 30e1* =6.7 cm 69. Let x(t) = 4e0s(or+ 4) (0) cos(g) 154 Copyright © 2014 by Nelson Education Ltd. a eor( E+) fofshoo-a(shoi] ( =-asin(g) [soy (Gy -# cos" (9) +(—A) sin (4) ) oot [+ i [eos (8) +sin®(#] ‘Thus, 4= bor |» I (0/4) -Asin(d) SIS 218 % e3 o 7 e18 a aa Also, = te 8° fo) eos) an) 5 af_2@ ay ‘Thus, p= tant! | — apo , Where Af, a pha p ane, pote ~ sak 3 M, = pl, = M fae =()v 2 RS R GM_m poe 2 pam M 2 oR ‘This is SHM, with the effective spring constant being GM R = . where f= mass of the carth and = radius of the carth Copyright © 2014 by Nelson Education Ltd. 155 ‘The time to reach from one pole to the other (one half eycle) is: = 42.2 min, and is independent of m. 73. When the object is pushed down a small distance x, there is a buoyant force in the opposite direction of magnitude equal to the weight of the water displaced: = ple = p Avy =(p,4g)s where (p,Ag) is the effective spring constant ra rn lt aan | O Ne PAS 78. (a) Use conservation of momentum to determine the initial speed 1 of the block: my=—mv'+Mu A mg Assuming mechanical cnergy is also conserved, we have 1 1 2.1 2 Aimy? =Lm(v +t B 2 z ( ) 2 Solving .A for v' and substituting into B, we obtain: Mu-my Lia 1 yf Mem) adage m 2 m me Fu2 -2nituy ¢ mv) + AE my? = MP? —2mbtuv+ nev? + mM 0= Mu [Au —2mv+ mul] O=u[ (m+ M Ju 2mv] Thus, cither «= 0 (the bullet misses the block) or: 2m us| \v m+M (b) The maximum compression of the spring (which is equal to the amplitude of the oscillation) occurs when all of the initial kinetic energy of the block is converted to clastic potcntial energy of the spring, dane = hit ar) 156 Copyright © 2014 by Nelson Education Ltd. Chapter 14—WAVES SS 1 (@) F, Fora transverse wave, the particles of the medium vibrate perpendicular to the direction of the wave. For a longitudinal wave, the particles move back and forth in the direction of the wave. () F. The wave speed of a mechanival wave in a medium depends on the elastic and inertial properties of the medium. (©) T. The speed of a wave in a medium can vary with wavelength; this is what causes the dispersion of sunlight into rainbow colours upon refraetion in a prism, (@)_ F. Mechanical waves depend on the movement of particles of the medium through. which they pass. (©) F. Since the speed of the wave depends on the properties of the medium, the speed will be constant in a uniform medium, (®_ F. A traveling wave always carries energy (g) F. The maximum amplitude cannot be greater than the sum of the amplitudes of individual waves (h) F. To produce a standing wave, the constituent waves must be traveling in opposite directions. ‘The light wave has the longer wavelength in this case, although it would be more accurate to call it an electromagnetic wave, because at this wavelength it is in the radio wave part of the electromagnetic spectrum, yeah 340 m/s 20000 30107 m 3x10" m/s 20000 s 2x10' m No. The speed of individual particles in the medium (the spring in this case) is not the same as the wave speed through the medium, ‘The wave propagates outward from the point where the stone hits the water surface. As the wave front expands the energy per unit length decreases. and therefore the amplitude decreases Copyright © 2014 by Nelson Education Ltd. 1s7 9. The answer is (c) for a mechanical wave, as is shown below (see Section 14-8): AK +AU i, = 4( whe) 2.2? cos" (kx — 2042) In Words, the total energy of a mechanical wave is proportional to the square of the frequency, so if the frequency is doubled the energy increases by a factor of four. 11. The two waves must have same frequency and amplitude to produce a standing wave. If the two waves have the same amplitude but different frequencies, interference will occur but standing waves will not be produced. If the two waves have the same frequency but different amplitudes, a pattern similar to a standing wave will be produced but without any nodes. 13, No, a difference in linear mass density could be offset by a difference in tension. (b) @=2af =2(10 Mz) =63 rad/s af =2.00m(10 s*)=20 m/s 17. (@) o=kv=20f kv _ (6.00 rad/m)(150 m/s) 2n 2a 43 Hz 1 © 4-2 i ae 2ho 7 __ 21.05 m k 6.00 rad/m 158 Copyright © 2014 by Nelson Education Ltd. 2 ‘The range of wavelengths is from 17 mm to 17 m. Wavelength range in the ocean: 280 Vz = 150000 Hz 500 mis » _ 1500 mis 25087 >» _ 1800mis OF 15000058 6.00 m 0 mm. Wavelength range in the air: f= 250 Hz f,=150000 Hz 340 mis 0.3 m? (1.5 m)—(4.0 mis )(2.08)) 412m? DQ.5,2.0) = = 69x10" m The maximum displacement is when the quantity (x—v1)is zero, The maximum, displacement is 0.25 m. ‘The minimum displacement is zero: 0 lim D(x, t) = i OO Ea +12 Copyright © 2014 by Nelson Education Ltd. 159 25. @) @) © @ &) 27. (a) 160 v= 3.0 m/s and the pulse is travelling in the direction of the negative x-axis DQ.3)= + =-2.36 em The maximum value for the displacement occurs when the value (x+3.00)is zero, so D=05m =a! _. 6.04(x+ 3.00) [(-soys0 +(x43.0r7 y cor +3.0¢}] DG,O= u(xt)= 209 = a u(2.0,3.0)= [> 0)(6.0+(2.0+3.0(3.0))") ‘Ppenge +3.0(3.0))] u(2.0,3.0)=1.2 em/s a -0.10{— = 2 020 ee 030 Dim) 0.0 oso 0.60 02s 020 ous 0 0s, Dim) 00s 1 PSE, a0 ~0.1s 020 02s xin) Copyright © 2014 by Nelson Education Ltd. (b) Reading from the graph, the leading edge of the wave moves 1 m to the right every second, so the velocity of the pulse is 1.0 m/s to the right. (©) For the given values of x and 7, |0.5—1.0|<1 , so the displacement is 2m. 42m if [vte|st von [-2m if |x+¢>1 29. (a) (mm) (b) The peak of cach pulsc occurs when the expression in parenthescs is zero, At f= 0, the peak of the D, pulse is at x= 10 m, and the peak of the D3 pulse is at x = 10 m. (©) The condition corresponding to the cancelling of the pulses is —— (x-SeH10) b4 (xe Se—1Oy 4 2f(45e—10) +4] -2[(-Se4107 +4] (x+5¢-10)' +4-[{s—Se410)'+4]=0 5t+10) -4=0 x £1060— 2054257 -1001+100— (2? 101-420-4257 —L00"-+100 20n—40x=0 20x(¢-2)=0 (x+5¢-10) +4- From the equation on the previous line, the pulses cancel when t= 2s (4) From the equation at the end of the calculation in Part (¢), when x=0 the pulses cancel for all times. Copyright © 2014 by Nelson Education Ltd. 161 31, First find the linear mass density of the wire. Hapa uel? 86 5) 2(0.020 em)") neosexi0 & Then use the formula relating velocity, tension, and linear mass density to calculate the tension in the wire. i mile 7 T, =v w= (160 mis) (9.88%10" pom) = 252.855 mis x som lm 1000g E 7 =25 SBM _95 5 § as 0.10 kg 9g 193 KE 30.0m ™ 100.0N = 0.2 km/s 20-107 kgim 35. First calculate the linear mass density of both strings in kg/m. fi g-lkg 3 4, =2.0 pin =2.0 218 2.910 kom y= 20 8 sideee Sw) g-lkg 3 =5.0 gm=5.0 2S __5.9,107 kgi By 28 Bm => -1000 8 em ‘Then calculate the speed of each pulse using the same value for the tension since the strings are attached and the tension is constant. 50.0 N = 1610? m/s 20-107 kgm ‘Now caleulate the time it takes for pulse A to travel the length of string A (10.0m). Then calculate how far pulse B would travel in the same time. After this time, the pulses will be travelling on string B only, and will therefore travel at the same speed. 162 Copyright © 2014 by Nelson Education Ltd. aL =(1.010" m/s)(6.3x10* s)=6.3 m So the pulses are now 20.0 m ~ 6.3 m = 15.7 m apart, and they travel at the same speed in string B, Therefore they meet half way between their current positions, 6.3 m + 6.85 mi = 13.2 m from the far end of string B. (@) ]=(us mJ? = mis aa y gh (©) From the result of part (b). £ 2a And therefore 9.81)(12) 2a =19 m/s All four waves move in the positive-x direction, because the argument of the sinusoidal function in each case is of the form kx-« . Because the wave number & is the reciprocal of the wavelength, the rank of the waves in order of increasing wavelength is b -0.5 mis k 2n/3 ‘The phase constant is 0.2 rad. The phase of the wave is 4.0x—0.5¢+0.2= 4.0(0.5) -0.5(2.0)+0.2=1.2 rad ‘Yes, the phase changes linearly with time for each fixed value of x The phase difference is (40x-0.51+0.2), -(4.02-0.5'40.2), 4.0(x, —4 )-0.5(4, —4 + 0.2-0. 4.0(0.1)= 0.4 tad ‘Yes, the phase difference is the same for all fixed times for the given difference in a-values, For a fixed time, the phase difference between two points that are one wavelength apart is 4.0(x, 407% < 27 tad 40 which is equivalent to 0 rad. Copyright © 2014 by Nelson Education Ltd. 165 57. P=—pra® 2 1 |r ao salFe 2npy a Pa ffaae pe P = {(200.0)(0.1000)27? (10.0 (0.010) P=0.88 W 59. (@) (b) A=05,9-1 (©) The two component waves have the same frequency and wavelength, so they have the same speed. Consequently, the superposition of the two waves also has the same speed. (4) ‘The resultant wave is a travelling wave, because the two component waves travel in the same direction 6L resultant From the graph you can read the phase constant is approximately ~0.4 rad. 166 Copyright © 2014 by Nelson Education Ltd. 63. 67. 69. Using Equation (14-37), = 2.636 rad ‘The phase difference between the resultant wave and either of the two component waves is Pat ais rad 2 (a) The minimum amplitude, which occurs when the two waves are out of phase by half a oyole, is |4,~A,| (b) The maximum amplitude, which occurs when the two waves are in phase, is 4 +4, Using the identity sin(a)+sin(5)= 2eos( 2 S}o (2) and letting @ and represent the arguments of the constituent waves, the resultant wave can bbe expressed as sin Dox. 1-24 Dest) 2asor AH in MB 24) Den= ee a (wna Ath) [aceredl mere} (fpmered emer} (a) For ahard reflection, the wave is inverted and travels in the opposite direction, therefore, its wave function is D(x.t)= 0.2sin(3x—41+ 7) (b) For a soft reflection, the wave is NOT inverted and travels in the opposite direction; therefore, its wave function is D(x) =0.2sin(3x- 41) Copyright © 2014 by Nelson Education Ltd. 167 7 168. (a) ‘The frequeney of the standing wave is Be-10 Hz and the wavelength is 7 22 105m 06 (b) The distance between consecutive nodes is half of a wavelength, which is 5.2 m, and the distance between anode and an adjacent antinode is a quarter of a wavelength, which is 2.6m. (©) D,(x,£) = (0.750 om )sin (0.6x— 2078) D1) = (0.750 em)sin (0.6+ 2077) (A) D(0.20,3.0) =(1.50 om)sin(0.6(0.20)) eos (207(3.0)) D(0.20,3.0) =(1.50 om)sin(0.12}cos (602) (0.20, 3.0) =(1.50 om)(0.1197)(1) DO.20.3.0) = 0.18 em (a) D(x.t)=(2.0 mm)sin (rx) cos(0.5rt) (b) ‘The first three nodes on cither side of x= 0 arc located at x=!1m, 12m, 13m . The first three antinodes on cither side of x= 0 are x=+0.5 m, +15m, +2.5m (c) Based on the result of Part (b), the distance between two consecutive nodes is 1.0 m. Using the idontity sno) +sin()= 200 = }s( 22) and letting a and 6 represent the arguments of the constituent waves, the resultant wave can be expressed as koe — ort kx + ot [hoe eat + [koe oxsty= nto Fare Tiron Da ae art, 2 DO.) = 2utes(-o# +454) n[ Bette) cA wt Do.n= 2st Copyright © 2014 by Nelson Education Ltd. 29. 81 ee cote wn HOSNe Ve When the tension is increased by 4%, the new frequency is i a0 wes 0198(260) = 265 Hz “ H f (a) The longest wavelength standing wave possible is 4.0 m, because at least half a wavelength must fit into the length of the string. b © fo 200.0 1" 3020) f=50 Hz, (©) No; the allowed frequencies are whole-number multiples of the fundamental frequeney. (@) The frequency af the second harmonic is 2(50) = 100 Hz, and the frequency of the fourth harmonic is 4(50) = 200 He, k (a) The fundamental frequency is iF QV a 1 25.00) 2(1.000 m) 0.000650 kgm f= 98.1 Hz sco harmonic fourth harmonie (b) The frequency of the second harmonic is 298.1) third harmonic is 38,1) = 294 Hz. 96 Hz, and the frequeney of the (©) The wave speed is 96 m/s Copyright © 2014 by Nelson Education Ltd. 169 (@) The new fundamental frequency is LfE ale pt 5.00 N 2(1.000 m) ¥ 0.000650 kgm f=116 He Copyright © 2014 by Nelson Education Ltd Chapter 15—INTERFERENCE AND SOUND 1, T. Yes, sound waves are longitudinal 3. The speed of a wave is the product of its frequeney and its wavelength, and the speed depends on properties of the medium. Thus, if the frequency doubles, then the ‘wavelength is divided by 2, or equivalently, is multiphed by a factor of 0.5, 5. T. As for light waves, angles of incidence and reflection are equal for sound waves as well 7. T. Yes, confining a wave between boundaries results in interference that produces a standing wave provided that the wave has an appropriate wavelength 9. F.Itis possible to have a number of different standing waves exist at the same time in an air column; they will combine to produce one sound. 11. Consider two sources of intensity J, and Jp, respectively. The corresponding, sound intensities are i, and B. Then, B= Wlog,, (4) and f, Pk tobe A -A~t0l ey arn] Now, if y= 2/, then Copyright © 2014 by Nelson Education Lid. 7 17. 19, 2 172 ‘Thus, the increase in sound intensity level is 3.01 dB when the sound intensity is doubled, . (@) Sound waves can interfere both spatially and temporally. ve fh ‘v= 343 m/s for air at 20°C ‘The speed of sound in water at 20°C is 1482 avs. ayers v 30m +30 G45 5 1482 mis s(ag= , on( 2-20) Z Now, f/=60 Hi, 50 2a =377 5" For sound in air, y= 343 mis, $0 sm _ nf _27(60) Ay 343 “Thus, the displacement amplitude ean be expressed as s(xf)=5, 60s(I.bx—377¢) k Ld ‘The amplitude of pressure variations is Ap, Bks, where s, is the amplitude of displacement variations, and the bulk modulus of air is .01x10° Pa, Qn _2af _ 2a(1000 Hz) zoo Mims _——_ A ___ (1.01«10" Pa)(18.32 m . Ap, = Bks,,, 80 when the displacement amplitude doubles, so does the pressure amplitude, Copyright © 2014 by Nelson Education Lid. 25. vy _ 343 mvs ot SS erikte fo“ 31, ~ 200.15 m) = 8m eso He 2. 2(0.20m) 29. The beat frequency is 296 Hz~ 294 Hz= 2 Hz, 31, The beat frequency is 440 Hz~ 350 Hz = 90 Hz, A =100f Lh =e 24) h =10]re(2)ne( 4) BR ~1oe(2) +1092) Pi, =1log(2)+95 4B 2B, =3.01 dB+95 dB f= 9848 Copyright © 2014 by Nelson Education Lid. 173 km _ 80000 m 35. 80—" = 2.2 m/s h 36008 vty, ay VEY, 34340 5 (100.0 He £ 35-2! ) Jf, =748 Hz vy, +¥, =19.68 ons Assuming , and y, are comparable, then each of them is about 9.8 ems. 39. Light takes only very slightly more than 0 to reach you, so this, time is negligible. The distance travelled by sound in 4s is (343 m/s)(4 5) =1.4 km. 41. 5, (x)= 10c08(kx— or) x sat) <2 tr mea) (x 15 (st) =1Bo0n( 0) 20004{ bro) 5, (ut)+ 5, (x1) = 10008 (ke ot) + 20.0r(e—onjeon( 2) # 20sin (asin) 174 Copyright © 2014 by Nelson Education Lid. 45. 5 (4.145, (34) = (10+ 102 Joos (sor o1) $102 sin (e—eor) CF) 5 (ut)+ s(t) cos(kxv— ot 0), for some constants 4 and 0 Expanding the expression on the right side of the previous line, we obtain Acos (kx — at Joos 0+ Asin kx ct)sin Comparing with equation F, we obtain Acos#=10+1042 and asin #=s10V2 Squaring and adding the two equations in the previous linc, we obtain A cos? 0+ Asin’ 0=[10(14+V2)] + [10v3] A (cos? O-+sin* @) =100[1+2V2+2+2] # =100(5+2V2) oy(5+2V2) A= 27979 4=28nm T= 110°" =10* x10" 10” x10%* =10"" x10% T= 3.16 «10% Wim? = 4n (23) (3.1610) P=21<107 W P=21mW 130000 “The speed of the car is 130% = nh 3600 8 vty, Lah vey, ‘The frequency received by the moving ear is £36119 GHz) Copyright © 2014 by Nelson Education Lid. 175 ‘This is the same frequency as the reflected waves, whieh are received back at the radar unit with a frequeney of (oi) = pom fe 3x10" 36.1 (35 +361 f, =9.999997593 GHz ‘The beat frequency is f, — f, = 24000 Hz.= 2.4 kHz Jo Guz) 47. G, 392.00 Hz C, > 523.25 Hz E, + 650.26 Hz G, + 783.99 Hz Similarly, 659.26 ~ $23.25 =136.01 Hz = f, 783.99 — 659.26 = 124.73 Hz = f, ‘The average value ig = 2475+ 1601+ aL vedLy, AL -$ 4(270 kim) “~123h = 86.4 kaw +i 176 Copyright © 2014 by Nelson Education Lid. 1, 20108" 2 = pone A reduetion in sound level by 60 dB corresponds to a reduction in intensity by a factor of 10 =10~, Thus, the new sound intensity is 0.2 4.W/m' Let y represent the depth of the well. The time needed for the stone to drop is t, where ‘The additional time needed for the sound of the splash to reach the top of the well is t,, where te y 343 ‘The total time is 2.8, 80 4 +4 —2.88 ——— 42) 428) 343? y = 25871 m or 35.7 m. 25871 m is rejected because f, by itself would bbe much greater than 2.8. ‘Thus, the depth of the well is 35.7 m. Copyright © 2014 by Nelson Education Lid. 7 55. Because the sound is loudest when you are equidistant from the sources, the sourees arc in phase. The condition for destructive interference is ade(ne da 2 where n is a natural number and 4 is the wavelength of waves emitted by the sources, Thus, f=122.5(n+4) 5 If this is the lowest order minimum, then = 0, and 57. (a) For the fundamental frequency, half of a wavelength fils into the length of the rod, so A, = 240 om. 6420 m/s 24m = 2675 Hz = 2.68 kHz 59, Sce the derivation of equation (15-23) in the text, which includes phase angles. 61. 141,47, =7,108" + 7,10 47,108” j [10™* +10 410° ] = £, [10% +10" +10") [2.00316 x10" ] 178 Copyright © 2014 by Nelson Education Lid. ‘To reduce the intensity sound level to 85 dB, the equipment must be used a distance r from the perimeter, where Copyright © 2014 by Nelson Education Lid. 179 Chapter 16—TEMPERATURE AND THE ZEROTH LAW OF THERMODYNAMICS TS 1. The two thermometers will agree provided that they behave in the same way as the temperature changes. The most likely value is not necessarily the average value; it depends on the shape of the probability distribution (a) Symmetric Probability Dist with Equal Mean and Most Probable Values 025 020 os ony) 0.05 od 0 3 10 520 o> lity Distribution with Uncqual Mean and Most Probable Values 02s 020 os 0.10 os of o 5 0 0 The triple point of water exists at exactly 0.01°C of 273.16 K. If liquid water did not expand when cooled, the melting point of ordinary ice would not decrease as a fimotion of pressure and the triple point would be the limit below which water would not exist in liquid state 7. No, because the gascs do not necessarily have equal concentrations (N/V) 9. No: for an ideal graph the corresponding graph would be a straight Line that would pass through the origin if extended. 180 Copyright © 2014 by Nelson Edueation Ltd il 15, 7 19, 24. ‘As the Sun rises, the temperature of the land increases more rapidly than that of the water, and the air just above the land becomes warmer than the air above the water. As a result the air pressure on land is lower than the pressure above water. This pressure difference forces the cooler air above the water to move toward the land, which we feel as a wind. For an ideal gas the relationship between pressure, volume and temperature is given by PV=NkE. Therefore, if the temperature inereases while the volume is reduced by half, the pressure will more than double. ‘The average volume occupied by each gas particle is 224% _ 22.4x10" em? 6.02x10" 6.02 x10" ‘The average spacing between centres of particles is =3.72x10 em (8.72310 om)!" = 3.341077 om =3.34 nm ‘No: when a system is in thermodynamic equilibrium, all macroscopic properties, including temperature, remain constant 101325 Pa fa) 110 psi =110 psi x 21°22 FS _ 758493 pa= 758 kPa @ NOs PS" [4.69595 psi (b) = 2arx A= 2m (35,0 em)(4.00 om*) = 880 em’ (6) ny? 12410 5 N apy 3(758423 Pa)(880 10" m°) Ne = 1 61x 10 particles 1.24x10™ J 1.24x10™ J pe 1.6110 particles “6,02 10" particles/mol N=0.268 mol (4)_mass of Nitrogen = 0.268 mol 28.014 g’mol = 7.51 g .268 mol «4.0026 g/mol = 1.07 g mass of Helium AL 1 LAT AL= (: kd x Je (00 m)(40.0°C = 20.0°C) a aLAT AL =680x10% m AL =0.68 mm Copyright © 2014 by Nelson Eduestion Led. 181 23, 40°F = (40°F -32°F) x 25. (a) 58°C = 58°C x” 4 3 F 136° F 3G (b) 58°C + 273.15 = 331K 7. @ Pram n=Pl a (5000 Pa )(792 m*) (1.38%10 J-K)(220 Ky 1N=1.30%10" atoms ‘The mass of helium is 1.3010" atoms =— «4.00 g/mol = 8.67 kg 62x10" atomsinol . (b) PV = NET yo MAE. (130x107 )(L.3810™ (293.16) P 101000 V=s22m' (c) The number of moles of helium lost is PY _ (101000)(65.0-52.2) 1 Oe mol (1.38x10™)(293.16) 6.0210 = 530 mol NT > (02x10 (1 38%10)(273.16) 101325 29. (@) PY =NKT >Y V V = 0.0224 m? 2.4L 3(101325)(0.0224} 0.028 Ls2. Copyright © 2014 by Nelson Edueation Ltd. fo (v), = 22 ootszsyowz20 ©) (y et mN 0.032 Nig, = 46 L's A) (Mac)! (one )y = 11.069 31, Fora temperature of —81.4°R, si 3{1.38%10}(210) (rq) = SE SO) econ m 28x10" (6.02%10" } Vg, = 432 mm/s Por a temperature of ~ 38°F, | = 294623 543 m/s = 0.54 km/s 33. ‘The average molecular speed is Yng = [FF 0 ma fea(sBa) ve Vg 4a ae y [ret lay nts, Letr= so dee vay Thus, aa (sa) (\)¢~ "de weRy? Copyright © 2014 by Nelson Eduestion Led. 183 aL 43. is Reading from the phase diagrams. water, oxygen, and carbon dioxide are in the solid phase. and nitrogen is in the liquid phase Answers will vary. For a 75kg person, the mass of water in the body is (0.6X(75) = 45 kg. am), =e 3(1.38x10™ (1x10) 35)(6.02 x10" mot” py (150000 Pa)(30 10” m?) Nk (3x6.02 x10" (1.38x10) T=180K ‘The distance travelled in time Af by a particle in an ideal gas travelling at speed v is vAz, so the average number of collisions ‘with the walls of a cubical enclosure of length Fin time Avis “t ‘Thus the number of collisions per unit time per unit area is vAt Z})_viy ac ‘The total number of collisions for all particles, per unit time per unit area, is (a) v Copyright © 2014 by Nelson Eduestion Led. From Equation 16-13, 5 3PV 80:0) ar GPr and therefore v,. % m mN Yee), ‘Thus, the total number of collisions per unit time per unit area is N ay fpr _(6.02x10") [3(101325 Pa)(0.0224 m* 7)" FV nav” (00724 m) aR 3x10" m The result is not exact, because (,, ‘5 A more precise calculation introduces a factor of 4, and yields the more accurate result of 3.3 x10" mst 45. @) PV =NIT > e=(E her Now, for hydrogen atoms, 1000 150000 * x 6.0210 particles/mole 1.007948 mole 8.96 x10" “(2 68x10" Pa 8.96 x10" }(1.38%10 ” )(13.6x10°) If instead we assume that in such extreme conditions hydrogen still exists in the form of molecules, then the pressure is half as much, P=8.4 x10" Pa, Copyright © 2014 by Nelson Eduestion Led. 185 49. SL SPINE (*’). SEN(XY' 3.359x10" em 5.8%10° mis (b) pain) %, If we consider the hydrogen to exist in molecules, then m is twive as large, and y,,, = 4.110" m/s 100 %0 t E 3 ie 20 o —80 ~70 —69 —$0 -49 -30 -29 Temperature CC) Using software, we obtain the line of best fit R = 0.197 + 117.24. For a temperature of 400 K, the predicted resistance is R=019x 400411724 R=193Q 190 180 170 yO + 17.26 160 150 40 130 120 50 100130200350, 3008 TK) Ray Answers may vary. Copyright © 2014 by Nelson Edueation Ltd. Chapter 17—HEAT, WORK, AND THE FIRST LAW OF THERMODYNAMICS TS 15 A constant volume process should be used, so that none of the supplied thermal energy goes into doing work on the sample of gas. A: solid only B:; solid and Liquid C: liquid only D: liquid and gas E: gas only All surfaces, including our skin, lose heat through radiant heat transfer with rates that differ in winter and summer months. The reason is the unequal temperature differentials that exist between objects and their surroundings in winter and summer months, Hence in winter months we lose heat at a faster rate than in summer months. ‘The force done by the systom on its surroundings is positive. The energy conducted through a thermal insulator is proportional to the temperature difference across the insulator (see Fquation 17-39), so if the temperature difference doubles, the energy conducted also doubles. ‘The food reaches an equilibrium state, and so does not undergo further changes. Because the amount of thermal cnergy that flows into one object is the same as the amount of thermal cnergy that flows into the other object, we can write me lat] =m. and therefore m mer Thus, for the final temperature to be the average of the initial temperatures of each object, the product of the mass and specific heat capacity of each object should be the same. In other words, the heat capacities of the (wo objects are the same. ‘The total energy that must flow is Qe moar Q=(1.0)(4186)(95) Q=397670 3 Copyright © 2014 by Nelson Edueation Ltd. 187 7. 19, 2 188 ‘The time needed is 397670 J 500 Vs Ar=795 s Ar=8.0%10" s Ate myeyATy =~ Maen Te mye y (I, ~100) = ~ tiga yea (1, ~10) yey T, 10000, = Moen onal +10 ge oe Myln Te + Marat: = LOO yop + WOM T, (IC + gue ner ) = LOOM 04 + LOM gC 1, = 00min + 10h ga ae Man + Pats 100(0.1}(897} + L0(0.5)(4186) + *(0.1)(897) + (0.5)(4185) T=l4 The final temperature of the aluminum and water is 14°C Q=mcal oss ° (oT kg)(SK) = 130 Ikg-K) ‘The amount of thermal energy needed to warm 20 g of icc to a temperature of 0°C is Smeal =(0.020)(2100)(4) Q,=168 1 The amount of thermal energy needed to melt 20 g of ice at a temperature of O°C is O,=mL Q; =(0.020)(. oO, x10") = 6660 T Copyright © 2014 by Nelson Edueation Ltd. 25 The amount of thermal energy that potentially could be supplied to the ive from the water to warm it to O°C and then melt it is Q,=meAt Q, =(0.1)(4186}(30) Q,=125585 ‘Thus, there is plenty of thermal energy available to melt the ice completely. The remaining energy. O,-(O, +@, )=12558- (168+ 6660} =5730 J results in the final temperature of the water being ocsar= me S730 J (o12)(4186) Tell4ac The final temperature is 11°C Because the process is isothermal, 7" is constant, and PV =nkT RL Vv Pp ‘The work done in the process is y= [Pav r pe it 1 W =nRT|)—dv I, W = nkE [Inc]? W = nk [In(V,)—ln(F,)] : wero!) W =(3)(8.314)(373.16 )in (1.3) Wy = 24005 By the first law of thermodynamics (sce Equation 17-16), in an isothermal process the work done by the system is equal to the thermal energy that flows into the system; thus L00 J of nergy flows into the system Copyright © 2014 by Nelson Edueation Ltd. 189) 27. Using Equation 17-36, P=ocAT" PL oer A P = “ 5 (5.67 x10" )(0.5)(630+ 273.16) P = 21 kWim? x (401)(4x10™)( 400-100) 0.005 P=9624W P=96KW 31. OSCAT 2 At At Me 400 (0.05) 400 Js 0.05Kis C=8.0kKIK 000. VK OT kg 0 KTckg-K) 33. The amount of energy needed to evaporate 4.0 kg of water is O=ml. =(4.0 kg)(2.26%10" kg) O=9.0%10° J fall of this water is evaporated in 2 h, then the power delivered to the water is 5 2 9.0108 J At (2.0 )(3600 wh) 3 kW 190 Copyright © 2014 by Nelson Edueation Ltd. fall of the heat to evaporate the water is generated by the athlete, then she must actually generate more power, because she will have to power her movements and her other body processes 33. W= [Pav w= font dv Vr 1 = ort {hay W = nk. ra W = nkT [ny]? W = nk7[In)—nh)] w=nevin{ 4 (1.5)(8.314)(273. rom( 3) W =-37405 w 37, Assume for simplicity that the shape of the shrew is a cylinder of length 0.04 m and radius 0.01 m, and that the shrew’s emissivity is 0.5. Then the power radiated by the shrew is Post" P=os(2ak? +2aRh)r* P= (5.6710 *)(0.5)(24[0.01 }+ 22 [0.01]f0.04])(273 +36)" P= (5.6710 )()(0.0005)(309)' P=081W 39, Because the intemal pressure is constant, the work done by the gas on the piston is W=PAV W=P(%,-%) W =(101000 Pa)(15.0-10.0) m’ W=505 ks ‘The change in internal energy of the gas is AU=0-W AU =1000 kI-505 ky AU =495 kJ Copyright © 2014 by Nelson Edueation Ltd. 191 41. ‘The work done in the constant-pressure process is W=PAy W = (1909 kPa}(7-L1)x10° m* We-16kI No work is done in the constant volume process, so this is the net work done. ‘The initial temperature is BY, =nRI, ‘The final temperature is RY, = RT, Bh on 3000 kPa)(7<10" m™) ‘The change in internal energy of the system is 0, because the initial and final temperatures are the same. Thus, there is 4 net flow of thermal energy into the system O=W-sU Q-W-0 O=-76kI 43, Because the process from A to B is isothermal, we can calculate the volume of the gas at B as follows: BY PM Ay BR, (1.00.10° Pa) (0.50 m*) 1.50x10° Pa 192 Copyright © 2014 by Nelson Edueation Ltd. Similarly, the process from C to D is isothermal, we ean calculate the volume of the gas at C as follows: PY. = PY. 0-00-20" Pa}(200 m) ~ 1.50x10° Pa V.=1.33 m? Now we have the information needed to calculate the work done in each phase of the process: Wey = mk wf We Wy =(1.00x10" Pa)(0.50 wo) 5) 7 1.00%10" Pa)(o s0m'}a(2) -20.3KI PKs) Wy =150 kd Wo =nRT wf We, = PVA . aval 20 Wg =(1.00200° r5)(20°)in( 2) We» =(1.00x10" Pa}{2.0 m’)In(1.5) Wop = 81.13 Wy = AW -K) Wy, =(1.00%10" Pa)(0.50 m? =2.00 m?) W, 150 kd Copyright © 2014 by Nelson Edueation Ltd. 193 ‘Thus, the total work done in the entire process is 20.3 kJ + (150 kJ}+81.1 kJ +(-150 kJ) LAI Woy + Wy Wop + Wy Woy tig Wrap Wy 45, The initial volume ean be ealeulated as PY =nRT, yo PRR ‘Pp (1)(8.314)(273.16+ 50) 100000 0.02687 a? ‘The final volume is three times the initial volume: 3%0,02687 m? V, =0.0806 m? The final temperature is (100000 Pa)(0.0806 m’) (8314) 7,=970K ‘The work done is W=PAV W = (100000 Pa}( 0.0806 m’ — 0.02687 m*) W=537k ‘The change in internal cnergy is Lo Copyright © 2014 by Nelson Edueation Ltd. ‘Thus, the amount of thermal energy that flows is Q=AU+W oO 06 kI+5.37 kJ =14ks 120 10 100 ee Pars) 0 05 1s 25 38 Relative valurne (F77,) The quantity on the left side of the previous equation is known (it’s the power delivered by the heater), and the quantity in parentheses is the slope of the plotted data. Using software, one can determine that the slope of the line of best fit for the plotted data is 0,0237056, Therefore, Q ar me( At Me 10.0 Vs (4186 ekg K)}(0.0237086 Kis) m=0.1kg Copyright © 2014 by Nelson Edueation Ltd. 195 Chapter 18—HEAT ENGINES AND THE SECOND LAW OF THERMODYNAMICS TS 1 9. uL This is not a good idea, because more thermal energy will be deposited into the room (trom the heating coils on the back of the refrigerator) than will be removed by cooling, as the refrigerator is not perfectly efficient The efficiency would still be specified by the Camot cycle efficicney, which depends only on the temperature difference between the hot and cold reservoirs. No. If the temperature difference between the waste energy souree and the hcat reservoir of the heat engine is too great, the heat pump will use more energy than it transfers to the heat reservoir. The reasoning is not correct since moving around to clean up the room increases the entropy of the Universe even though the things in the room become more ordered Another example is a pendulum swinging in a vacuum on a frictionless pivot. The time- reversed motion is also perfectly realistic. 100 Ha. 1h @) 5.8 100 km, h 3600s =1.6 mL/s () 3 S3 kis 196 (b) The refrigerator must remove thermal energy from the interior of the refrigerator at the same rate that it flows in through the walls, so as to maintain a constant temperature. (©) The amount of energy released per second by the propane combustion is kg Iday 0h 50M 0,50 88. x kg day 24h 36008 290 W (@) The amount of thermal energy delivered to the surroundings extemal to the refrigerator is the energy delivered by the propane burner and the energy delivered by the back of the refrigerator, which is 290 W + 12.5 W= 300 W Copyright © 2014 by Nelson Edueation Ltd. (500 + 273) ijntn eee) (1000+273) (| =0.3928%100.0 ky \w|=39.28 ks r]=3.9 Mr 19. (a) ‘The (otal amount of thermal energy that flows is Q=meAr Q=10me and therefore the total amount of thermal energy that flows per mole is, 10me n where m is the mass of the gas and is the number of moles of gas. (6) ‘The change in entropy of the reservoir is Copyright © 2014 by Nelson Edueation Ltd. 197 24. 198 () @ © (a) >) ‘The change in entropy of the gas is AS=me[inT],” nef In(r-)-In(7,] T, ssemen(Z] i As ‘The overall change in entropy of the system is AS =0.05me -0.04879me AS = (1.21010) me The net change in entropy of the system is positive, so the process is not reversible. ‘Using a value of 43 J/(a'K) for the heat conductivity of steel, the rate at which thermal energy flows through the bottom of the pot is AST x p_ (43)(0-030)(500.0-10.0) 0.010 P=63kW ‘The rate of change of entropy of the bumer (assuming its temperature is constant) is AS _Q/At at T ARLP ArT AS -63 KW. At (5004 273)K As ae Ko) Copyright © 2014 by Nelson Edueation Ltd. (©) The rate of change of entropy of the water is calculated as follows 2) T d= == *53 KW a 273410) K 120 JK -8) Note that the rate of change of the entropy of the water depends on its temperature; the calculated value is the initial rate of change of entropy. (@) The net rate of change of entropy (initially) is 220 JK -s)—82 (Ks) 40 MK -8) 23, ‘The volume of such a cube is V=(20nm)y V =(2x10 m) V=8x10" mw ‘The density of silicon is 2329 kgm’, so the approximate mass of silicon atoms in the cube is m= pl =(2329)(8x10") =1.86 x10" kg =1.86 x10" g Because the atomic mass of silicon is 28.085 g/mol, the number of moles of silicon in the cube is ‘Thus, the approximate number of silicon atoms in the cube is 66x10 molx6.02%10" atoms/mol = 4x10" atoms ‘Thus, there are about 400,000 silicon atoms in a cube with edge length 20 nm 25. (a) AS men( as (200.20?) s45)1( 7 AS =-0.77 JK Copyright © 2014 by Nelson Edueation Ltd. 199 27. # = (200.0 x107)(385)In| — (20n0-a0?)asspa{ 3) AS =-53 VK @) AS =metn| 2 (d) AS = me! ee 1 AS =(200.0x107*)(385)In| —— (20n010*)oasymn( O22) AS 53K No. Heat engines and heat pumps are never 100% efficient. The amount of work that the heat pump could do to restore a temperature difference would always be less than the energy input into the engine, which would make the temperature difference even smaller. ‘The data supplied indicate that the engine would have an efficiency of 535.5%, which is greater than the maximum possible effivieney of 50% for the given temperatures. Since the data are erroneous, the application should be rejected. For simplicity we will assume that if there are 1 particles they are evenly spaced out along, the length of the system Z. We will also assume that the spacing of the particles does not get less than AL, the width of the detector. There will then be N intervals of time, each of duration AL’v, where a particle is in the detector and the detector reads Y,, The average detector reading will then be ty (imi 5 Jr (Qar =r, Similarly, we can calculate the square of the standard deviation as } i -¥) at “He -2F +P Jar Copyright © 2014 by Nelson Edueation Ltd. To see how this behaves as the number of particles inereases, itis most interesting to examine the ratio ] ‘This ratio goes to zero when NV =1, At this point, there is one particle in the detector at all times, Clearly, the simple model we have used here breaks down when the number of particles > ‘The inorease in the entropy of the water is the sum of two terms, the increase in entropy as its temperature is increased to the boiling point, and the increase in entropy as it vaporizes asence(Z)-2 3; T Te As =meotn{ £|4 Ty T kg)(22.6%10° Jk (1 ks)(22.6 10° kg) AS =(1 a) 4186 749-59) a AS= KK ‘The change in entropy of the wood is AS Ag = 22x10" J (500+ 273) K AS =-2.6x10" VK ‘The change in entropy of the room is Ag = 20x10 204273) K AS =6.8x107 WK ‘The net change in entropy of the entire system is 68x10" VK -2.6 410" VK =4.2x10" JK. Copyright © 2014 by Nelson Edueation Ltd. 201 37, Because the volume is constant, the work done in the process is zero, Thus, the amount of thermal energy that flows is equal to the change in intemal energy of the ideal gas. Thus, the change in entropy is nr[in(z,}~In(7,)] Tr.) AR In| (*) we) syoy(sst4)m( SO 39, Assuming that the eoeflivient of performance depends on the difference in the temperatures as follows where. is a constant, Using the given data, the value of A is CPyAT A= (3.5)(20-10) 35 When the air temperature outside drops to 10 the coeffivient of performance is ce 202 Copyright © 2014 by Nelson Edueation Ltd. 41. (@) Assuming that the coefficient of thermal conduct which thermal energy crosses from water to air is xa a (2.25 W/an-K))(10 K) d P=(22.5/d) Wim* ity of ive is 2.25 WAan'K), the rate at P where dis the thickness of the ive layer in metres. (b) ‘The rate at which the entropy of the air changes, per square metre of ive per second, is (c) ‘The rate at which the entropy of the water changes, per square metre of ice per second. is as-2 T 273.15 ag = 28240 wK (4) To form another centimetre of ice requires the transfer of the following amount of energy per square metre of surface: ml pL Q~(1000 kgim"}(0.01 m')(3.33910" 1 2 ‘The time needed to form an additional 1 em of ice is 3x10° J Q_ 333x101 P 25/d Ws Qayasni0'ds =4Lidh SIO Ke Copyright © 2014 by Nelson Eduestion Ltd. 203 43. 45 47. (©) Letting B be a constant of proportionality, Bxdx = dt [Bxav= far Ba Seer 2 ZG B where ¢ is a constant related to the initial thickness of the ice. If the ambient temperature is T, and if the object does not change its temperature as it falls, then the change in entropy is the sum of the increase in entropy as a result of the temperature increase after it falls and the decrease in entropy as a result in its loss of potential energy as it falls; as = AT mgh T T States with higher energy have more microstates, and states with lower energy have fewer microstates; from this perspeotive, the state with lower energy (When the object is on the ground) is more ordered, as there are fewer possible microstates. No, the second law of thermodynamics is not violated, beeause only minute portions of the universe are currently highly ordered; overall, the amount of order has decreased In a eyele of the Camot engine, thermal energy {Quis transferred from the hot reservoir, thermal energy |Qc| is transferred to the cold reservoir, and work |IV/ is done, where [2e|= 1+ |] ‘This work done is then used to drive a heat engine that operates between the same two reservoirs, transferring thermal energy back from the cold reservoir to the hot reservoir. Ifthe heat pump is more efficient than the Camot engine, then the amount of thermal energy transferred back to the hot reservoir is greater than |x ; this means that with each eycle of the combined processes, thermal energy is transferred from the cold reservoir to the hot reservoir with no work being done; this violates the Clausius statement of the second law of thermodynamics, and so we can conclude that the efficiency of the heat pump cannot be greater than the cfficiency of the Carnot engine. Ifthe efficiency of the heat pump is equal to the efficiency of the Camot engine, then the combined process can be cycled indefinitely, retuming the hat and cold reservoirs to the same state at the end of each cycle, with no expenditure of work. But this contradicts the assumption that the heat engine is irreversible, so this is not possible. Copyright © 2014 by Nelson Eduestion Ltd. 49. ‘This leaves the third option as the only possibility: ‘The efficiency of the irreversible engine must be less than the efficiency of the Camot engine. Answers will vary. For a sea with a temperature around 22°C at the surface and 4°C at some depth below the surface, the maximum theoretical efficiency would be about 6%, Extracting energy would lower the water temperature, and could affect local sea life. Large-seale operations could alter occan currents, and affect coastal crosion, silt deposits, and possibly even Barth’s climate. The core of Earth has a temperature of about 5430°C, so the theoretical efficiency with a surface temperature of about 22°C is around 95% Large-scale operations could affect aquifers and local wildlife habitat. Copyright © 2014 by Nelson Eduestion Ltd. 205 Chapter 19—ELECTRIC CHARGES AND FORCES 1. © According to Table 19-1, negative charge will be transferred from glass to polyethylene, so polyethylene will end up with a negative charge and glass will end up with a positive charge when the tape is removed 3. ‘The answer is (b) plastic wrap and glass, since these materials appear the furthest away from cach other in the tribocleotric series Ag _ 50x10 C 5. ©) J 058 7. (b) You can argue this by quoting Newton’s third law of motion, or as follows: 9. (a) For the first situation with two +O charges, the magnitude of the forve is |_1_(-2)+9) J 4m, AaB, For the second situation involving the two charges +20 and — 20, the magnitude of the foree is Ane, 7 I ane, Therefore, the electric field points in the (—j) direction, 13. (b) The electric field due to the positive charge is fe zs, r ‘The electric field duc to the negative charge is 208 Copyright © 2014 by Nelson Edueation Ltd ‘The sum of the electric fields midway between the two charges is B48, 87), 2-1 Si) 476, °° dna, 2a, r* ‘Therefore the direction of the electric field midway between the two charges is /. 15. (b) The positive ends of the dielectric molecules are attracted toward the negative external charges (and the negative ends of the dielectric molecules are attracted to the positive external charges), 250x102) _(250x10")(1.60x 10" C} 17. fied {Bate (oosim (eet) i ) 40x10? c=40 90 At Ts Is 1 (re\(-e) 46, (53.0010 m 1 | (@oox10 c}{-1.0%10 ©) Br (885 10 kg SAORI 22x10 — 8.22x10 ———__ othe "5 (5) s22x108 Set 5 e| =8:22x10° N (b) m—=8.22x10* N r 22x10* ™ 33.0 pm)x 8.22 x10 N 91x10 kg 19 «10 m/s Copyright © 2014 by Nelson Edueation Ltd 207 21. 25 27. 208 ‘The positive charge must be placed (o the left of the 1.00 wC charge in order for the net force on the latter charge to be zero. Let x represent the position of the new charge. Then, 12.00 nC _ 1 4.00 wc Azz, (3.00 om) 2(3.00 em V2 (3.00 em) 4.24 om ‘Thus, the new charge must be 4.24 em left of the 1.00 uC charge P=gE F=2(1.60x10" C)(5.00 Nic) =1.60%10" N paad 1.75x3.34x10 C-m=ed 75%3.34x10 C 1.60% d=3.65x10" m = 0.0365 nm d= ‘The linear charge density of the charge on the rod is dg = suds we 75.0 0c 25.0 mm d= (3.00 pCim}ae The magnitude of the electric force exerted by the rod on the point charge is 1_Qdg B= | pee hoo 446)" ea PA 9 G00 pC'm)ar) ‘loo 4765 x (3.00 wC/m) 9%" ate tr he Copyright © 2014 by Nelson Edueation Ltd (1.00 2€)(3.00 snl Ars, (1.00 uC)(3.00 sci) 4n(8.85x10 F =0.0540 N (8.85 x10" }(6.67 x10 (5.98 x10" S.7Lx10" © By symmetry, the foree on each charge is directed away from the centre of the triangle. The magnitude of the force on each charge is cos(30") © 4(8.85x10-) (0.0325 m) F =0922N 35, ‘The two positive charges are equal and are equidistant from the point P, so the field from each of them balances to zero at P. Similarly, the two negative charges are equal and are also equidistant from the point P. so the field from cach of them also balances to zero at P. ‘Thus, the net clectric ficld at P duc to the four charges is zero. Copyright © 2014 by Nelson Edueation Ltd 209 (16710 )(40,0") 1.60 107 E=42NIC ‘The direction of the electrie field is in the positive x-direction. 39. Sketching a diagram and considering the directions and magnitudes of the fields will lead you to conclude that the only possible place at which the eleetric field is zero is on the x-axis to the right of the electron. Let x represent the position at which the electric ficld is zero, Then, 3(x-0.20x10) = 43 (x-0.20 x10) =x (5 -1)x-5 (0.20107) 3 (0.20x10°)} co) ATS jam x 41. The deflection of the drop in the y-direetion is ays da(asy and therefore. a 24 ary ‘The drop is deflected while it is in the deflection region, so the time interval is Ay v Ae and thus, the acceleration of the drop in the deflection region is 210 Copyright © 2014 by Nelson Edueation Ltd 45 By Newton’s second law, E mm and therefore the charge on the droplet is gE 2vAy m (Any 2 Aym (ay 2 Aypl E(ax) 2(25.0 mvs) (0.22510 m)(1.00 kg/L)(2.00-10" L) (95.0 KNIC}(0.0125 my" g=3.79x10" © ‘The torque on the dipole is F=(nR +H) E) = (ne + yeb)(-F) F=2reb(-£) £=2(25 nm\(Lo-d0- C)(30 KNIC)(~ 24x10" N-m)(-K) By symmetry, the component of the clectric ficld in the direction perpendicular to the z-axis, is zero. Let O represent the entire charge on the ring; then the amount of charge on an clement of the ring of length cs is w( where R is the radius of the ring. ‘The magnitude of the eleotrie field at the indicated field point is E (Ll S)st« ie Copyright © 2014 by Nelson Edueation Ltd 2 (0.10)(25.0 wo) [(0.04y' +(0.10)' ] B=(1.80«10" Ne) 47. The electrie field becomes weaker towards the right, so the point charge must be located to the left of x= 2 m. Therefore, the point charge must have a negative charge. because the electric field points towards the point charge. Q “ans, (xa)! Q=4n8,F, (xa) Q=4n6,E,(x—b) Dividing the two previous equations yields E,(x-a)’ ndfun, (exif -180(x— 4) = -80(x—9)" 9(x? -8x+16)=4(x? -18x+81) Ox? - 72x +144 = 4x? - 72x 4 324 Sx* = 180 xt 236 6 60m =4n8,8,(x-a)" 4a (8.85 «10? )(—180 kN/C)(—6 — 4)" 2.0 mC 212 Copyright © 2014 by Nelson Edueation Ltd Chapter 20—ELECTRIC POTENTIAL AND GAUSS’S LAW 1 @) 3. @) The potential energy is negative, but is greatest (i.c., closest to ero) when the two particles are farthest apart, which is when the negatively charged particle is at position D. 3 @ —2@--_42 are.) 470, 5 4Q 4as,\r| das, 7. (b) Regions of higher electric potential would repel and slow down an injected positive charge; on the other hand, regions of lower electric potential attract and speed up an injected positive charge. 9. (d) The electric field is the negative of the gradient of the potential function, and so is zero in the y-direction and z-direction, and has x-component equal to Gx. LL. @) The flux is positive because electric field lines are directed outwards from the charge, and the result is independent of the placement of the charge because the flux integral is independent of the placement of the charge 13. (a) ‘The net charge inside the surface is zero, so the net flux through the surface is also zero, by Gauss’s law 15. (a) See the discussion on Page 541 v7, u,= 2 4aeyr (1.60:<10)(-1.60«10") a(R x10" )(S50-* m 434x107" J 27.1 eV Copyright © 2014 by Nelson Edueation Ltd. 213 9. V O2KV (2 4x2,00%10" € aaer 4n(8.85x10-)(0.05¥2) 25. (a) ) 2 2a(0.0400 m) 25.07 NC ‘The cquipotential surfaces arc planes perpendicular to the raxis. AV = HAc AV _(78.0-25.0) V 5m, EO =20Vin Thus, = 75.0V at a position of x= 2.5 m 2.45 V9 45x1.6x10-" —* 9210" J/moleeule molecule molecule 3.9210 J/molecule 4.184 Veal 37x10" cal/moleoule 165 20 87 pC him Copyright © 2014 by Nelson Eduestion Ltd. 0.0344 Cim? 3.00 eC ar 37(00275 m) The flus through the surface of the sphere is equal to the flux through the semi-circular diameter of the sphere, which is O,=aRE where /is the magnitude of the electric field and F is the radius of the sphere. For r rp, @ Gaussian surface of radius r centred at the centre of the charge distribution cneloses all of the charge, so by Gauss's law Gre) E-4ar? =p (172), gare,” -(S) ‘The direction of the electric field is radially outward. Because the inner eylinder is conducting, all of its excess charge lies on its surface. Thus, by Gauss’s law, the clcetric ficld inside the inner conducting cylinder is zero. In the gap between the two cylinders, draw a Gaussian surface that is a cylinder of radius » and Iength Z coaxial with the other eylinders. By symmetry, the elcetrie ficld is constant on Copyright © 2014 by Nelson Eduestion Ltd. 213 the curved part of the eylinder, and the flux of the electrie field through the ends is zero, ‘Thus, by Gauss’s law, E-A=Qle, (US we) 5 pa (1HC)1 2ae, Jr 2.7010" E-2ark Similarly, outside the outer eylinder. draw a Gaussian surface that is a eylinder of radius r and length / coaxial with the other cylinders. By symmetry, the electric field is constant on the curved part of the cylinder, and the flux of the electric field through the ends is zero. Thus, by Gauss’s law, E-A=O!e, Bom = SHOE % SNL 2a, Jr 899x108 6.0% 10 5.0 10% 4.05 10% 3.0 10% E wim) 20% 10% Lox 105 = 0.0 x10" 0 005 6.0 os rom (1.60.10) aa(e85x10™)Q0IVF a) J 216 Copyright © 2014 by Nelson Eduestion Ltd. 41, ‘The charged particles are identical, so by conservation of momentum they will have the same kinetic energy. The initial potential energy of the system is py - hh Aer (25.0 uc} an(8a5x10" (501 U =160.568 J Thus, the kinetic energy of each particle is 47} (0.30) +(0.415) m 015m 01S m m 1 20x10% 4n(8.85x10")| 0.15 m ¥=120MV If Qy =-12,C, then the result is 1” = 556 kV 47. When the alpha particle reaches its closest approach to the gold nucleus, it stops momentarily, and its kinetic energy is zero, At this distance, 4d ‘ney Copyright © 2014 by Nelson Eduestion Ltd. 207 (2e)(79e) Fee (2 x1.673 10 +2 «1.67510 )(L.50 x10") rs 483x104 m 49, , Where r is the radius of the earth. 2 9 150(476, )(6.37x10°) =-6.77 «10° C 40,7 51. @) frm? 1.44 keV 53. ‘The spherical cavity is inrclevant to the calculation of field and potential in this problem. Because the sphere is conducting, the electric field inside the sphere is zero, Outside the sphere, the electric field is the same as that of a point charge at the centre of the sphere, according to Gauss’s law. Therefore, the electric field outside the sphere has magnitude tn (885x10")F 2160 and is directed radially outward. ‘The potential outside the sphere is therefore 2160 218 Copyright © 2014 by Nelson Eduestion Ltd. 55 57. and so the potential atthe surface of the sphere is yy — 2160 0.12 V=18.0kV Thns, the potential at all points within the sphere, including in the cavity, is the same constant V = 18.0 kV. For a cylindrically symmetrical charge distribution, one can use a coaxial cylindrical Gaussian surface of radius rand length £. to determine the electric field as follows E-2nk=Z = E-Qark. u 2aeyr where p is the linear charge density. Thus, the magnitude of the electric field between the inner wire and the first conduetor is (the ditcetion is radially outward) 15.0 wC/m 2n(8.85x10")r 270 kNIC Similarly, the magnitude of the electric field between the two conductors is (again, directed radially outward) (15.0-7.00) wC/m 2a(8.85x10")r 144 = ke Similarly, the magnitude of the electric field outside the largest conductor is 15.0-7.00-8.00) wm 2r(8.85x10")r =0 In cach case, the xcomponent of the electric field is the negative of the slope of the given graph. The magnitudes are listed as follows (without mention of sign): (@) 116 Vim wb) 0 Copyright © 2014 by Nelson Edueation Ltd. 219 59. 61 220 (©) 281 Vim @ 150V/m For a uniform sphere, the potential and cleotric field near the sphere are 800 V 15x10" m =533K . 50x10? C 4a(8.85x10 ")(250x10% m) V = 1.80 MV (b) When the drops coalesec, the volume of the single drop is three times the volume of cach small drop. so the radius of the new drop is 250310 m)= 36110 m ‘The new potential is 30 4a,R 3(50x10° c) 4a(8.85%10)(301 x10 m) 3.74 MV v v (©) Forthe smalll drops, Q _¥__180MV Azer 25010 m For the large drop, 3.74 MV =7.19x10" Vim =1.04x10" Vim Copyright © 2014 by Nelson Edueation Ltd. 63. Using Gauss’s law, and a Gaussian surface that is a eylinder of radius r and length Z coaxial with the wire, we can determine the electric field a perpendicular distance r from the wire, as follows: Eda. Q & ‘The direetion of the electrie field is in the radial direction ‘The potential a distance s from the wire is Similarly, the potential at s = 2a is v Fg, (82-2) Copyright © 2014 by Nelson Edueation Ltd. 21 65. 67. 2m Dividing the disks into thin concentric rings at radius r, we can make use of the result of Problem 64. If the total charge on the disk is O, then the charge on each ring is 2mrdr na 2rdr q=9 Dasa } Ie ma site [ Fae - #| oa rr] (@ 3MVin (b) Answers will vary (©) Answers will vary. (a) Answers will vary Copyright © 2014 by Nelson Edueation Ltd. Chapter 21—CAPACITANCE SS 9. UW By definition, the capacitance C is given by: _160x10%C 40 40K Correct answer: (b). The electric field intensity # can be calculated as: lov 10m =10°Vim od Correct answer: (4) ‘The separation between the cylindrical plates is small compared to their radii for A (but not B) and therefore the parallel plate approximation is approximately valid for A. Correct answer: (b) Let us say the capacitor Cis added to the existing capacitor C, . The new effective capacitances for a series connection C,,.. and a parallel conn ction Cy ae given by: It follows that the only way that the new capacitance is slightly smaller than C, is in series conneetion with a large added capacitance C which makes the quantity 1/C small Correct answer: (¢). The following equation can be written: 56.4 _ 264 dere EE The voltage across the capacitor -\ does not change beeause the capacitor is still connected to the battery. fos 1c Correct answer: (b). The following equation ean be written: Copyright © 2014 by Nelson Education Ltd. 15 17. 19. 21 Hence, Va, >E=48, Correet answer: (d). Improvement in rectifying the power supply output ean be achieved by inereasing the RC constant. Therefore, the addition of an extra capacitor in parallel will increase the effeotive capacitance C and therefore increase the effective RC constant of the rectifying cireuit Correct answer: (b). The following relationships can be written: 1 3 1 pepo et __ °~aRC 2 2m(ARVC Correct answer: (b). Q_6x10r°c Vv 120V 510° F = 0,500 mE (@ v= £a=2400%.42x107 m=4.80V m 1L7x10SF=1L.7 uF The following equation can be written: yA %A dod It follows that ar 88510 CON mm? x9 x 7.5" 10m? cS 115x107 F =1.36x10°m = 136mm, The following two equations can be written for the equivalent capacitances of 7 identical capacitors C’ connected in parallel (C,,) and in series (C, ) AC HC 4640 nt @ lai o-ftstit. 2) ’ (2 t'e @ Copyright © 2014 by Nelson Education Ltd. 27. ‘The following answers can be derived based on equations (1) and (2): 10°F x10" E =4 (parallel) a b) nek =5 ©) nse (series) © Ont (parallel) SxlOVF 25x10" 2 (series) ‘The circuit should have three parallel branches, where the first two branches each contain a single 500 pF capacitor, and the third branch contains two 500 pF connceted in serics . "The following equation can be written: 5A _ KbeA ad It follows that cd 16910" Fx 210m 691 8.8510? Nm? x 6.510? m? «8.5107 mi: x Based on the values from Table 21-1 the diclectric material might be mica. @¢, € a 2.5 yE 5c = 0.5% 25x10 x12? I= 180/00 (b) C, =2C =2x5% 10°F = 10a 0.5x10x10% x12°J= 7203 fav _ [2x125%1077 Veo V s0x10"F =10.7V The pemnittivity of free space #, is equal to: & = 8.85x10 CNM =8.85 x10 Copyright © 2014 by Nelson Education Ltd. 225 Further, § = 8.8510 ale : a L 2 ‘ 5 602x10"C C_ 2m x 8.8510" Bim 5 PE “sa) 35. 37. The capacitor with the two dielectric insertions is equivalent to two capacitors connected in parallel, each space between the plates filled with one of the dielectric materials, and with plate areas equal to half of the initial plate area. Hence, one can write that the equivalent capacitance C, is given by: earn) 41, (@)_ Two equations can be written: Fama a F ee 2 From equations (1) and (2), it follows that: 10910 "eG 7x10" ms x 1.602 x @ (b) 1 = Hd = 40 «10? Vim x4 10% m = 159Vim (c) Electrons move in the direction of the plate with higher electric potential; that is, towards the positively charged plate. 226 Copyright © 2014 by Nelson Education Ltd. 43, 45 47. (a) ‘The electrons were initially at rest, the following kinematics equation ean be written: 2.50 x10" u/s a 7x10" ms 3.57ns (©) The following kinematics equation can be used: . vt 2.508 x10! m [ee 2a 2x7 10 mis* 44.6 mm Hence, the distance travelled by electron s is larger than the separation between the plates of the eapacitor d. All capacitors in Figure 21-22 have the same charge Q on their plates. The following relationships can be written: 2 6, = 2k = 2 = Q =16 0 1 2B HC 3, = Cn oye ““ Tale @ c= 2 = 1508C > sone Vv 60V 8.8510"! Pim x 202104 m? oy ¢- S454 $0X8.85:10" Fim x2010 mt 95 d 25x10 °F oy P3610? (i) ya BARI ASOI oy @ bebe —2Y _e 2 vim @ 28.32x10%m ‘The energy density 1 in a region of space where the magnitude of the electric field is £ is given by wo Copyright © 2014 by Nelson Education Ltd. 207 Using Gauss’s law in the region just outside the sphere with charge Q gives: Q Q 2 Aner ® Combining equations (1) and (2), it follows that e (350x107) © DNA ai BDA ar 33m 8.9510 "Bim x(8.509107 Eda? 49. The following diagram and notations can be uscd for this problem 51. The energy stored in the cylindrical eapacitor (/ is given by: toy: 2 L ux volume =uar It follows that: 2x 2x10" Jem ¥ 25x10 4m? 1010? m. = 5.50mF 53. Answers may vary 28 Copyright © 2014 by Nelson Education Ltd. Chapter 22—ELECTRIC CURRENT AND FUNDAMENTALS OF DC CIRCUITS ST 1. (a) FALSE, Historically, the direction of the electric current is in the direction of motion of positive charges if the charge carriers were positive, (b) FALSE, The resistance depends also on the wire dimensions (R=, ot ) (© FALS temperature, Free electron density 7, is a characteristic of the metal independent of (@) TRUE. p=p,(I+e@AT), o=1/p (© TRUE, The following equation is obtained by combining equations (22-7) and (22-11): 2 pen, (b) The equivalent resistances for circuits II and Ill are R, =10R and 2, R/LO, respectively; R is the resistance of one light bulb. Given that P=1/* / R., it follows that the power output of circuit ITT is larger than the power output of circuit I which is larger than the power output of eirouit TL 5 © R= + RIN” RIN 7. @) BaP Re=Vir = 12V + 100A «3600s = 43210" J = 4320KF 9. (b) The equivalent resistance for a cireuit powered by a3 V battery in which the current is 1=45< is given by: av__2 R =< "745A 3 ‘This equivalent resistance is obtained by connecting all six of the 4 (2 resistors in parallel 4Q_2 {a =i0 6 3 Copyright © 2014 by Nelson Education Ltd. axa aoov 2 17. @) The following series of equivalent circuit diagrams show the step-by-step calculations of the equivalent resistance R, ® g Pw f u er gileer alle R R patlesra? = g2¥ atoms aS RAY son 15 19. @) 21. (a) ‘The current is greatest early in the experiment. 23. (a) —— .3x10°” electrons/s @ 16x10" C/etectron @) y-t- — 74 6,410" A mx4x10° mr 4 (©) Q=Mt=2Ax28=4C 230 Copyright © 2014 by Nelson Education Ltd. 25. 27. (a) Using data from Table 22-2: PSC) = p(20°C)(144.3x10°K AS K =1.687x10" O.m=1.7x10°O-m 20 K))Q-m => o(15°C) = p(1s°Cy" = 5.91072 mt I ar 4x2 Aenyesy, = Fea En 4 en, ®X4x10° m? x1.6%10 Cx8.4x10" m = 4,710 m/s Oe e Fp om _ 5.93 10° Q7* -m™ x9.11x10" kp 251x107 Sf ™ en, (1.6010 CY x80" (@) »,= = Matt ALOT is 2 ADOT RE 19.710" Vim <1 Vin et 16x10 Cx2.51x10" 8 (©) @) The answer will not change (b) The drift velocity v, will double its value (©) The answer will not change. (d) The electric field inside the wire £ will double its value. (f) No change in any of the quantities. (2) (@) The resistivity 7 will inerease and the conductivity o will decrease. (b) The answer will not change (©) The average time between collisions 7 will decrease (d) The electric field inside the wire 12 will increase ‘The following two equations oan be written: m=dy¥ =d,,Al=> Al= = Q @ In equations (1) and (2), symbols d.,, and p,, denote the mass density and resistivity 1.7210 Q.m. Also, d., =8.96 g/m’ values of copper. From Table 22-2, , Copyright © 2014 by Nelson Education Ltd. 231 29. 31 232 Plugging the wire length / obtained from equation (1) into equation (2) gives mPa, _ [LT8X1O7 kg x1.72X LOD -m _ 9 4 aon? Rdg, Y 3428.96 x10" kg-m> 8 gt? = 20m 17210" Q-m 10'W_y 5 120V Hence, only one toaster ean be operated safely. 40.45 x10 Q-mxS6 x10 m nx 04 x10 mm =202 For convenience we ean assume that the first resistor has the least resistance: R = R,,, Then, @) b) (e) ‘The energy saved per year is: E=Pt, boys = 1000 W x 2.x 60 s/day x 365 days/year = 441063 =12kW-h ‘The annual monetary savings is savings! year ~ 12.1 7kW -h x 6.5 cents/kW +h ~ 79 cents Effective ways to reduce your electricity bill are: replace incandescent Light bulbs with compact fluorescent lamp (CFL) bulbs, unplug the fridge and other appliances which run on stand-by mode (TVs, computers, toasters with elcetronie display. ete.) when caving for long periods of time, tum off the light when leaving the room, check the heating insulation of your house (particularly important for electric heating, but keep in mind that other forms of heating use electricity as well), Copyright © 2014 by Nelson Education Ltd. 37. (a) Connect the four 1.5 V batteries in parallel, atteries are connected in series which are then connected in parallel with (b) Two 15 V bi the other two 1.5 V batteries conneeted in series. Hi (©) Two 15 V batteries are connected in parallel and the combination is then connected in series with the remaining two 1.5 V batteries (@)_ All four 1.5 V batteries are connected in series o—i} 4} 4} i} 0 39. (a) Based on the circuit diagram from Figure 22-30 one can calculate the equivalent 2, as follows: e Rak ® LFF Copyright © 2014 by Nelson Education Ltd. 233 ‘The current trough the battery 1, is divided in two through the resistors B, C, D, 1. G. and I. Therefore, gala Tp Py Lora ° 2 42 ‘The currents through resistors EandF, 7, and 7, can be calculated using Kirchhoff"s laws in the loop with resistors E and F, yielding the next two equations Iptle=ly le 74 420230 105 12 24), 36 5 oan 21105) 105 ©) y= 3.43V @) P, -ir-(2) XSW =1.63W (4) R= (3) xsW=041W 2 x10W =0.82W. } x1OW =1.18W x) x15W=0.78W 234, Copyright © 2014 by Nelson Education Ltd. (©) Pay =P, +P, +P, + P+ Pp +P, +P, +P; +P, = 6.9W, which is cqual to the power value from question (a) using the equivalent resistance R., as expected. 41. ‘The following equivalent circuit diagrams oan be used to calculate the equivalent resistance R, 43. The figure below is based on the circuit diagram from Figure 22-33; upper left-hand side loops | and 2 are indicated along with currents J, J,. J,. 7,.and 1, and left-hand side nodes P and Q pst for ORY Kirchhoff's laws in loops 1 and 2 and node P indicated in the eireuit diagram above yield the following equations LR+LR=LR>1,-1,41, a alt, @ I,R+1,R=21,R=>1, @ Copyright © 2014 by Nelson Education Ltd. 235 Using equations (1), (2), and (3), the following relationships between the four currents can be derived: 1=3l, Hence, the total current 7 is given by: 1=2i,41,=61,+41,=101, Based on these relationships the equivalent circuit diagram is shown below. RoR we e fi Lok & h a | 4k WM 8 RoR z 5 4 ® R g ‘The equivalent resistance R, is then given by: R=|2 are +z) R°2R 45. @) q(t) 4qu.(1-2" Re : rf DY dun = CP = 300 x10 Fx12V =3.6mC ale) Ge =e In{1=0.99)=—zxIn(10")=2%3In10 A 1)= ayy: (1-2) =3.6mC x (1c) = 2.3mC H2t)= Fane (1 9(3€)= dua (1-e* =3.1mC 3.6mCx(1-e* 36mCx(1-e)=3.4me 236 Copyright © 2014 by Nelson Education Ltd. _ Rv © 5x10" ©) r)= he" xe =2.4mA xe" =0.88mA 1(3c)=2.4mAx 47. (a) The energy stored in the dry-cell battery is approximately equal to: = IVE = 0.67 AX1 SV x5 x 36008 =18090F IkW -h = 10? W «3600s = 3.6MJ If the energy from the dry cell were taken from the electrical outlet it would cost 180905 cents cost x10 = 0,0502Scents 3.6 x 106 . KW-h Given the cost of the D battery of approximately $1, the energy stored in the battery is almost 2000 times more expensive than energy purchased from the electrical energy grid suppliers (b) Using the example given in Figure 22-10, the average Canadian houschold electrical V-h=1683MI. The price of this energy monthly consumption is approximately energy using D batteries would be: ott taie = ASSN S1__ s03.000 18090 alter battery 49, Notations and directions for currents are indicated in the circuit diagram shown below. hob Re Applying Kirchhoff’s laws in the loops 1 and 2, and node B, which are indicated in the figure above, one obtains the following equations: Loop]: 2/,R ay Loop 2: 2/,R+3/,R=2LR @ Copyright © 2014 by Nelson Education Ltd 237 Dividing by & both sides of the equation (2). one gets that: ae) 4-2h =, 044 © Using equation (3), A aIA+04A=14A 6) Using Ohm’s law the electric potential differences between different points on the circuit can be easily calculated: Veg =2R=4V Ving =31,R=6V Voip *Vup +V ep +V ep = (S+0-6)V=-1V. 4 tV yp t¥ gg =(S4540)V = 10V =(446+0)V=10V 51. Current directions and notations in the circuit diagram (b) are indicated in the figure below O) AR AM N BLO (a) The current through the batteries in citeuit (@) Zyjoy yy 18 eal to 3V43V _2V 1 = 1 R a 238 Copyright © 2014 by Nelson Education Ltd 83. Applying Kirchhoff"s laws in the right-hand side loop of circuit (b) and node B, one obtains the following results: Loop’ LR=0V>/,=0A Q Pa Node B bth = i, @ Based on equation (3), the current through the batteries in circuit () Lesuyy py #8 equal to: 1, tenes) = 3430" _ 37" \2R R Using equations (1) and (4) the ratio of the currents through the batteries in the two cirouits is equal to A, 1 @ o tanorer oy 3 (b) The ratio of the electrical potential differences between points A and B corresponding to the two circuits is given by: ‘The ratio is less than 1 due to the higher current in circuit (b) (short-circuit of bulb 0). (c) The following equations ean be written for the power output (proportional to light brightness) of the bulbs in circuits (a) and (b). respectively: v2 BaPeba OS p, =P, =, p=0w *R ‘The brightness of bulbs M and N in cirouit (b) is 9/4=2.25 times greater than the brightness of bulbs M, N, and O in cireuit (a). The equilibrium temperature will be reached when the power dissipated by the resistor is equal (o the rate of the heat lost to the surroundings. The inerease in the temperature of the resistor is curbed by two processes: the increase in resistivity and the heat loss. Sinve the heat loss is the same for the two wires at the same temperature. it follows that the ratio of the powers dissipated by the two wires is approximately proportional to the ratio of the cquilibrium temperatures reached in the two wires. The following relationships can be derived for the two cases: parallel connection (a) and scries connection (b) Ry(Ty) _ Re (Ty) Re D+ Ge (Ty 1h) pIST, oT Re (lr) Role) Rt (, -1)| Copyright © 2014 by Nelson Education Ltd 239 )_ BoP) | Rr d+ Gy (ls -20) 8) HEA) ayafisfee,-2)] %, >Ty 55. The following useful notations are indivated in the eireuit diagram below. RoLUL R Ry Ry hth (a) Applying Kirchhoff’s laws in loops 1, 2, and 3, and nodes 1 and 2, the following equations are obtained: Loop 1 B=LR+LR, a Loop2: &, =1,R,+4R, @ Loop 3 AR + +4, (+R )=LR @) Nodel | L=h+h “ Node 2: Hlth 6) Using cquations (4) and (5) it follows that: «6 Lahtith 2 a(t th)R+ Loh ©) R+R, Plugging the current /, value from equation (6) into equation (3) one obtains that: Rx RR ‘Equations (3) and (7) have two unknowns: currents 7, and J,, which can be solved numerically to give UyRy ts +1 RA R)= [4-(+4)8] o t ~- 2 4-065 (8) 60 1 81 L, =s{i, +2) Sa Oh. 9 13 t2)= 05 O) Copyright © 2014 by Nelson Education Ltd 57. Using equation (6), one can find current [,° 15—(0.675-0.65)x10 14.75 2045 “25 =0.59A, (0) Using equations (5) and (4), respectively, one ean obtain the following values of J, and J, Ty 41g 1.2658 ay T=1,+1,=0615A (2) (b) Voltages across resistors can be easily calculated suing Ohm’s Law: R =0615Ax100=6.15V iy =-0.65 A x10Q=-6.5V Vg = iy = 0.59Ax15Q =8.85V Vy, = 1K, = 0.675 Ax 200= 13.5 V fn, = Uy #4, )R, = (0.65 + 0.675) x 200 =0.5V Ve, = (1,42, )R, = (0.65 + 0.675) x100 =0.25V (©) If the batteries are real, then small resistors will be connected in series with resistors R and R,. Hence, currents J, and I, will decrease, The following useful no! jons are indicated in the circuit diagram below. The following equations oan be obtained by applying Kirchhoff’s laws in loops 1, 2, 3, and. nodes 1 and 2: Loop l: ge AR +R + hy a Loop 2: 6, =-,R,-L,R,+ LR Q Loop}: 6-8, 51,2, @) Copyright © 2014 by Nelson Education Ltd 2al Node 1: J, Node 2: I, y+, ©) From equation (3) it follows that (12-10)V =0.1A 6 202 @ Combining equations (1), (4), and (5), one obtains the following cquation: HAUL MR ARE LR, M Combining equations (2) and (5), one obtains the following equation: V,), it follows that there is a surplus of positive charges on the back face of the rectangular piece of semiconductor. Using the historical convention that electric current direction is established by the bulk motion of positive charge carriers, it follows that current flows from left to right in the rectangular piece of semiconductor from the circuit diagram of Figure 23-48, Using the right-hand rule, the direction of the magnetic force 7’, is from the front to the back of the reotangular semiconductor as shown in the figure below. Therefore, if the charge carriers are positive, an excess of positive charges will accumulate on the back face of the semiconductor, consistent with the measured potential difference: V7, > /,. Ifthe charge carriers are negative (i.e. clectrons) the direction of the current will be reversed. the magnetic force direction F, will be the same, however, the potential inequality will change: V, B, Correct answer: (c). The radius of the electron’s curved trajectory r is inverse proportional to the magnetie ficld B: roll It follows that if the magnetic field is deercased by a factor of 2, the radius of the electron is, increased by a factor of 2: However, the deflection decreases, but not by a factor of 2 since the deflection is produced only by a fraction of the clectron’s curved trajectory, or, in other words, the radius and deflection are not directly proportional. For each diagram from Figure 23-57, the magnitude of the magnetie field [3] ean be calculated as follows (b) B-28, Copyright © 2014 by Nelson Education Ltd 247 In the results written above, , = //274,7, where r is the distance from the centre of the square to any of the wires positioned in the comers of the square. Therefore, the ranking, from the lowest to the highest, of the magnitude of the magnetic ficld |B] is as follows: (e)>(4)=(4)>(a)=(e) Vy) 19. The force on the electron F* is given by: Pa(-)FxB @ ij aA | ¥xB)= 40 8\x10' =(4.87 4.6.47 -2.41 )xtot mys 2 j08 06 0) Using the results of equations (1) and (2), F 1.6 x10" x 8x10" x(-0.67 +0.8) ~0.38) i 2.8%(-0 Gi + 0.8} -0.38 jf Also, 21. The magnetic force F acting on the particle is in the direction of j and its magnitude =12.8x V0.6 +0.8 +0.3 IN =13.41N given by [F[= 9x8 10.10" x30" 3107 x1 52 45m y (a) The radius of the circular motion associated with a magnetic field 3 acting on a particle of charge ¢ can be determined by recognizing that the magnetie foree Fis the centripetal force F. : a my my Bake ype ae ’ eae e 248, Copyright © 2014 by Nelson Education Ltd ‘The mass-to-charge ratio m/q ranking of the cleotron, proton, and @-particle, respectively, is as follows >, /e], >T, 25. (a) ‘The proton’s speed v, can be calculated using the following equation: Br _1.60x10 ENO STAG a 8.6 x10" mis = * 1.67%10" ke 6 kms (b) ‘The proton’s angular frequency , is eB _1.60%10"°C x0.9T > =8.6x10" rad’s rom 16710" ke 27. The radius r of the trajectory of a particle of charge q in a uniform magnetic field B is given by my qb , ay ‘The speed of a particle accelerated by an electric potential difference {” ean be calculated classically as follows: Q Substituting the speed v given by equation (2) into equation (1), the following equation is obtained, qv m 1 amv _ ov & i qgB q BY q 8B Copyright © 2014 by Nelson Education Ltd 249 Based on equation (3) the following ratios amongst the three radii of the deuteron, @i-particle, and proton r,. %,, and r, . respectively, can be written: 250 Let ¥. ¥%, ¥ denote the speeds of the electron at the surface of the neutron star equal to 5%, 10%, and 20% of the speed of light in vacuum, respectively. The corresponding electron radii 7, 1, and ¥, can be calculated as follows: m 9.LL LO kg 0.05 x3 x10" m/s . . = 0.2810" m =2.8x10"m 1.60 x10 x3 x10" vy 9.ALx 10 kg x 0.13% 108 m/s 1.60x10 °C x3x10T 11x10 kg x 10° 1.60 x10 PCr x3 x10) 5.710 m =114x10" m ‘The corresponding magnetic forces F,,, F,,, and F, are equal to: ey, B= 1.60 x10 C 0.05 x3 «10% mis x3x107T = 72 uN, y, B= 1.6010" Cx 0.1 «310% m/s x 3107 =144uN vB = 1.6010 "Cx 0.2%3x10%m/s x 3x10" T = 288 uN ‘The magnetic field B, perpendicular to the electron’s velocity By =(0M.20 4 180) = |B |=B, = V20° +18 wh = 26.947 a) ‘The radius of the electron’s trajectory R is given by: amy 9.107" kg x6 x10" mvs BL 1.60410 CxV20° +18" x10 The geometry of the clectron’s trajectory is depicted in the figure shown on the next page The radius of curvature in the y= plane and the clectron’s deflection are denoted by R and A, respectively. R 12.696m =12.7m, @) Copyright © 2014 by Nelson Education Ltd Applying Pythagoras’s theorem in the right triangle OCB OC? +CB* = BO?=(R- AY +d? =k? @) From equation (3), the deflection Ais given by: A= RNR —@ =12.696— Vi2.696" —0.5" =9.8 mm (a) The magnetic force per unit length /°/7 is given by Feat 8Tx10A=18Nim (b) ‘The magnetic force per unit length /")/ after the magnetic field is halved and the current is doubled, is: Fog 8 1 2 2 = BI = © -18Nim The magnetic field at the centre O of the circular loop 3 is equal to: Braper-nag + woot = Bagge say + (Big The magnetie field created by the lightning bolt can be approximated by the formula’ patil 2ar ‘The magnitudes of the magnetic field at 5, =10 m, 75 HIO'A _ 990 yt tht _Axax107 mi; 2xax1Om Copyright © 2014 by Nelson Education Ltd 251 39, 252 Axaxd0TmA * x10'A eae = 20nT 2xax100m B(5)- A= 2a 2ar, By comparison, the Karth’s magnetic field is roughly 50 421" (a) ‘The magnetic ficld magnitude B (1°) produced by a wire carrying current J with, diameter d at adistance r from the centre of the wire is given by: B(r) = Poless fom _ tat or Qa Qar aid md a a Based on equation (1), 2x 4a x107 ‘25 2mm ax10x10m 10mm B(2mm) =0AmT (Sm) = 22 AKZO TAT x 25A_, S mM gay mx10%10° m 0mm (b) The magnitude of the magnetic field B(r) at a radial distance 7 within the cable rSmm ‘The graph of the above function B= B(r) is shown below. © 5 10 15 20 28 30 35 40 45 sn ss ean) Copyright © 2014 by Nelson Education Ltd. 4 (©) Applying Gauss’s law for a uniformly charged insulating sphere of radius R and charge Q, the electric field E(r') inside and outside the sphere is given by 2 za. réR Ame OR . Rr=\ 6 ) r>R 48,7 Applying Ampere’s law for a long wire with radius R and carrying current I, the magnetic field B(r) inside and outside the wire is given by AL” pcr agra | 2k a Ht rR 2ar Relationships (3) and (4) are similar, except for the more rapid (~1/r*) deerease of the electric ficld E(r) outside the uniformly charged sphere. Let a= 20cm and 4 = 30cm denote the dimensions of the rectangular wire. The force acting, on each side of the rectangular wire pulls in the outward direction. ‘The forces pulling on the parallel and equal sides have equal magnitudes, but opposite directions. Hence, only two force magnitudes are to be calculated: Hand F,. They are given by: °b [mA * 300m wl'b | 4x x10 TMA x10"A? X30 _ 59 py 2na 2% 7% 200m = xB LOT TMA x1OEA? 1300 _ 40 yg yn 2ab 2x mx 2em 3 (a) The magnetic moment of the loop 4i,,, of radius / and carrying current I is: 3144-0 (-#) Peg Jak’ (-£)=210.4%(0.1m) «(-A) (b) The magnetic moment of the coil ji,,, is given by = Niji Bis =100x(7x0.1)A-m* (—£)=31.4.A-m? (€) (©) The torque experienced by the coil Z.,, is given by 2 Copyright © 2014 by Nelson Education Ltd. 253 254 314 A-m? & @ ©) ih, (b) fa =314Armek ©) #,, =314N-m(-7) (© If B=1.077, the results of questions (a)-(e) change as follows: (a) The original result is not changed. (b) The original result is not changed. (©) The torque experienced by the coil 7.,. is given by: Hiy »B “Hg Bi =31.4N-m(-i) If B=LOM'&, the results of questions (a)-(e) change as follows: (@) The original result is not changed. (b) The original result is not changed (©) The torque experienced by the coil 7... is given by: ij xBelo 0 loo (®) By definition, the potential energy of the coil is given by: usp Based on the equation written above, the initial and final potential energy values U7, and U,,, respeotively, are given by: U, = FthoiB A\\B (vectors 4 and B can be aligned parallel ot antiparallel), u,=0, BiB ‘Therefore, the change in the potential energy of the coil AC/ is given by AU Copyright © 2014 by Nelson Education Led. 45. (@) b) © ‘The direction of the magnetic force F., on the electron is perpendicular and away from the wire, as indicated in the diagram on the left in the figure shown below. Its magnitude /, can be calculated as follows: Fy a ev ey Hal = LSX10" C1. 5x10" mis 49 x10” TA 5.0 2ad 2xmx2x10! m =1.2x10 = 0.121N ‘The direction of the magnetic force F, on the proton is perpendicular and towards the wire, as indicated in the right-hand side diagram from the figure shown below. Its magnitude 7, can bo calculated as follows Mol _ 16X10 Cx1. S10" mis x4 7 107 TAT XS.0 evB =ev. ad 2xmx2x107m ‘The radius r of the circular trajectory induced by the magnetic force acting on a particle with mass-to-charge ratio m/q and moving with speed v perpendicular to a uniform magnetic field of magnitude 2, is given by: my r = 1 es @ ‘The magnetic ficld B(d) orcated by along wire carrying current J at distance d is given by: al B(d)=—— 2) ( 2ad ® Combining equations (1) and (2). the following equation is obtained: 2ady = e) q Hl ‘Equation (3) indicates that the radius r of the charged particle trajectory for which ¥ LB, is directly proportional to the distance from the wire d, Qualitatively, the trajectories for electron and proton are shown in the figure on the next page. Copyright © 2014 by Nelson Education Ltd. 255 wo o 47, Let 2, and R,, denote the resistance of the upper and lower ares, respectively. Also, let J, and /, denote the currents carried by the upper and lower arcs, respectively. ‘The following equations can be written by applying Kirchhoff's and Ohm’s laws: Teh+h, a i _®; 4 (2) ae @ Using equations (1) and (2), it ean be derived that: 6) “ ‘The magnetic ficld magnitudes created by the two semivireles are given by 0) pat Mh! gal oy 2 22 OR 1h’ 5, - At ©) 2R, DR, Let & denote the unit vector perpendicular on the plane of the two ares. ‘Then, the total magnetic field B ereated by the two ares is equal to” 49. (a) Assuming a constant magnetic field throughout the length of the solenoid and applying Ampére’s law, the magnitude of the magnetic field in the centre of the solenoid B, can ‘he determined as follows NL _ Ax x10 TA x50x1A _ B =5ax1 L 40x10? m = NI 256 Copyright © 2014 by Nelson Education Led. (©) Let us assume an infinitesimal element of length de along the solenoid. The infinitesimal number of windings dV corresponding to this infinitesimal element is equal to aN = Nav! L ay For convenience, let the x-axis be the central axis of the solenoid and assume that the centre of the solenoid is at x~ 0. The infinitesimal magnetic field d(x) ereated at distance x by the infinitesimal solenoidal element of length dx and radius R can be calculated by employing the Biot-Savart law (see Example 23-8): HIRE 9 yy a MIR __ ot 2(¥ +2) 2 (a eit) B(x Let B(x) denote the magnetic field along the central axis of the solenoid as a function of distance x from its centre. The function B( can be determined by integrating the value of dB(x) from equation (2) as follows: G) The limits of integration in equation (3) are: ,=—(L/2+x) and 1, =L/2—x. The following mathematical identity can be demonstrated by using the trigonometric substitution x= Rtan(0): a a (very? R veer +c @ Where C denotes an arbitrary constant in equation (4). Hence, the magnetic field B(x) from cquation (3) is equal to: NIR® Nl —2x a(x) wR MND ae L 2 6) Ey AL | ur2ry e4R? Y(L-2x) +4R* It can be noticed that |x| = 0.29m BBB, 16x10? CxS1.3x10°T (©) The gravitational forve can be neglected due to the very small mass of the electron. (a) Let 8, ‘soe; = 557° and I... denote the Earth magnetic field and the maximum current intensity to be calculated, respectively. Then, J... can be calculated as follows Halos 7 _ 27Bayy _ 240 x30 10" Qnd mm Axa x10™ Copyright © 2014 by Nelson Education Ltd. 825A O) Mag = =41A 63. Letus assume that the magnetic field of the Barth B,,,,, is entirely horizontal, pointing towards North, and with a magnitude B,,,., = 55d’. ‘The net magnetic field B,,, will then be horizontal and pointing towards North: OSSmT7 Buy = (B+ Bases (7) = (Sx10" +55x10°)(-7) ‘The distance from the wire 7 at which the total magnetic field is zero can be calculated by equating the net magnetic field magnitude B,, with the magnetic field magnitude ereated by the wire at this distance B, (7°) I 1 _ 4x mxt0? Tmax S0A B, (7) Bg = re AL 0 XXL TMA SOA aay 2ar 2nR, 2x A¥S.OSS¥10°T Therefore, the total magnetic field is zero at a distance 2 mm below the wire as indicated by the magnetic field veetor diagram shown in the figure below. N w E Bye 65. In the limit that the length of the solenoid is significantly larger than its diameter, the magnitude of the magnetic field B inside the solenoid can be determined employing Ampére’s law: dynl = 4x 0x10 TmA™ x10" x10A =4rmT =13mT Copyright © 2014 by Nelson Education Led. 263 ‘The magnetic permeability of iron 11... is given by: Hoon =H (Zroo +N) Henee, the magnetio field inside the solenoid with an iron core B,,, is given by: Boy = B( Yoon FI =4 x 2 x10 T x (410° +1) = 50T 67. The relative percentage increase or decrease (AB! B.,,)*100 of the magnetic ficld ‘magnitude for the three substances compared to the magnetic field magnitude in air B,, can be calculated as follows: B In the cquation above, the magnetic ficld magnitude in vacuum is denoted by, and the approximation K,, =1 is made For the three substances at a temperature of 2017, the following results are obtained: (a) 4100 = (Kyou, 1)%100 = (0.999991 -1) «100 = -0,0009% (&) > x100=(£,,..., -1) £100 = (1.000019 -1}x100 = 0.00019% ©) x100= 0% OF 69. The Biot-Savart law is: Q@ In the polar coordinates (R,) characterized by perpendicular unit vectors (78). the cross product in equation (1) can be written as: 2 db @ In equation (2), fi denotes the unit vector perpendicular to the unit v« Idi x7 = Idipx RF =iRdlii= indicated in the vector diagram from the figure shown below. Com (2), the infinitesimal magnetic field a from the Biot-Savart law is: 264 Copyright © 2014 by Nelson Education Ltd. 7 ‘The total magnetic field at the point C, B(C), can be obtained by integrating over the angle ¢ the infinitesimal magnetic field magnitude dB resulting from equation (3): aya [tel gg a Hl B(C)=| 22 dp= 3 ("eR ter (@ Using the notations from the vector diagram from the figure shown below, applying the Biot-Savart law, it follows that: a\ dx || = 94 Jae . equation (6) simplifies to: = B(R)= o 2aR The simplified expression from equation (7) is the known formula of the magnetic field magnitude ercated at distance R by awire carrying current Based on the solution of part (a), the magnitude of the magnetic field at point P, BP, is given by: a(R) Pecos 2, 7 8) Tis easy to verify that B(R)=28(P) if the length L. is replaced by L/2 in equation (8) as expected. That is. the magnetic field at point Fis the algebraic sum of the magnetic fields generated by the two halves of the wire. ‘The magnetic field generated inside the toroid 8, can be calculated by applying Ampére’s law using a cireular loop of radius R MNT 2ARB, = yNI=B, 2aR a ‘The magnetic field outside the toroid #., is zero, since the enclosed current J... is zero. Copyright © 2014 by Nelson Education Ltd. 75, (b) The magnitudes of the magnetic fields inside and outside, B,, and B,,, , for the solenoid with A’ windings and length Z are given by: @Q 6) ‘The magnetic field magnitude of a toroid of radius 2 is equal to that of a solenoid bent into a circle. Then, z aah @ Substituting the length Z. from equation (4) into equation (2), the result rom ‘equation (1) is obtained. (c) Based on the comparison and analysis from question (¢), it can be inferred that a toroid is a solenoid bent into a cireular shape. 2 taNT _ 4x a x10" Tm. 0" x15A @ B, Seat Tink OSA om ak 2e nx 05m @ p, = fONL Axamto TmA "x10" x15A gor L im B, ile) 8, (4) ‘The ratio B, (¢)/B,, (d) written above is equal to the toroid-solenoid length ratio. This result is expected since the length of the solenoid is equal to the diameter of the toroid. It is advisable for people who have pacemakers to keep their electronic devices (such as iPods) as far away form their pacemakers as possible, Copyright © 2014 by Nelson Education Led. 267 Chapter 24—ELECTROMAGNETIC INDUCTION 1 3 u 13, 15. 268, ‘The correct answer: (lh). ‘The correct answer: (¢). ‘The correct answer: (a), ‘The induced voltage € in the case described in Figure 24-41 in which | =1m denotes the length of the metal bar and 3 =1T is the magnitude of the uniform and constant horizontal magnetic ficld, is given by: The correct answer: (d). ‘The following relationships are valid Ny Y, 25000, Mh 1000 It follows that 120000 10001000 1, «1000 =10 107A x10" =10.4 br =120V ‘The correct answer: (4). ‘The magnetic flux through the loop is decreasing, therefore, the induced current is such that magnetic field in the loop is in the same dircetion as the one generated by the current- carrying wire. Hence, the current runs clockwise and there is an attractive force between the loop and the wire. ‘The correct answer: (b) ‘The magnetic flux through the ring increases until it reaches about half of the permanent ‘magnet length; therefore, the induced current is such the magnetic field generated in the ring is opposite to the magnetic field of the permanent magnet, Hence, the current runs clockwise during this time. The direction of the current changes afterwards. ‘The correct answer: (a), The inductance of the solenoid Z, is given by the following expression: Copyright © 2014 by Nelson Education Ltd. 17. Based on the previous equation, the following solenoid inductance values ean be calculated. mu(Nx2R) mpd NR) OE cg A OT. L 7 2N x2R) RY OD Basco a RHGNGORY (NEY T TL mu(2N%xRY_ mu(NRyt Gong gs L T © Eaanag = HOR *RY BANE) 2. 7 xy (xR) © Big RT 1 AR m2 OU Henve, the ranking of the inductance from the largest to the smallest is as follows: (4)>(@)=(e)>(4)>(2) ‘The energy density 1, stored in a solenoid with NV turns, of length L., radius R , and through which runs a current J is given by 2 Leen x, ONT 2e HN? I? 2 _#QNY Ppt 2(21)° ar @) u, Ol tee a oe oi 22ny 4 ‘The ranking of the eneray density 1, is as follows ()=(¢)>(a)=(4)>(2) Copyright © 2014 by Nelson Education Led. 269 19. ‘The current as a function of time J(/) in an RL circuit is given by i Tele =0.28>r=—m(4) (tafe if Bina) ‘Therefore, the ranking of the time follows the ranking of the L/R time constant. Cirouit 1; 2 = 20#107H ng R 10k Binnie 2 20RIUTEE 4 R 20Q Cireuit 3: 4 = 20810 ing R 202 3 ce Cirouit 4; © = 2 9.6mm R 502 Cireuit 5: = 2010 6s R 5302 ‘The ranking of the time 1, from the largest to the smallest, is: ()=(2)=(3)>(4)=(3) 21, When the iron bar is inserted into the air-filled inductor, its inductance L inoreases due to an increase in the magnetic permeability se. Hence, the energy stored in the coil increases, which, in turn, reduces the available power on the bulb. ‘The correct answer: (c). 23. (a) The magnetic flux as a function of time #, ©, (¢), is given by: Bt) A ®, (1) 8 (0-5 -B(t)w(1-vt)=B(e)w(vt-2) wM The rate of change of the magnetic flux d@, / ct is then given by: a(t) Blt) =“) ab SEE awn (net > ° vB(t)4 (vr 2. = [ (@+r-9 The time ¢' during which 3/4 of the length / is out of the magnetic field is equal to: BL _ 3x0.Sm =0.258 4y 4x1 Sm/s Numerically, equation (2) gives 0.25-[1.5-(0.002-0.25 +-0.001}+ (1.5-0.25—0.5):0.002-0.25]Wb dt =0.55mWb 270 Copyright © 2014 by Nelson Education Ltd, 25. (b) The time ¢" during which the entire length / is out of the magnetic field is equal to: 0.5m vy 1Sm/s 3 ‘Numerically, equation (2) gives’ Ws os [is (ov $+0.00 (1s dt 3 a =2=11mwb (©) @) Radin ) Se = 13m Wb ‘The rate of change of the magnetic flux through the inner (denoted by index 1) and outer (denoted by index 2) solenoids is the same. Therefore, it can be written that: 107TmA*-100 4 A 0.5m . ‘The induced emf # is: > (810% at 0.18 <107)Wb 6x 107)W ev ‘The magnitude of the induced emf & is given by |_A8(0-2)| _ NsR Mt At Therefore, the magnitude of the magnetic field B is equal to: ¢_ 45x10°V x 0.028 NS 206x107 m* | at =75mT According to Faraday’s law of clectromagnetic induction the induced emf 2 in a coil with N loops and resistance R is given by: Copyright © 2014 by Nelson Education Ltd, am 33. (@) Employing Lenz’s law, the magnetic field generated by the induced current J flowing 2m through the rectangular loop has to be in the same direction (upwards) as the applied decreasing magnetic field. Then, the induced current /- runs counterclockwise, as seen. from above. Hence, the magnetic force in the diagram shown below. is in the direction of the «v-axis as indicated 150m ‘The magnitude of the magnetic force as a function of time F(t} can be determined as follows: BE 468) Fi, (t)= B(QLI(1)= B(6). R z w Further, a(Bs ab as s(jp Seay 2 - ORO @ Attime ¢ in which the bar moved to the right a distance ~(), the rectangular loop area S(1) is given by S()=£[x, +2(4)] e) In equation (3), x, = 150em.. Using equation (3), one can write that: d_ i ak 4 at dt @ ‘The magnitude of the magnetic field dependence on time B(t) is given by B(t)=B, -Ct 6) In equation (5), B, = B(¢=0)= 15m, and C =0.3mT/s Hence, O) Copyright © 2014 by Nelson Education Ltd, Substituting the results of equations (3)-(6) into equation (2), (Bs * ) LC [xy +x(1)]+£(8 -Cr)v(0) Mm Equation (1) can be re-written as: Bact)e R {(2-Coyr(t)-C [ag +2()]} Bo COE tee) c| + x(t}4 w(1)]} @) Attime t= 0s: x(0)=07m and v(0)=Onvs. Therefore, equation (8) gives 15 10°F «0.3107 10,54 m? och. Sm 5 22 (b) ‘The acceleration at time = 0s, a(0) is equal to: x10 —x0.3?m’? ¥1.5m (0) | ACE 8 m mR 22% 0.25 kg 15x10°T x0. a(0) 38. @) The magnitude of the induced emf « in the coil is given by SF [scosantea,)] =2nB5|sin(27 78 +2, | Hence, the maximum induced emf ¢,,. is 2n (BS =2x mx OOH x3 x10? Tx 100 x10“ m? = 1OmV Face (b) When .V =100 loops are used: = 27 fBS =100 x 2% a x 6OEz x3 x10 100x104 my Sas (e) ‘The time dependence of the induced emf, 2(¢), such that (0) £(0) = ey, €08(2m fi) 37. (a) The number of turns in the secondary coil NV, can be determined by assuming that the magnetic flux variation dD, / dé trough a single loop of either of the two coils is the same. Hence, Copyright © 2014 by Nelson Education Ltd, 23 @ @) From equations (1) and (2). it follows that: a ee =1000x 5 =100 (b) If the transformer is 100% efficient, conservation of energy leads to: Ke ) VY _12V R 13a ‘Using equation (3), = 0.8A=800mA 0.08A =80mA. ‘The ms values of the two currents are: 0.084 Inst == STA v2 2 1, _O8A Tog =e = 8A 570m A v2 V2 (e) The primary current has the same value as calculated in question (b) 1,=80mA 1 L If the transformer works with 90% efficiency, equation (3) ean be re-written as follows: O91N, = LY, Hence, Sekt. % 9 x0.08A x120V =720mA 274 Copyright © 2014 by Nelson Education Ltd, 39, (a) ‘The magnitude of the electric field induced outside the solenoid E(r,), r> R can be derived using the following equation: QarE (ret a) Further, the induced emf ¢ can be determined using Faraday’s law of electromagnetic induction: a, at a(t) :) vere 8 Nar a e ot dt\ L L ‘From equations (1) and (2), 1 MyN7R? fing, CO8( 27 ft) E( r,t) = = 3) (= L 2) (b) The magnitude of the electric field induved inside the solenoid £(r,1), r- F(t) E, =1080N n= P= F(t) oe id Hi iz 1 © a0-A\ 30° t SAR hi 1 agibe 1 La aet ; J X80 =F vat fie iN OF i] \; ‘a= Wem aK maa? BK 51. The induced emf in the band ¢ is: ay { (bar) aarp Ht =-2nBC(r +t) di dt dt Copyright © 2014 by Nelson Education Ltd, an In the previous equation, 8 = 50.0mT and C = 0.025em/s. Hence, = 2x x 0.05T x 0.025 x107 = x(5+0.025 x¢[s])x10?m 9 (140.05 «¢[s]) ev 3. (a) Newton’s second law on the downward axis in the inclined plane gives: ma = mg sin(p)=F, e0s(9) => a= gsin(y)~“Eeos(9) @ (b) The magnitude of the induced emf in the red #(v) is | e(v)= [Floste = BLvcos(g) Q (©) The magnitude of the magnetic force /', is given by 5 a Bt F,=Bi=BLL = F, Ebvo) 6) R R Using equation (3), equation (1) can be reawritten as: dy [BLoos(#)] = gsin(g)—"—— a To solve differential equation (4), the following notations can be employed: mR ____O.3kgx20___ go, 6 [Beeos(@}]} [0.57 0.2m xv /2] . y s= gsin(¢)-— 6 vis gsin(g)— 7 © With these notations from equations (5) and (6), equation (4) can be reewritten as o It follows that the value of quantity x at time ¢, 2(¢), is given by x(t) = x(0) ®) Attime t= 0s , »= Om/s. Henee, ~(0)=gsin(9) © 278 Copyright © 2014 by Nelson Education Ltd, Going bavk to the original notation from equation (6), gsin(¢), anata rent I-s ‘| (0) T a 48.) gn «| =302x/1-¢ & | al) 8 (d) Using equations (2) and (10), the expression of ¢(1) is Numerically, va98 x¢0d{1- 785 s(t)= BL (estate Numerically, (1) =0.5T x0.2m x98. (© From equation (11), terminal velocity y, is equal to: y, = 302m/s 55. (a) Using Ampére’s law. _ BNL _ 1.210 Tim x10" x10A, 2ar 2xA7x03m =64T 2arB = wNI=> B (b) ‘The magnetic flux through the toroid ®, is given by nit 2ar ®, = BSN. I ‘Therefore, the inductance of the toroid, L., is equal to: BN? 1.20 "Hm! x10" x10 x15%10%m? 2nr 2xax03m L =96H fo) Be BNE L2x10*Tim x10" 20A sop ar 2xax03m WINES 1.210 Hm™ x10® x10 x15 10% m? 2ar 2xax03m =96H 57. (a) The current 7 the car engine draws when running at is operating speed is: 12V'-10V 0.42 =5.0A Copyright © 2014 by Nelson Education Ltd, 279 (b) During the start-up there is no induced back emf’ &. Therefore, the current /* during this time is equal to v_pyv a 30A, R 049 59, (a) The sketch of the oscilloscope trace that can be observed is in the plot (a) from the figure below. Two pulses corresponding to the magnetic fluxpassing through the two coils will be formed (b) If the speed is doubled the time between pulses will be half as long and the amplitude will be twice as large, as sketched in the plot (b) in the figure shown below. (©) If the number of loops in the left coil were to double, the amplitude of the first pulse will double as sketched in plot (c) from the figure shown below. wo (A) ‘The measured speed v is equal to d 2m = 570m t 35x10" (©) For higher speeds the two pulses merge; a high speed can reduce the time separation between pulses below the width of the pulse (Le. the two pulses cannot be resolved), Increasing the distance poses two problems: (i) the projectile trajectory is affected by gravitation over longer distance, hence, the projectile might not pass through the centre of the second coil and lead to an inaccurate measurement, (ii) the speed of the projectile is affected by air friction over long distances leading, again to inaccurate speed measurements. 280 Copyright © 2014 by Nelson Education Ltd, 61. Let 4 and «, denote the ems induced in the two loops of radii 7; = dem and 7, = 2em, respectively. Then, @B 4 £ 1 i a aB = 2 at ® The ratio of the loop resistances R, /R, is: =2x052=19 RR ‘The individual loop currents run in opposite directions, therefore, the net current in the wire J is: ax (16=4) 10m? x0.5x10? > 1 es R+R (1+0.5)Q ‘The net current flows counterclockwise in the large loop of radius 7, and clockwise in the small loop of radius 7, 63. (@) The current time dependence in the series RL. circuit, 7(t), is given by: i(0) a Then, Vq(t)=1(t)R=12x(1-2* @ V,(t)=ne F =120e%1V @) Based on equations (1) (3), the values in the Table 24-3 are as given below. Time a fl E a “) C ” vo voor 008138 oot ot 4m 728 0sts 2 02 759 4a 0082 o4 1038182 wee Copyright © 2014 by Nelson Education Ltd. 281 (b) The following relationships can be used to check the values from the table above at any given time 1 V,()+V,()=s=12V w& VAY MO-—R (6) (c) If the battery is not ideal, the resistance of the battery r has to be added to the circuit resistance R needed to move the bar to the left is 65. (a) ‘The external foree # 3x10°T) x(0.6m)' xL.Sm’s b) Ey = = 0.972 yN =1 uN © Fer Q 5a HN = 14 (c) The energy delivered to the resistor £, in time ¢ is given by: BLvt R x10 1) «(0.6m)' x(1.5m/s) x38 i) =4yt (d) 52 B 67. (a) The time to reach 99% of the maximum current /.,,. can be calculated using the known time dependence of the current, I(r): \=« sal(-2) |-segeac=sm R Tess 52 ‘The value of this current J is 1= 0.99% 1,., 0.99 ZN =2.4a 5Q (b) The voltage across the inductor Vis 2V x(1=-0.99)=0.12 282 Copyright © 2014 by Nelson Education Ltd, 69. 1. (©) The time dependence of the current /(F) when the switeh is in the position b is an exponential time decay from the initial maximum value /,,.., a8 shown below: toy eos dt The time 1’ needed to decrease the current to 50% of the maximum value is, then, given by: 40107 ¢=~tn(2)= R sa In(2)=5.5ms (a) The time dependence of the emf ¢(1) can be derived as follows: s(t)= e = -o fxs at)] NBSersin( ext) = 2nNBlnf sin (2a /t) (b) ‘The maximum emf ¢,,.. is equal to: 12007 fgg, = NBiwreg = 10" «2.57 X0.3 0.6 m? x29 x2. = 57 (©) The frequeney of the emf is equal to the rotation frequency of the coil f. The amplitude of the generated emf can be increased by increasing the number of loops V the magnitude of the magnetic field B the loop arca Jw , or the rotation frequency of, the coil f ‘The magnetic and electric fields energy densities, 11, and 1, . respectively, are given by: B uy = ye, 2 ‘The lowest values, (it, and (1,),, are: (4) <2 Cty 2x4x 2x10 TmA™ sssx1o {40 m 2 Copyright © 2014 by Nelson Education Ltd, 283 ‘The highest values: (v1, ), and (1(,),, are: 2 s7x10*T) (iy -2 (st) asaioe 24, 2xdxaxdO TmA m sssxi08 © -{rs0!) ; (u), = 2 =1.0x107 2 2 m ‘The magnetic field energy density at the surface of the Earth is more than 10,000 times larger than the electric field energy density. 73. The smallest voltage «,,, induced in the coil of the breathing monitor can be calculated employing Faraday’s law as follows _BaewN(A5)e _ 599.4 1, 30-10°T:5-104%m* = 3 4V vf min 60s 75. (a) ‘The current intensity can be calculated as follows. P_ ww vo 2x10V =5x10°A=SkA ‘The magnetic field at the surface of the inner conductor, &, is given by: _ tel _ 44x10 TmA* x 5x10" 2na 2a x25x10m 0.040 T= 40mT (b) The magnetic field at the inner surface of the outer conductor, B...» is I 210-7 TmAM! 5:10? fyl AR AO? TRAN SAPA 4 yy, wen Fa(ao by 2an(25150)-10°'m 3 3m (©) ‘The magnetic field at the outer surface of the outer conductor, By, 42 is B, or ‘The result expressed by the equation above is duc to the fact the current through a circular loop enclosing the superconducting coaxial cable is zero: 1, = 0A (d) ‘The infinitesimal cnergy of the magnetic field dU’, in the space between the two conductors of the coaxial cable is given by: (# ' B Bry] Da oy av, 20) ty POD appt 28D arte Ela 2 2th 2, sar 284 Copyright © 2014 by Nelson Education Ltd, Integrating the equation above, the total magnetic energy U7, is (3) Hence, the magnetic energy density, 11,, is given by: Holl (2 aif 4 of?) al »(2) An x107 Tm x25 x10°A? xIn(2) a(# a) 4 x(8—2.5")x10 me MLL pdr aa =0925 (©) The magnetic energy stored in the space between the conductors I, over a distance 1215000 is equal to: P “ " 42x10" (/=1500%m) = 4 w(5) = - 4a a : mA“ 25310" A? x15 x10%m x In (2) 4a =2.6MI (Q) The pressure exerted on the outer conductor p as a result of the power transmission can be calculated as follows: 4 soe > Sx10°TxSx10°A fo 3 pa 0.2 kPa 2nh 2axsxlo'm 3a 77. (a) The voltages across cach inductors connected in series are given by: (b) ‘The voltages across each inductor connected in parallel are equal to: . dl, di, Vy Vga anh hank Ba Copyright © 2014 hy Nelson Education Ltd. 285 79. (a) 286 mW ‘The sum of the currents is Tshth+ Hence, 2, (teeny! ‘The voltage across the inductor, V, in a series RL eirouit is given by: V, =2e * = In(¥,)=In(e' t 7 Hence, the semiclogarithmic representation of the funetion V’,(¢) is a linc where the slope, b, and intercept, a, are given by a=In(z) ‘The data from ‘Table 24-4 gives the following linear graph with a= 2.485 and = 0.333 ms - 25 pe atite 20 om 2485 b= —0333 ms! o123 45 67 8 9 ans) It follows that: r= b= 3.0ms Further, = L=1tR=3.0x10"s%20= 6.0mH Copyright © 2014 hy Nelson Education Ltd. (b) r= b=3.0ms (©) The cleotriec current 1(¢) is given by Ho=4[1-c*| af “ys ‘The following table can be generated using a spreadsheet program. The last column gives the current values for cach time according to the equation written above. Time 10 35082162 1.701 1372719852361 20 Gi) 25 sats ome 5382 30 Adi 1a05 8.798 as ays 1ata ata 40 ates 1182 Asie 43-2678 0.985 Aa so 2267 O18 4.867 ee Bo 184 0.des 5188 85 1375 ste 5318 70 1184.82 5ai8 75 0905 0.018 8.507 no aM 0.182 550 Copyright © 2014 hy Nelson Education Ltd. Chapter 25—ALTERNATING CURRENT CIRCUITS ST ‘The RLC cirouit is above resonance ‘The inductive reactance is direetly proportional to the AC angular frequency © ‘Therefore, a low frequency © translates into a low inductive reactance , . Qualitatively, the inductor’s emf depends on how fast the magnetic flux varies. Therefore, the induced emf is directly proportional to the AC frequency @. 5. The capacitive reactance 1X is, by definition, given by: wy oC ‘Therefore, capacitive reactance decreases with frequency. 7. No, the rms value of a voltage can never exceed its maximum value. An ms value of a voltage equals its maximum value only for a constant voltage 9. ‘The phase angle approaches the value 7/2 as the frequency inereases significantly 11, The following equations can be written 288 Copyright © 2014 hy Nelson Education Ltd. 15. @ x, =00=10 M4107H=10 5 N26 _ V2x20V 12 B) haus Y. v2. A2x20V = 22x10 A= 28nA 10° Ne i ©) fue © 16) iasin( 1-2) 19. ‘The inductance J, can be derived as follows: pat 1 Wnf=z=ol= $= ut vic 4n° x(10'Hz) 10°F 21, The factor by which the resonance frequency changes is denoted by fap / fo, and is equal to: fe [E Ve wren to N20 2B. @) sin(0)= 0 = 50001 = 2kn=0 = 8, = 01,2 (6) sin(0)=1 50000 274, , 2mtDe, 924, 2 10000 (©) sin(0)=-1= 5000 = D7, , Catz, 2 10000 Copyright © 2014 hy Nelson Education Ltd. 289 () @ © 27. @) 22.1109 | 9000 rad/s. () 25H 250V_ RYl+[tan(¢)} 10002414 [tam (0.79) 29. The inductance £ can be calculated as follows © =18A a ropes pera i 2 SL =C(ROY =107 Fx(10' 2x18) = 0.2311 RVC (2) et) 31, The maximum number of bulbs that can go in the cireuit NV is equal to: 33. Yes. the energy delivered by the emf is cqual to the magnetic energy stored in the magnetic ficld of the coil and in the clectrie field energy associated with the charge stored by the capacitor. 290 Copyright © 2014 hy Nelson Education Ltd. 39. ‘The inductance of the motor J, can be calculated as follows ae] -eos \ =120miL 2a x OOH ALL _ sin(2a0-4)=10y2V =14V 45. The resistance R and the capacitance C can be calculated as follows: a ) 47, Py = Maer jig, = ma? Z = min= X, X,=>9=0" (resonance) 49. R= =0 Copyright © 2014 hy Nelson Education Ltd. 291 51. Using equation 25-39(c) derived in seotion 25-6: R 2ak 1 1 AMF L 4a? x(101.5<10°H2) x10°H Af == R= 2aL Af = 2a x10*H x40 x10°Hz = 2.5m2 =0.25nF 53. A resistor R and inductor £, would be selected based on the following two principles: (2) The resonant frequency @, would be equal to the pre-selected frequency of the emf © o=a where the capacitance C, would be the approximate middle of the range in which the capacitance varies with temperature AC = AC(T’) (2) The resistor 2 would be large enough to provide a width of the resonance peak Aw to match the range of resonant frequencies Ae. given by the range AC: Ao ‘The voltage amplitude across the resistor V/s... ean be recorded using an oseilloscope for various controlled temperatures of the capacitor. This will be the calibration curve of the thermometer. This curve can be subsequently used to measure unknown temperatures. 292 Copyright © 2014 hy Nelson Education Ltd. Chapter 26—ELECTROMAGNETIC WAVES AND MAXWELL’S EQUATIONS ST 1 9. ret 15. 17. Itexplains the generation of varying magnetic fields from varying electric fields. (a) & and B waves are on the same axis (© # and B waves have different phases (d) and B waves have the same amplitude. (Sound waves are not fields; they do not propagate in vacuum Gi) Sound waves are not transverse waves (ii) Microscopically, massless particles called photons are associated with electromagnetic waves, quasi-particles called phonons are associated with sound waves. Deceleration and acceleration in a straight linc, a curved trajectory, or oscillations No, i cannot because of the energy loss that is associated with the emission of electromagnetic radiation. ‘The radiation pressure is twice as large on a surface that is a perfect reflector than on a surface that is a perfect absorber. The difference is duc to the change of total momentum of the system (radiation + surface) which is equal to twice the initial momentum of the radiation for the perfect reflector surface, and equal to the initial momentum for the perfect absorber surface a, (@) B=-—S ~o3201 2ar & B= 4222 00u7 Qar (@) I(¢=0s) (b) i(¢=60s) Copyright © 2014 by Nelson Education Ltd 293 19. 8 294 . Bye ALTO" (@) B(r=0s) fy I(t = 60s nr B(t=60s)= (a) Sis in the y-direction . @) 5 is in the(—2)-direetion (©) Sis in the z-direction . (@ Sis in the(—x)-direction ae Tpt ¢ E can be oriented as follows: (@ along y-direotion or (—y)-direetion if the electromagnetic wave travels along (-2}-direction or 2-direotion, respectively; (ii) along z-direction or (-2)-direction if the clectromagnetic wave travels along yedirection or (y-direction , respectively. Qn @ 4 1.57x10°%m =1.57nm =1.91%10" Hz (b) &, = ch, =1500 Vim (©) Bl x,0)=(15001 /m) sin( 4.0010" x—ar)(—5) (4) The wave travels in the x-direction using (x,t) from part (c). Copyright © 2014 hy Nelson Education Ltd. 25. 27. 29. Be) e B(t)= 3107" in the (-2) direction Microwaves < infrared light < yellow light < blue light < X-rays < gamma rays, @) f= 2 6.67x10" He = 5.6310" Hz (b) © = 4.2910" He B= 2e8 = f2, 78VA @ Favre Mie fm 3 P By =—b = 245 = 24 > = 45 KOT oe oar * ) B= )2K08 pager 78Vim ar Re - za P 5 = 2H 8 = Py AAS x10" T qe tar 7.8104 Vim ©) By =V2Mes = [2146 4m 7 J2n,5 = [24, Po nasxto'T Ve" gar Copyright © 2014 hy Nelson Education Ltd. 295 39. ‘The average Lorentz forve (F, ) is zero due to various orientations of the eleotron speed ‘with respect to the magnetic field of the sun’s light: eb, + evB,sin (5,8) =2epyeS = 6.0 x10" N 41, B= Sat =4000kW-h 1.5910? Wem? 0.610 Pa pas 22 2333x100" N @ 45. ‘The electromagnetic wave is polarized along the y-axis. 47. rom Malus’ law; J =0.25/, = /,(cos 0)’ >= 00° Therefore, the angle with respect to the vertical is: 90° =30° 1, (cos 45° = #2 (cos 5°) =0.25 Win? 51. (@) i, =1,(cos90°Y =0W/m? (b) i, =/,(cos90°) =0 Wim* (©) 1, =/,(c0s90°Y =0 W/m? 1, [vos(45°-30°)] = 0.75/, (cos15°¥ =0.70/, J, =1,(cos45°) =0.35/, 296 Copyright © 2014 hy Nelson Education Ltd. 55, When the unpolarized light is incident on the first polarizer oriented at angle @ with respect to the vertical, the transmitted wave will decrease its intensity by a factor equal to (cos 45°)" and its polarization plane is oriented at angle @ with respect to the vertical. The second polarizer is oriented at an angle Y with respect to the first polarizer. so the intensity is decreased by a factor equal to (cos. and so on. Therefore, the following equation can be written: 3, 0.5m 8pcGM,,, “Thus the diameter is: d =2r = 1m. The following numerical values were used in the equation written above: G=6.7x10" m?-kgt-s* (universal gravitational constant) P,,, =3.910" W (radiation power of the Sun) Moy, = 2.010" kg (mass of the Sun) (b) For any radius of the particle 7, the ratio between the two forves is given by: Therefore, if the particles have a diameter smaller than I uum, the force due to the radiation pressure of the Sun will execed its gravitational attraction force ‘Yes, the tail will be longer as the comet approaches the Sun. For particles below the radius value calculated in question (a), 1, , the radiation pressure will exceed the gravitational attraction of the Sun. ‘Thus, these particles will be pushed further away from the core of the comet and further away from the Sun, forming a longer tail. Copyright © 2014 hy Nelson Education Ltd. 297 Chapter 27—GEOMETRIC OPTICS 1. The distance between an object and its image in a plane mirror is equal to twice the object- mirror distance. Therefore, if the object-mirror distance is increased by Av, then the object image distance will increase by 2Ax a.,—d,=2d,—2d, =2(d,—d,)=2x0.5 m=tm A plane mirror inverts images in a direction perpendicular to the plane of the mirror: ic, plane mirrors perform depth inversion. Plane mirrors do not interchange left and right; see the discussion in the “Making Connections” box on page 732 of the text. ‘The word “AMBULANCE” is written from right to left on the front of ambulances so that drivers can easily read it in their rear-view mirrors. 5. ‘The correct answer is (¢). 7. The side-view mirrors of ears are convex mirrors where the image of the object is smaller than the object (magnification M < 1), Because of the less than unity magnification, the objects appear further away than they really are, The convex mirrors are used because the image is upright and because magnification less than one allows the driver a larger field of view of the road behind the car. 9. (a) Yes, an image of the rose will be formed. (b) ‘Tosee the image of the rose in the mirror the observer has to be located in the lower left region of the rose-mirror assembly as shown in the figure below object image ot plane A tion observer 11. ‘The maximum number of images that could be viewed is 3. The number of images is equal to the number of times a ray of light parallel to the axis of symmetry between the two mirrors is reflected. 298 Copyright © 2014 hy Nelson Education Ltd. 15 17. 19, 21. ‘The answer of question 12 is based on the fact that a reflected ray of light from the toe of the observer has to reach his/her eye as shown in the ray diagram from Figure 27-12, The correct answer for a spherical mirror is (f) based on the image properties of spherical mirrors given in Table 27-1 A concave mirror has to be chosen that magnifics when object-mirror distance d, is in the range: R/20 and dj >0) 2) Copyright © 2014 hy Nelson Education Lid 307 Ie follows that, 1 1 bee i =—xS =0.45 aha fo“ cme oasen a (b) The distance between the lens and the CCD sensor is equal to the lens-image distance d d, = (Md, =0.1<0 45 em =0.045 om 57. (a) If n=152 is the index of refraction of the glass prism, the refraction law on the left- hand side face of the prism and the ray geometry indicated in the diagram shown below yield the following equations: sin (1) = nsin(r} =r =sin Jes Pr a nti) Mr=90° > i 0° — 34.7 =55.3° ° ‘The critical angle Q, for the glass-air boundary is o,2sin*{ 1) -a11° 12 Since the incident angle on the right-hand side face of the prism ("= 55.3° is larger than the critical angle of the glass-air boundary, it follows that the light ray undergoes a total reflection at this interface and exits at the base of the prism. (b) The angle between the emergent beam and the horizontal is denoted by a in the ray diagram. Using triangle NOC the following relationship can be written. = 138°, a =(90°— 7") + 45° = 135°. 135° $5.3" = 80° (©) ‘The light ray exits trough the base of the prism, so the height of this point is zero, 308 Copyright © 2014 hy Nelson Education Lid 59. (@) The focal lengths 7 and f, of the lenses with optical powers 4.0 D and 5.0 D, 61 respectively, are: =m =25em =20em wipe Ble (b) Tho image-lons distance for the first mirror d, can be calculated from the thin lens equation: d. 50 em x25 “dy-f 50cm-25em , = 50cm “The image formed by the first lens is an object for the second lens: d.,,=d-d, = 90 cm—50 em=40 om ‘The image-lens distance for the secand lens is df, _ 40 emx200m d,-f, Wom—200m =40 em ‘Therefore, the image is formed 40 cm behind the second lens. (c) No, the combined power of the two lenses cannot be used to solve this problem because the distance hetween them is large relative to their focal lengths. The combined optical power describes a system where lenses are very close together Simplified notations for incident and refractive angles are given in the ray diagram shown below. Notice that @ =/ in this new notation. Using angle relationships in triangle MON: Sa(i-r)t(rSisit eri w@ ‘The following angle relationships can be written: AMN+ MNA = (90° —r}+(90°—i')=180°—g=> r+i'=9 @ The refraction law applied at the entry and exit boundaries of the prism gives: Z in (f sin (:)=nsin {r= r=sin [= 4 @) A 9.02) sin(r')=r'=sin“[nsin(/)] = r'=sin™[nsin(6—r)] @ sin ( ‘Using equations (1)-(4), the deflection angle 4 is given by’ Copyright © 2014 hy Nelson Education Lid 309 (5) 63. (a) ‘The focal length of the convergent lens has to be equal to the sum of the distanes between the lenses and the absolute valuc of the focal distance of the divergent lens as indicated in the ray diagram shown below, Hence, the distanec between the lenses. d is: d-F,-\Al=2/4 o d= Bo 1BI= 218 (b) The distance between the lenses is equal to the sum of the absolute values of the focal lengths of the two lenses as indicated in the ray diagram shown below. ' teh thl=4lh| — rr ea 1 +lAl 4A] @ 310 Copyright © 2014 hy Nelson Education Lid 65. (a) ‘The ray diagram is shown below. (b) ‘The focal length of the mirror /,,... is given by: oame = =25em>P,,,., =4D a The focal length of the lens can be calculated by employing the lens maker’s equation: 1 1 = -1)) —-— (25 ie i +] @ The values of radii R, and R, are given by Ree @) R,=-R=-50.em o ‘Therefore, Fae x50 em=1.5 mF, =0.67D 1.33-1 (©) The optical power of the system P., is equal to the sum of the optical powers of its ‘components P og = Paanoe + Pag: #40 D 40.67 D =4.7D Copyright © 2014 hy Nelson Education Lid 31 67. The converging plano-convex lens produces a virtual image if 0H One of the solutions of quadratic equation (3) is not acceptable because it gives & 7, ‘The remaining solution is «@) ‘The geometry and the balancing of forees yield the following equations as indivated in the diagram shown on the next page: d, =1c0s(0) (6) R=isin(8) ) mene Yi tan (0) = eS san( 0) 2), ca(0) = o mg 2 ol Combining equations (5) and (7), the following result is obtained! g g d,-2sa- |£ 8) ge @) Using the result of equation (4), ow |_ 28 J 2x98 is? entig Holi ai ime my mam Copyright © 2014 hy Nelson Education Lid 313 73. (a) From the ray diagram geometry of this optical system shown on the next page: d,=-L/2= 40 em /2=~-20 om ay ‘Using the thin lens equation for the second lens (L2): a0 d, =20 em)» 30 em igen aban) ites From the ray diagram geometry d, =L—d,, = 40 om—12 em =28 em ® Using the thin Lens equation for the first lens (L1): 12em @ 314 Copyright © 2014 hy Nelson Education Lid (b) ‘The magnification of the optical system Mis given by dd, _ 2 4,4, 3 7 M=MA, (©) Yes, the compound microscope can produce an upright final image if the image produced by the first lens (L..) is between the two lenses. That is, Rim, =m, = 64x eT Boum Correet answer: (c). Using the constructive interference condition for the double-slit experiment and the small~ angle approximation, it can be determined that the angle separation between the central spot and the first-order bright fringe on each side is approximately given by (sce also equations (3) and ) from the solution of question 4): o~ d Hence, for two different separations between the two slits d, and d, , and the same wavelength 2, one can write that Correet answer: (b). ‘The constructive interference condition for a diffraction grating is dsin(0)= mA ‘Therefore, since d is constant, the bright fringes which are nearest to the central white spot correspond to the smallest wavelength 2. For the stellar visible light spectrum, the ‘minimum and maximum wavelength values correspond to the colour violet and red, respectively, Correct answer: (4). Copyright © 2014 hy Nelson Education Lid WL 15, 17. ‘The destructive interference condition for single-slit diffraction is: wsin(0)=ma Therefore, for a constant slit width w, a larger wavelength 2 will yield a larger angle @ satisfying the equation above; hence a more spread out diffraction pattern in both directions Correct answer: (b) The first order of constructive interference for a light beam of wavelength 4 in vacuum. normally incident on the soap film with index of refraction 7 is: = Aa4nt ‘Hence, as the soap film evaporates its thickness ¢ decreases, and so does the wavelength of the light 4 for which the interference condition holds. Correct answer: (b) ‘The resolution improves iffan electromagnetic wave with smaller wavelength 2 is used Correet answer: (b) Geometric optios can be used in cases where the electromagnetic radiation wavelength 4. is much smaller than the size of the object to be investigated, The smallest dimension concemed in this case is 1 mm. 32 nm = 0.532 x10" mm <« 1mm @ A Geometric optics can be used. (bh) 4 pm =5 x10? mm < Imm Geometric optios ean be used. 80 nm = 0.1810 mm < Imm @A Geometric optivs ean be used, B: Feet OEE ai enc ee f 88x10" Hz Geometric optics cannot be used (a) ‘The phase difference between the two radio waves travelling the same distance is zero given that the initial phase difference between the two antennas is zero, Therefore, there is constructive interference which can be observed by a detector positioned at the halfway point between the two antennas (b) The destenetive interference occurs when the phase difference is equal to 180°. This corresponds to a wavelength diffrence equal to odd multiples of 7/2. Ifthe detector is moved by adistance equal to 2/4 to the left or to the right, then the phase shift Copyright © 2014 hy Nelson Education Lid 317 19. (@) ) (b) ) 318 between the two detector will be 180° , corresponding to destructive interference condition. The minimum distance d,,, required is then equal to d_c ___3x10'ms d, ee a _ 4 Af 4x1500x10° Tz =50m ‘The constructive interference condition for the double-slit interferometer is: dsin(0)= mA ‘The first-order bright interference fringe ooours for m= al 63310? cow (eae Sa 25x10" m From the geometry of the double-slit experiment shown in Figure 28-8 from the textbook, it follows that the distance hetween the first-order bright fringes denoted by d, is d, = 2D tan(Q ) = 2x 45em » tan (0.145%) = 0.228 em = 2.28mm. in the above equation. Hence, jase (0.002532) =0.145° The distance from the centre of the m=1 image d, = 2.00 em is given by: 4, = Dian(4) In the equation written above D denotes the distance between the diffraction grating and screen. It follows that: =m *( 2) a tant { 22207 | 24 530 D 0.750m ‘The number of lines per centimetre N’ can be calculated from the constructive interference condition: sin(Q)__sin(1.53°) a 700% 10° m A sin(Q.) = 2= ‘The distance d, for 2= 400 nm is equal to: d, = Dian(4)= DQ © DNA =0.75m x 38143" x400%10" m=1. 14 om Assuming a hard reflection on both interfaces by the antireflection film, the minimum film thickness ¢ can be calculated by using the destructive interference condition: A _ 123010" m an x38 =226nm ‘The constructive interference condition is: AL, ant Sha ” m Copyright © 2014 by Nelson Education Ltd For the first-order of the constructive interference (m =1), the above equation gives that: A= 2x1.38x226 nm = 624 nm 25. The width of the slit w can be calculated using the destructive interference condition for a single-slit diffraction is md _1x589x10% m sin(@)— sin(1.13") 29.3 gem 27. Only bright fringes within the central maximum corresponding to the single-slit diffraction pattern are counted. The first-order destructive interference condition for the single-slit a ‘The constructive interference condition for the double-slit experiment gives: ma sin(@)=— 2) (a) a 2 From cquations (1) and (2), it follows that: d@_ 0.125 mm w 0.0150 mm Eight bright fringes can be seen on each side. Therefore, including the central peak, 2x8+1=17 bright fringes are visible 29. Using the Rayleigh criterion, the smallest angle 4, separating two points that ean be resolved with light of wavelength 4 and using a circular aperture of diameter D is: 4a sin(@,)=1.22% (8, )=1.225 Assuming that the average light wavelength the eye is most sensitive to is 2=550 nm and L 2.85mm,, then = 55010 m in| 1.22 | 2510?) 0135" If the distance between the observer and the car is 7 and the distance between the lights is d=1.45m, the following geometrical relationship is valid: tia 2 pe 4 s__ Ss __rg ttc 7 (4) 2x tan(0.0135°/2) 2tan[ Copyright © 2014 by Nelson Education Ltd 319 31. ‘The bright fringes counted when going from evacuated to containing the gas are due (o the phase difference between the two anns of the Michelson interferometer that is induced by the wavelength-reducing shift (./ 1) duc to the light passing twice through the arm containing the gas with index of refraction . If the length of each arm is denoted by /, this ‘means that the equivalent length of the arm filled with gas is equal to nl. Given that the only phase shift is duc to the presence of the gas, the constructive interference condition is ma=2(n-1) It follows that dy, MSO? my ooog ab 2«0.365m ‘The constructive interference condition for the two-slit experiment using light of wavelength A and the distance d between the two slits, is dsin(0)=ma Based on the equation written above, the angular separation Q between m=0 and m=L spots is given by: If the angle between one m=2 maximum and the center is denoted by @, then the angle between the two m=2 maxima is equal to 24, =1.30°. Hence, , = 0.65°. The spacing between the slits @ can be calculated from the constructive interference condition: 2A _ 2x480x10 m sin(@) —_sin(0.65") = 0.0846 mm = 84.6 sem 37. @) Let @ and @, denote the deflection angles for the first-order interference fringes corresponding to the wavelengths /, =350nm and 7, =1000 nm, respectively. Using the constrictive interference condition for the diffraction grating with NV lines per unit Jength, one can calculate angles Q, and 0, as follows: 0, =sin" (NA) snr [00 (10° m)'350510° me] 12.19 0, =sin (NA) 6.98 et [s00.(107 m) * 1000x107 m] (b) The effective width of the detector w is equal to: w = 2048 pixels «14-4 = 28672 am = 28.7mm pixel 320 Copyright © 2014 by Nelson Education Ltd 39. AL (©) Given the deteotor width w calculated in question (b) and the grating-sereen distance denoted by D, the following geometric relationship is valid: w= D[tan (4, )tan (4 )] Hence, the distance D, in millimetres, is equal to: _ 28.7 mm tan (,)—tan(@) — tan(36.9°)—tan(12.1°) D 535mm The constructive interference condition for the reflected light rays between the two glass plates separated by distance ¢ is given by: a In the previous equation, a hard reflection of light on the glass plate equivalent to a 180° phase shift was assumed. Equation (1) can also be written as: @ ‘The maximum distance between the glass plates is ¢,.. = 75 sem. Hence, using equation (2), the maximum number of bright fringes that ean be observed will be given by: Dug 1, 2x TSX10% m 2° 532x10"m = 282 @) (a) The Rayleigh eriterion for a cireular aperture of diameter D and light wavelength 4. gives the angular resolution @, as: 0 =sin™ (\ nd) D Assuming 4= 501m the value of 0, for the France-Canada-Hawaii ‘Telescope is 350x107 m @, =sin™|1.22% 3.6m - 0,0000107° 0.0000107°= 0. 000107°x-3600 7 = 0.038" (b) . ‘The array approximates a telescope with a much larger effective diameter D In this ease the diameter can be approximated as being equal to the distance between the two telescopes D~=3000km, The angular resolution @, given by the Rayleigh criterion is: sin'{ — S10 ms___) _(g,gy4)> 3x10" m x1.42 «10° Hz Copyright © 2014 by Nelson Education Ltd 321 45. The Rayleigh criterion relating the angular resolution 6, for a circular aperture of diameter D with the radiation wavelength 2 is: a sin(@,)=1.22— (4) a It follows that: 1.222 _ 1.22%500x10? m a REN TREE = 0.0839 m =8.4 cr sin(@,) sin[ (1.5 3600)°] meson 47. The destructive interference condition for the diffraction on the slit of width w is: wsin(A)=m4 @ Therefore, the angle @, of deflection corresponding to the first dark fringe (m =1) is given by. @ We can now specify the condition that the first three bright fringes originated from the two- slit interference pattern within the angle 4, on each of the two sides away from the centre: dein (6,)=3% ® ‘Using equations (1) and (2), the following equation is obtained: gp 2d 0.125 3S 417 ym 49. Answers may vary. 322 Copyright © 2014 by Nelson Education Ltd Chapter 29—RELATIVITY 2Gm 5. d. Since ry, = 2", doubling the mass would double the Schwarzschild radius. 7. ¢. Space-time curvature is greatest when m/r? is greatest. 9 4 0.5e40.5e I, ¢. v= note 0.5e)(0.Se CESS) 13 ¢ 15. ———_—_ = 1.26 «10* N/a? 17. Total distance r= 27(R,,,, +10°) = 2a (6.37 x10" +10° )= 4.6310" m x *. 4.63x10" @ e228 = 46x10 _ q O52 (05)(3.00%10) 0s x10" 5c) _4:63%10"__ 95 = 0278 (0.5)(3.00x10") 19. v= 4.40 ly = (4.4)(9.47x10" )=4.17 «10 m x_ 47x10" @ (=== (0.75)(3.00 10") 41tx10% _} (0. 75e)" Tago =1.22%10" s =1.85 x10" s Copyright © 2014 by Nelson Education Ltd 323 21. 25, 324 Relativistic kinetic energy is given by K=me" CF ia ‘This can be rearranged to give cei gor Vooory SP a, 1 => v= 3.00« 10") 1 J ‘The speed is not quite equal to the speed of light, but the difference is extremely small. no prey, Conservation of total energy with the assumption of zero rest mass of neutrino gives 2 2412 2,liy mol =m,cttomyieme toms po + TMV, Me +> Mcuty deise Sieve pmiM = me? mae ‘The left side of this equation represents the total kinetic energy released. = Ksnasa =(m, —m,—m, Je > E gosus = (1.674927 x 10 1.672622 107 —9.10938 x10 (3.00 x10") 1.2547 x10" J 1.254710" mies 1 6922x10™ SK, =, = 0.7831 MeV For Mei the time interval is zero, that is Mug =0. ‘Mei sees two events separated by 300 m in space, that is (Av + Ay? + a2"), = 3007 Lisees the same events separated by 500 m in space, that is (Av + Ay? +24), = 500" Copyright © 2014 by Nelson Education Ltd 27. Since the spacetime interval for the two observers must be the same, (cA, f(a + AS? + AY, = (ogy (axe 4 ANE =2 (At) = 0-300? + 500* — 160000 vi60000 3-10" 3x10" At, =133-10" Af, —1.33 4s Radius of the event horizon can be calculated from 2GM Se e Using the given mass of M = 12M, =(12)(1.989x10" ) =2.387%10" ke (2)(6.672%10 }(2.387 x10") (3.00x10")" 35.4 km 35391 m. A light year corresponds to the distance travelled by light in one year. = xa cr = (8.00% 10°)(1x 365.25%243600)= 9.4710 m (a) Since radio signals travel with speed of light, the total time will be 2x (25.3 ly) = 2%(25.3%365.25 x 243600) = 1.60 x10? s = $0.6 y (b) The total round trip distance is given by 2) = 2e x( 25.3 ly) = 2(3.00 x10" )(25.3x365.25 x24 x3600) = 4.792107 m ‘The time taken by the ship in Earth's frame of reference is %_ 479x1 v (0.985)(3.00 10") 62x10" 5 (©) For an observer on the spaceship, the distance seems contracted, that is Copyright © 2014 by Nelson Education Ltd 325 4.7910" (0.988) c* (0.985)(3.00x10") Se 80 <10* s 33. (@) The time for the passengers in the transit line will be dilated according to This can be rearranged for the speed of the transit line as van fa Its given that the time for the passengers is half the time for observers on the stations, that is r=. 2.60 10° m/s (by 15 = S81 003s 35. An electron accelerated through a potential difference of volts gains a kinetic energy of VeV.. Therefore, in this case the kinetic energy of the electron is K =32 keV =(32000)(1,602 x10) =5.126 x10" J 1 a) K= Ar, =1.49x10 s=14.9 ns 3.010" (b) Since time is dilated, we can use the formula =eft-(M2) -20- 200x108 ms 7 200) “3 Copyright © 2014 by Nelson Education Ltd 45. Let v, and y,, be the speeds of spacecraft and meteorite with respect to the home planet. ‘Then the speed of the metcorite relative to the spaceship is given by vet, oe BSE oo 1 )(0.5e" qaeeemee For an observer on the spaceship, the meteorite will seem to be traveling at this speed. The time taken by the meteorite to pass the spaceship is then given by L 250 (0.91)(3.00 x10") 29.17 x10" 0.917 pes 47. The velocity of the object with respect to C is given as v4, =0.3¢ C itself is moving with a velocity of v,, = 0.4e with respect to B, which is moving in the same direction with a velocity of v, = 0.5¢ with respect to A. Therefore the velocity of C with respect to A is given by Wypt¥e _ O4e+ 0.50 pe tere 1, (0-40)(0.52) e e 0.75e Now we can calculate the velocity of the object with respect to A palates O7SC+O3C 9 96, Yea ) (0.752 )(0.3 wo ot 49, ‘The Schwarszchild radius is given by R= 2Mt = 2)(6.67210""')(10* 3g = MOT NO) ct (3.00%10°) Mase density is given by p= 10 7.32410" kyim! Sart, Aa(vasxi"y Copyright © 2014 by Nelson Education Ltd 329 Similarly, for the maximum mass we have . (2)(6.67210™" )(10"} = = 1.4810" m (00.108) 4 4 a) SAR gig —7(LABx10°) Note that here we are assuming that the laws of geometry and physics inside a black hole are the same as we currently know them. This may not be true. 51. (@) The relativistic kinetic energy is given by This can be rearranged to give (me myo? +K Given: K =7.0 TeV = 7.0 10° MeV Rest energy of a proton is mc? = 938.257 MeV wy et-| 258257 Yo 0000 938.257 + 7.0 x10 (b) The time taken by a proton to complete one revolution is, x 27000 oy (0. 999)(3.0010") Hence the number of revolutions per sccond is given by 01x10 Here Fis the total energy. (938.257 +7.0 x0") - (938.257) = pm =7.0 «10° MeV/e = 3.710" kg m/s 330 Copyright © 2014 by Nelson Education Ltd 53. (a) Each particle is moving with a velocity of 0.8c. We have to calculate the velocity of one of these particles with respect to the other. For that, we use relative speed formula yew 2 ly Spygate __ gorge: ° Lo-BeVO-Re) (©) Kame 1]=3.59mc? [_(os7ee} yoo 55, Conservation of energy gives met, met met Copyright © 2014 by Nelson Education Ltd 331 ‘This equation can also be written as @) Conservation of linear momentum gives mi mu from equation (1) into this equation yields 57. Assume an elevator in fie fall on Earth, An apparatus on the floor of the elevator emits a photon as the elevator starts falling down. A detector on the elevator’s roof measures the frequency of the photon. If the height of the elevator is Ar, the time it will take for the photon to reach the detcetor in the clevator’s frame of reference is given by aah Foran observer on Earth, during this time, the elevator has attained a velocity of ar ieee 332 Copyright © 2014 by Nelson Education Ltd But we know that g = SM Ar =y For the observer on Earth, the photon undergocs a shift in frequency, duc only to its relative velovity, given by ‘This, according to the equivalence prineiple, becomes Af _GM Ar for For small Af as compared to f and small Ar as compared to r, the above equation can be written as if dye i Integrating from infinity to R, we get fal GM or ar GM. er dr A Taylor series expansion of the right side of the previous equation gives fi, GM Gim? Bigg oot Ao SI to OR 201K In the limit c?R, >> GA, this can be written as Fs (- 2GM y h OR But we know that frequeney is inversely proportional to time. acm eR, Copyright © 2014 by Nelson Education Ltd 333 59. (a) Tau Ceti fis a habitable exoplanet that is assumed to be orbiting a nearby star Tau Ceti. 334 () © (d) (e) ‘The planet is about 11.905 light years away from our Sun. It is most likely 2 to 3 times the size of Earth. Not much data are available about its atmosphere but it may be habitable with a surface temperature that can support complex life. A reasonable time of travel would be 10 years such that a 30-year-old person would be 40 years old when she reaches the new pianct. ‘The time in the spaceship’s frame of reference is given by: Here x is the distance from Earth to the destination. x= 11.905 ly = (11.905)(9.46x 10") = 1.13 x10" m The above equation can be rearranged in terms of velocity. ayn Let us suppose ¢ = 10 y in the traveller's frame of reference. 10x 365.25 x 24x 3600) (uasaory (3.00 x10* ) =v =2.30%108 ms Since the trip takes 10 years, about 200 people should be sufficient for this trip, given that cnough people will reach the new planet even after deaths duc to discases, accidents, and natural causes. Moore calculates that about 150 to 180 people are sufficient to sustain a population over many generations Since the spaceship is to travel with a speed very close to the speed of light, its mass should be as small as possible. Sputnik 2 had a payload of about $00 ke with a passenger. Even though this underestimates the mass requirements of very long travel times, still we will use this to calculate the payload of the spaceship. Since we have 200 passengers, including crew members, the payload is M = 200x500 =10° kg Copyright © 2014 by Nelson Education Ltd (®) ‘The energy required for this mission would simply be the kinetic energy of the spaceship, that is 1 — h— Be myc? 1 (0°)(ea0°) = s.02x10 5 ' (x10 Annual energy use in Canada in 20L1 was about 1.7010” J, This implies thatthe mission will take °°2*19—" & 5900 times more energy than the annual energy —_— I (2.30«10°) 70x10" consumption in Canada, Copyright © 2014 by Nelson Education Ltd 335 Chapter 30—FUNDAMENTAL DISCOVERIES OF MODERN PHYSICS 1. In 1897, J.J. Thomson, postulated the existence of particles smaller than atoms while trying to explain the existence of “cathode rays” that he produced in a glass tube. He postulated that these smaller particles were minuscule pieces of atoms. Later on Philipp Lenard performed more experiments on these cathode rays and showed that if cathode rays were particles, their mass must be smaller than the masses of atoms. Atoms have concentrated positive charge, a large empty space and negative charges. Some atoms can spontaneously produce positive, negative, and neutral rays. When subjected to external light, some atoms produce light of different wavelengths. 5. Bohr quantized the angular momentum of electrons bound to atoms. 7. A blackbody spectrum is continuous whereas a discharge spectrum has peaks at certain ‘wavelengths characteristic of the material, 9. ‘The thermometer works by detecting infrared radiation from the eardrum using a device such as a thermopile. 11. Hydrogen: g = 1.602 «10° C, m=1.674x10 kg mF 21602410 9570x107 Cig im 1674x10 Oxygen: q = 1.28210 C, m= 2.657 x10" kg, gq _ 1.28210" 4.82310" Cikg all, m 2657x10 15. (a) The object with higher intensity (black linc) is at a higher temperature (b) No. 15 1875x107 (b) The equation for the wave number is given by = 0" =|m " ‘This can be rearranged to give 1 1 v Copyright © 2014 by Nelson Education Ltd By trial and error, you can determine that for n, ~ 3 and 7, = 4, 161 momo which is sufficiently close to the calculated value to conclude that these are the correct values for n, and 7. + ~o.as611 16 (©) The wavelength is 1875 um corresponds to light in infrared region of the EM spectrum, 17. (@) ‘The electron experiences a centripetal force equal to the foree due to the magnetic field; that is gre (Lo0rs10™)(0.203)(18107) m 9.109 x10" (b) ‘The electric ficld must balance the magnetic field; that is gE = QB > B= vB = (6.426 10" (1.8% 10°) = 1.210" Vim v © EF _¥ 1380 = 10? m =1.3 mm EB 1187x10 300 _ 15000 vim ad 002 19. (a) (b) Ifthe electron is travelling in a straight line, the force on it due to the magnetic field must be equal to the force duc to the electric field. This means that the force due to the magnetic field can be calculated from F = gE =(1.602 x10" )@.5000)= 2.4 x10" N (© The forve due to gravity is simply the weight of the electron; that is, P= mg ~ (9.109 x10)(9.81)=8.94 xL0™ N Copyright © 2014 by Nelson Education Ltd 337 2 338, (@_ Yes, itis reasonable to neglect gravity since the electric and magnetiv forces on the clecton are greater than the gravitational force by a factor of ~ 10! ‘The weight with buoyancy accounted for is given by 4_3y Wasa? (P-Pa)e The weight without buoyancy accounted for is wad aapg ‘The relative error between the two is Poy POP =e x100 (@) Py =821 kg/m? Poy =1.204 kg/m? x100=—124 499 0.147% 821-1.204 (D) Pron, = 950 kg/m? Pay = 1.204 kgim® sen — Pe 100 = 174 _ 399 0.127% Pracx Pay 950- 1.208 hPa w 3kQ? SR (8.988 10" (791.602 x10" ((10") 8.010 J Copyright © 2014 by Nelson Education Ltd 25. (a) Given K=25x10" J = baw = 2.5.10" 2 = my =5.0x10 ‘The stopping distance is given by 192" (1.60210) Sn= y °* Fessn = soa07) em (b) 10x10" J = tin? =L0t0" 2 = mv? =2.0%10 ‘The -particle penetrates to a distance from the centre of the nucleus given by 79 __r9ftsor0 * (8854x107 )(2.0 x10 ) 27. (a) Kinetic energy gained by electron is K =1000 eV =(1000)(1,602 x10 3.6x10™ m =1.602x10" J ®) K 3m PE __|(2)(1-so2x10) => v= J— =.|—*____* = 1.87510" mi Yim ~V~ 9.10930" ™ (©) K = 1000 eV 29, (@) By =2.28 eV =(2.28)(1.602 x10") =3.65 x10" J Hy _ 365310” 4 s.Sbet0" Hz © T= 662610 ©) oh = 3010 5.442.107 m 544 am fy 5512x100" Copyright © 2014 by Nelson Education Ltd 339 3L @ fas Calculated values are listed in the table below. () Kye =@AV Calculated values are listed in the table below. 1 ASIE=15: 250 0.65 1.206 +15 1.057E-19 240 07 1.250615 1.9906-10 230 7.08 19086115 1.748 19 (c) The plotted curve with the fitted straight line is shown below. 20 18 16 Ma yo TE Se 6.9934, 120+ 15 L2anE+ 15 i Frequency (Hz) On-+ 1S (@) The slope of the line is m=(710'*)(10")= 710 Is ‘This value is fairly close to the Planck's constant of 6.62610" Ts. (©) ‘The intercept is the work function in joules, that is 6.993 x10" KE, G224x10™, 1.60210" 6.993 x10" J= 37 eV 340 Copyright © 2014 by Nelson Education Ltd 33 39, ‘The speed of an electron in a Bohr orbit is given by nh amr We know that for n= 1 the radius is r = 0.529 x10" m (also called the Bohr radius). (1)(6.626 x10") (2a)(0.109s10 Y(0529510") v =2.19%10° m/s ‘The period is the time taken by the electron to complete one revolution; that is 2n)(0.529x10"" 2H COOSA") 2 sie v 2.18810" (@) The wavelength of an incident photon is (6.626 x10 )(3.0010") © (50000)(1.602 x10") ‘The wavelength of the scattered photon is h ——( = 3.505107 m Ae, 1=cosé) 6.626 x10 * (9.109 x10" }(3.00 x10") 3 A412 x10" m SU3S4Sx107+ 1~ces 40") ‘This wavelength corresponds to an energy of ne (6.62610 )(3.00%10°) ae 4112 x10" 4.83410" 1.00240 4.83410 302 keV (b) ‘The energy lost by an X-ray photon is carried away by the scattered eleotron. EB, = E-E' =380-301.7 = 48.3 keV ‘The oe-particle will stop at a distance r when the potential energy at this position equals its initial kinetic energy; that is 4m, or Here Q is the charge on the nucleus. (an)(8.8542<10 ")(7.0>10")(5.0-1.002«10"") (2)(1.602x10") SQ=19x10" C Copyright © 2014 by Nelson Education Ltd 341 4 45 342 0.0 20.00 40.00 60,00 $0.00 100.00 rc<10 my ‘The average separation distance can be calculated by equating potential energy with average thermal energy; that is 1 (1.602410) Gas, AP (ea(SBS410™) (L3BIn10™\(LSx10") Sre74d 0? m Note that we have used average thermal energy for this ealeulation. There arc many particles at the higher energy tail, which would have much lower separation than calculated above. haw ‘ux i the radius of the spectrometer magnet in this ease. (1.602510 )(300 10°05) (39.948)(1.605 x10 ) Yee = =a = 3.7x10' mis ‘The electri field is given by ‘When electrons are travelling in a straight line we have ef =evB £20000 22.510" ms B 08x10 Copyright © 2014 by Nelson Education Ltd ‘When the electric field is removed, the electrons move in a curved path according to me = evB my _ (9.10910 7" }(2.5 4107) eB (1.602 x10" (08x10) ‘The maximum length of the plates ean be determined by applying the Pythagorean 1777 ma =17.77 om theorem to the triangle shown in the next figure. Loge = NL7.77° 17.27" 42cm with a=2" anap= ° iT To find the frequency at which intensity is maximum we will differentiate this with respeot to frequency and equate to zero; that is at —Aepe” G eh Hl (oF -1) After simplifications, we get pp=3(l-e*) Copyright © 2014 by Nelson Education Ltd 343 This equation can be numericall BP = 282 Mf 22.92 iT ‘The frequency that corresponds to maximum intensity is 282k Som =F olved to give > 49, To probe the electron structure using an electron, its wavelength must be smaller than the radius of the target; that is 4.<10"7 m h . =2<10" m PB = p> 107 =(10")(6.626 10) => p, > 6.626 x10" kg m/s The relativistic energy-momentum relation is, pret =P — mic! ‘The condition on momentum can be written 5 pie >(6.626x10") ct => EF — mic! > (6.626 x10") 38> (602600) dame! 2i> |leszni0” Ft + met = E> y(6.026x10°Y (8.00010 + (9.10910 (3.00 «10° => E>1.988x10" J Or E> 124.1 GeV 51. Let us compars the charge-to-mass ratio of an clectron to that of a proton. €) _1602x10 9.109 x10" 758 x10" C/kg = 9.57610" Crkg re) _ 1602310" m), 1.673x10 Copyright © 2014 by Nelson Education Ltd ‘The charge-to-mass ratio in a crossed-field apparatus is given by E mB Since it would be impractical to change the length of the plates e (the radius remains the same), the only variables we ean change are the electric and magnetic ficlds. Dividing the charge-to-mass ratio of the electron by that of proton, we obtain F, YB _1.788x10"" = 1835.84 © 9ST6x1 vi te ‘Therefore the ratio of the square of the magnetic field to the electric field must be changed such that this condition is satisfied. 53. Given =1300 Wim? Or =1300 Im ‘This means that every square metre of Earth reccives £ = 1300 J of energy per second Nhe = 300 a Here NV is the number of photons, 3002 '1300)(S00 x10” - wy = 13002 (1300)(500%107) aa tot The (6.626x10*)(3.00%10") Hence about 3.2710" photons strike the surface of Earth per square metre per second, Copyright © 2014 by Nelson Education Ltd 345 55. Hydrogen combustion produces approximately 286 k/inol of energy 346 According to Statistics Canada in 2007 the average household in Canada consumed about 106 x10” J of energy. The number of moles of hydrogen needed to produce this much energy is Hence the number of hydrogen molecules needed per year is Ny = (8. Thx10°)(6.023x10) = 2.254310" Since one water molecule is needed to produce one hydrogen molecule, therefore = 2.23410" y, This implies that the mass of water needed per year is 2.234x10" (18) 6.023 x10" = 6.68105 g = 6680 kg Meacer Copyright © 2014 by Nelson Education Ltd Chapter 31—INTRODUCTION TO QUANTUM MECHANICS No. A particle at rest would have a definite position and momentum, which violates Heisenberg's uncertainty principle. Yes, since quantum mechanically, all we can do is associate a probability that the electron will be found at some place and time. However, the probability of such events oveurring are extremely close to zero. ‘The de Broglie wavelength of a particle at rest does not exist, however, see Question 1 (a) No, since the energy of an incident photon is less than the difference between the energy of the ground state and first excited state of hydrogen (b) Yes, since the energy of an incident photon is greater than the difference between the energy of the ground state and first excited state of hydrogen (©) Yes, since the energy of an incident photon is greater than the difference between the energy of the ground state and first excited state of hydrogen Forn=4ton=2: AE =255 eV =1.13 eV 12.09 eV Forn=5 ton=3:A#=0.97 eV Hence AE, < AE, < AF, < AR, Forn=6ton=3: A Forn=3ton=1: AE All of the given quantum numbers for the total angular momentum are possible Relativistic momentum is given by Copyright © 2014 by Nelson Education Ltd 347 x10 0.9" (a) d= 8 92010 OPT p10 (9.109 x10" }(0.9)(3.00 x10" ) 6.626 x10-* 0.99¢)° ) a Sie7eet 0290 5.4610" mm (1v9x10" (oo)(R0010)V 15, Relativistic momentum is given by E pe [pt + mee® Substituting p 4 in this equation we get sp he. hi 212.35 m/s Since this velocity is non-relativistic, we can use classical formula to calculate kinetic eneray. =m 2 K 3K =F (0.10910 )(1212.38y' = 6.694 x10 J = 994 10 1.602 x10" Therefore the electron will have to be accelerated through a potential of 4.18 4V |18 10% eV 17. The uncertainty in the x-position of the electron is Av =1.0%10° m Hene the uncertainty in the x-component of momentum is fi 1.055%10* S35" Dy(LOnI0) =53%10 kg mvs 348, Copyright © 2014 by Nelson Education Ltd 19, 2 25, ‘The minimum kinetio energy, assuming the electron is constrained to move along the x-axis, is given by “(apy __ (527107 i =15d0" J 2m (2)(9.109x10 ‘The uncertainty in the momentum of the object is 0.001)(0.2) Henee the uncertainty in position is Ay 2 10s. 0™ 2ap (2)(0.0002) Ap = may. 0002 kg m/s 6x10" m ‘The uncertainty in time is equal to the mean life, that is A= LAAT @) N= (1.44)(10) = 1.44.10 8 ‘The uncertainty in energy is fh 10sx10* aa (aoe) (b) AP=(1.44)(10)=1.44 x10 s The uncertainty in energy is h 1.05 «10™ 2Ar (2)(14410 AB. 6410" J all 2 2 (9.109.10")(10' = 488.10" 3 The maximum allowable uncertainty in energy is AK Co (4.55.10) =9.100 J 100 1.05x10™ (2)(9.1x10) =6x10 5 Since the square of the wave function integrated over the whole length of the space axis for the one-dimensional case must be a dimensionless quantity, therefore the units of the wave function are m™? Copyright © 2014 by Nelson Education Ltd 349 (1) (1.05510) 2" (2)(9.109.10")(10") =6.02x10" J (2) (1.088x10™) a? (2)(9.109%10)(10-™ = 241x107 J P (vosscto™} # (2}(9.109 x10" )(lo™ The probability density is given by =5.42x10" J Probability density (my ') x(m) 29. ft ag (Lossx10™) 2? amit =~" 105 09x10 )(sox10"} = AE = 2.4110 (rf rt) (a) AB= 241010" (27=-P}=7.2%10 J () Ab= 241-108 (9-P)=1.9%10" J 350 Copyright © 2014 by Nelson Education Ltd @) AB A1x10™ (3° =2?)=1.2x10 J A107 @ AB= 241x107 (100° -17) = Fu Con to = P= [2a Since m= 1 “(2)(9.109<107")(0.1 10? 18110 2x10" I =1x10° m=10nm 35. (@) Using a caleulator: sin(0.1) =0.099, e0s(0.1)= 0.995, e°" =1.105 sin(. p=01—2 = 0.099 c08(0.1)= 0.995 a1 01 2 108 Using a calculator: sin(l) = 0.841, cos(1) = 0.540, e' = 2.718 Ff dl 3S! 7! 841 sin(l) = Copyright © 2014 by Nelson Education Ltd 351 roe cos(l) 1-4 — 540 Or Tt a 6 pop op 2 éette lth ong 2 Sl 4! SI 6! Using a calculator: sin(10) =-0.544, eos(10) = -0.839, el” = 22026.466 Since sin(10)= sin(10 —3) = sin (0.5752) = 0.544, therefore the series expansion gives (-0.5752)" fo 5752)" 3 3 Sinee cos (10) =~ cos(10 — 37) » cos (0.5752) = -0.839, therefore the series sin(10) = (-0.5752) 0.544 expansion gives (-0.5752)' | (-0.5752)' 10)=-f1- 0.839 cos(l0) a ai miso +12 2026.46 7 Go? (oy (iay' (ey @) ef 140+ 2 Uw is a 18° ee 3 al St Fa ca 24! a it aor cos9 isin -1) 10 24 [tos = d= f. 312 ‘The normalized wave function is therefore given by 15 : 3 7a) sia (4-*") for -2Pae => P= 0.36 (since the nommalization factor of the wave funetion is = 1) 25 RT AD Png OD Since the nonnalization factor of the wave function is = 1, we can write this as The exponent is ~ 72 fant 7) 7 2(0.5x10") 694.3 1.055 x10) 2(1.673 x10" )(20-10)(1.602 «10 = Pre’ = P= 3.0610" 354 Copyright © 2014 by Nelson Education Ltd (@) For m, (b) Form, =0, the lowest possible values are /= 0 and n= ‘Therefore the number of protons tunnelling per second is given by wx (10")(3.06%10% ) = 3.06102" € Hence the time it will take 1000 protons to tunnel through the barrier is s=1.03«10"" y The orbital angular momentum quantum numbers are given by 1=0,1,2, For n= 4, we have 1=0,1,2,3 The magnitude of the orbital angular momentum is given by La Vien => L=h.VPh. Joh. STA ‘The 2-components of the orbital angular momenta are giv Hi (AFM seon(P-V)L cog 1 L, = mht with m, ‘Thus, for 1 = Thus, for 2 = Thus, for 7 = —2h,—h,0,h, 2h. Thus, for 7 = Hi, —2h,—h,0.A,2H,3h We know that 1=0,1,2,....(m—1) and m, =~L(A FL seoos(P Vad 3, the lowest possible values are f= 3 and n= 4. (©) Form, = +4, the lowest possible values are = 4 and n= 5. Copyright © 2014 by Nelson Education Ltd Aaa 55. Rie tre® a ‘The most probable distance ean be calculated by differentiating the above equation with respect to r and equating it to zero; that is AR) 0 Hence the most probable distance is equal to the Bohr radius. (108.7)(1.602>10" 37. @) m= =1.881x10™ kg (3.00x10°)' (mse) _ moo)” (1.881 +107" )(8.988%10° J (1.60210 2(1.088x10" (Losxa0*° ‘i = a, = __.2.56x10"% m (8.98810" (1.88110 (1.00210) 39. L=la Fora solid sphere Shae mie 3 Copyright © 2014 by Nelson Education Ltd 61 055 x10 ) ——$—_—~, = 251 x10” rad/s (2)(0.10910)(10) The linear velocity of any point on the surface is given by =(10")(2.51%10")=251<10" mis This speed is fur greater than the speed of light, which shows that the idea that the electron can be a solid spinning sphere is inconsistent with special relativity. @ Pehti ht hleohoh ©) 52 fjg0h oJ =A. Gh, /R.0 Jo 7h -h +jh Forj=3: J. =3h,28,2,0,-A,-2A, 3h For f= 2: J. = 2h,,0,—h,-2h For j=1: J, =h,0,-h Forj=0:J.=0 According to Pauli’s exclusion principle, each subshell cannot hold more than 2 electrons. Shell 1 with 7 =1 has 1s subshell and therefore holds a maximum of 2 electrons. Shell 2 with =2 has 2s 2p subshells and therefore holds a maximum of 2(2)' =8 cleetrons. Shell 3 with n =3 has 3s 3p 3d subshells and therefore holds a maximum of 2(3)' =18 electrons. Hence, for m=1, maximum number of electrons = 2(1)* for m= 2, maximum number of electrons = 2(2)" for m= 3, maximum number of electrons = 2(3) ‘This can be generalized for the nth orbital with a maximum number of electrons of 21? Copyright © 2014 by Nelson Education Ltd 357 Chapter 32—INTRODUCTION TO SOLID-' 9. u 15 Decrease a d b Silicon @ F=(\ala=a (0) ¥ ai x(ac0s0")i + asin(30") asin 30") SV =a’ sin(30" )heasin@o' yk = 7 1 m’ of the metal has a mass of m = 856 kg 16.8 g of the metal contains 6.023 x10" atom. . (856)(6,02210") @ => 856 kg of the metal contains V = —————__— = 3.069 x 10 atoms. 0.0168 => 3.06910" atoms occupy a volume of | m’, 1 » Hence one atom occupies a volume of ———— on 3.069 x 10" The volume of an ion is given by Ar(ax10Y ¥, 65610" m? mr ‘Therefore the volume occupied by the conduction electrons is V, = 3.258 x10 —1.656 x10 = 3.092107? m’ The percentage of the volume occupied by conduction electrons is then given by 3.092107" Se F X100 = 94.9% 3.25810 ‘The Fermi energy is given by B34 am\ Copyright © 2014 by Nelson Education Ltd 17. 19, 21. 2(0.10910)(9.691.6021 (10410 ‘Now, the number density of atoms is given by (2.44)(6.022 x10) 33 Hence the number of conduction electrons per atom is Ys po 445x100" = 4.45 x10" atomsiem* = 4.45 x10" atoms/m? electrons/atom ‘The effective mass represents inverse of the band curvature (@) Positive as curvature is positive away from the band gap, (b) Negative as curvature is negative just below the band gap. (©) Positive as curvature is positive just above the band gap. .00 x10" conduction electrons (@_N=(1.00%10")(1.00 x10) (b) The total energy transfer rate is given as 1.00 Js. Hence, the average energy transfer rate per electron is 1 = 1.00x 10" Tis 1.00 x10! seitgr (2.0)(1.60210"") (1.381 x10 )(500) Aeaor ee" 7.05 x10 Since each phosphorus atom contributes one electron, the number of conduction electrons due to phosphorus in silicon is given by 6.02310 Nip, = 2S AIO 1.9444 x10" electrons’ 30.9758 Copyright © 2014 by Nelson Education Ltd 359 ‘The density of silicon is (Py = 2.33 g/om’ = 2.33x10° gm? Hence the number of conduction elcotrons in I g of silicon is Nq= tot = 4.2910? electrons 233010 ‘We need to increase this by a factor of 10°, that is N‘ = 4,29 x10" electrons Hence the mass of phosphorus can be calculated from (img, (1.9444 x10" )+4,29 10" = 4,29 10! my, =2.2x107 9 =0.22 ue 25. For forward- and reverse-bias we have respectively T=, (eo -1) andi_=1,(e*"* -1) ev _(1602x10")(0.3) A 212.889 (1.381x10")(270) 0 x 10° 27, P= =(0.7)(0.1)-0.07W 29. (a) 4=(0.62%10") (0.6210) =3.844 x10" m* 4 = 0.4010 m xd _ (8854x107 )(4.0)(3.844 x10") d 0.40510 34x10 F (b) The total charge on a plate is given by Q=C¥ =(3.403 x10” (24) = 8.16810 7 C And the total number of elementary charges this corresponds to is 8.168100" 1.60210 = 510 360 Copyright © 2014 by Nelson Education Ltd 31. GIVEN: Leonsing ce where C is the proportionality constant ‘The relative percent change in current is given by i-l 100 x100=(1-| = se") 100 = 0.51510" (9 = 0.51% 10"/483 =1.12x10" m™ on (tee "Ml 100= 90% FS i000 vim @) EF d (b) The force experienced by an electron is Feel Hence the acceleration is given by (1.602 x10 )(1000) m 9.109%10" 8x10" mis? (©) Assuming zero v=18x10"¢ ms tial velocity, we have ig 1 a TAY dd) K 3 (2-109 x10 )(1.758 x10!r) = K=00140 J (© K=0.01e = = =00281 Vs dt () 0.028¢ = 1x10" = 1=3.6x10 6 af = 3(1.758x10")(387 10) 1.1210 m1 ae Copyright © 2014 by Nelson Education Ltd 361 i) : 2(o.10910"}) 3 @y- tO) | eee) 1010" 2m" | (1.085 x10) _ + _ he B-E, 03 © ==. AD (8617x107 03 ) ar. AP (8.61710 }(300) 1-910 lve 1 ® SE\= Tene = rer, v4ro( ha] f (a) 86+ (8.51710) 400) 1-86 ev é 1 (b) £=8.6+(8.617 x10" )(490)n{ ——-1]=8.5 ev ® +(Bs17x10*)( ern ) ° al 9.109 x10" wsiTOM 43. (8.85410 )(5)(10"") =440d0° m=4nm 10x10" 362 Copyright © 2014 by Nelson Education Ltd 45, 47. 49. 31 (0.9x3.010° (107 This distance between the circuit elements on a chip does not seem reasonable, ‘Therefore the signal takes much less time to propagate than I ns (10! )(10 = 100 s ‘The Fermi wave number depends on the number of electrons that each atom RC contributes to conduction, which is =. for a certain lattice parameter. 2H a = A=2a k n a (@) Since the Fermi energy for metals is fairly high, only a tiny fraction of electrons elose to the Fermi level have their energies increased when the metal is heated. (b) Number of particles in a given energy interval: nl E)dB = 2¢(B)f (Ede Since at 0 K the cnergy states below the Fermi level are fully occupied, we have f(£) =1. Henee the number of electrons within thermal energy of the Fermi energy is given by = N's 2e(E, kT But we know that 3N 4E, 3NKT 2E, oN, er N 2&, gE, =n’ For copper , =7 eV. Therefore at 7 = 300 K we have Nt _ (3)(8.617 x10" )(300) N (2)(7) =0,0055 (©) No, the electronic contribution to heat capacity in metals is very small since only a fraction of the electrons are within thermal energy of the Fermi energy. Copyright © 2014 by Nelson Education Ltd 363 Chapter 33—INTRODUCTION TO NUCLEAR PHYSICS | 1. Nucleon: A particle in an atomic nucleus; specifically, a proton or neutron Nuclide: A species of nuclei with a specific number of neutrons and protons. Isotopes: Species of nuclei with the same number of protons and different numbers of neutrons Decay Constant: Proportionality constant that characterizes the relation between the decay rate and the amount of a radioactive substance Half Life: Time it takes for half of the atoms of a sample to decay. Decay Diagram: A diagram that specifies the sequence in which seme nuclide decays. Q-value: Net energy released or absorbed in a nuclear reaction Spontaneous Fission: Spontaneous breaking up of a heavy and unstable nucleus, with a release of one or more neutrons, Radioactive Dating: A process through which the age of a substance is estimated based on comparison of abundance of some radio-isotope and its decay products. Proton-Proton Cycle: Fusion of hydrogen nuclei resulting in production of hydrogen, helium, electrons, neutrinos and a release of energy @-decay: In this process the parent nucleus emits an @-particle and transforms into a lighter nucleus. Br -decay: In this process a neutron in the nucleus of the parent transforms into a proton with the emission of an electron and an anti-neutrino. decay: In this provess a proton in the nucteus of the parent transforms into a neutron with the emission of a positron and a neutrino. 5. An @-particle has 2 protons and 2 neutrons The decay of *$U into “f3Pb shows that at least 10 protons have been converted into particles. The total number of neutrons lost in the decay are therefore given by 238-206 -10 = 22 If 10 protons have converted into 5 e-particles, then 10 neutrons out of 22 must have been used as well. This leaves 12 neutrons. We know that neutrons can convert into protons. These 12 neutrons can therefore make amaximum of 3 a-particles Hence the maximum number of @-particles that can be emitted during this decay is 8 3 Copyright © 2014 by Nelson Education Ltd 13. 15, 17, kg ir _ kf Gri /r~ Grit (8.98810 )(t.o2.10)" © (6.672 x10" )(1.673 x10 N=Ne* Given: V = 0.25, = 0.25N, =Nye* ‘The binding energy per nucleon for heavy nuclei (A> 200 ) is less than the binding energy for nnolei in the range 50< A £120, Therefore when a heavier nucleus fissions into two lighter nuclei there is an increase in the total binding energy. This excess binding energy is released in the fission process. ‘The iron nucleus has the largest binding energy per nucleon of any other nucleus. ‘Therefore, if two iron nuclei were to fuse into a heavier nucleus (tellurium), the total binding energy of the tellurium nucleus will be less than that of the two iron nuclei. Therefore, there is no release of energy in this process; on the contrary, energy must be absorbed in the reaction for it to occur If this were the case, all protons would spontaneously decay into neutrons, and so shortly after the big bang there wouldn't be any protons left, and the universe would have been simply a large collection of neutrons E=me 940" 2 5.610 MeV 1602x107 = £=(0,001)(3.00%10")' = 9x10" J E= me 3110" 1.60210" = B= (0.145)(3.00 x10") = 1.3110" 8.15x10" eV Copyright © 2014 by Nelson Education Ltd 365 19. In the beta deoay of a neutron, the maximum energy AF available to the electron and the neutrino is equal to the difference in the rest mass energies of the neutron and the proton AE=(m, =m, Jo" =1.29 MeV Since AF is much less than the rest mass energy of a proton, the kinetic energy of the proton can be ignored in this devay Ifall 1.29 MeV of encray goes to the electron, its momentum is given by BP A [xe mee = 1.270.517 =1.18 MeV/c ‘The maximum momentum of the neutrino corresponds to the situation where both proton and neutron are formed at rest. In this ease the energy of the neutrino is given by (assuming the neutrino is massless) E,=p,c=1.29-0.511=0.779 MeV Hence, the neutrino’s maximum momentum is, P, = 0.779 MeVie 21. May =12.000000 uw may =13.003354 w => my =(12.000000 )( 0.9889) + (13.003354 )(0.0111) =12.011137 uw => my = (2.011137) (1.6605 x10” = 1.994410 kg 23. B(A,Z)=[Zm, +(4-Z)m, -M,,, ]931.5 MeV (a) B(2,1) = [1(1.007276 )+ (2-1)(1.008665)- (2.014102 -0,000548)]931.5 MeV => B(2,1)=2.2 MeV 22 =B,, (21)= 1.1 McV/nucleon () B(4,2)=[2(1.007276) + (4—2)(1.008665)— (4.002603 ~ 20,000548)]931.5 MeV = B(4,2)=28.3 MeV 28.3 4 = 7.1 MeV/nucleon 366 Copyright © 2014 by Nelson Education Ltd 21. ©) B(6,3) =[3(1.007276) + (6 —3)(1.008665)-(6.015122-3 x 0.000548) ]931.5 MeV. = B(6,3 (4) B(56,26) = [26(1.007276) + (56-26}(1 008665) —(55.934939— 26 x0,000548)]931.5 MeV = B(56,26) = 492.2 MeV = 2, (6.26) 222 88 Mev/aston (e) B(208,82) = ['82(1.007276 ) + (208 -82)(1.008665) ~(207.976627 ~ 82 x 0.000548 )]931.5 MeV => B(208,82)= 1636.4 MeV 3 16364 _ 7.9 MeWiimeloon =B,,, (208.82 208 Muyo =Mie —Zm, (@)_M,,, = 2.014102 0.000548 = 2.013554 w (b) M1, = 12.000000 —6 x 0.000548 = 11.996712 u (©) M,,. =15.994915 ~8 «0.000548 =15.990531 u (@) M,,, = 55.934939— 26 x 0.00054 55.920691 u (©) M,_ = 238.050785 ~92 x0,000548 = 258.000369 u BAZ (a) For ;Be = B (84) =15.8(8)-18.3(8}° -0.72 = B(8.4) = 89.87 MeV Copyright © 2014 by Nelson Education Ltd 367 ) ©) @ For 2C = B(12,6)=15.8(12)-18302 7 0. 72) (12y _ agg l2= 246) 12 = B(12,6)= 85.82 MeV For {Sr ; (aay = (84,38) =15.8(84)—18.3(84)3 -0.72 8 (84): yy 7 (84-238) 1 +12.0- =1433.4 MeV v4 = B(84,38) = 722.4 MeV For Pb (s2)' (208) (208- 282)" = B (208,82) = 15.8(208)-18.3(208) -0.72 -23.2 = B(208,82)= 1612 MeV 2 (927 = B(238,92) = 15.8(238)—18.3(238) — 0.72 (238) (238-292) . =-B.2—_—— +I. 1433.4 MeV 238 = B(238,92) = 1791 MeV In2 (5730)(365.25 «24% 3600) 3.83310 57 Copyright © 2014 by Nelson Education Ltd 37. al 4 = 2.210" Bq in2_ in? fa (21/600) Ae * =2,2x10%e 917 x10% s* oxra0" Jats) 5x10" Bq A=AN ‘We know that for carbon-14, 2. = 3.83310 10° A 3833x107 2.609 x10" =e ae 6.023 «10 2.609 x10" atoms of carbon-14 x14 =6.06 x10 ¢=6 we AN 20x10" 3.0x10" In2 6.67x10" 67 x10 st 0x10" § =330y ~[ve2seto" ose) A= 1000e =970 Bq _inz_ mn? @ ae Toyceenay NHI st Soave) noo ) =6.8Rq Ay = 20,010 Bq A=15.0x10% Bg at (= 240 5 Ae* inf 2 )-- inf BA") 1 20 109 5 a} 240°" | 20x10" n2 1.20x11 =578 5 (@) The plot is shown on the next page (6) Slope = —0.0206 = 2=0.0206 Copyright © 2014 by Nelson Education Ltd 369 43. 45. 41. 370 In2 ln fy © 4a-F = pons @ ° 0.1 -02 = =0,0206« 10 20 30 40 = 03 2 04 2-05 1.6 07 08. Time (3) (@)_ Not possible since charge is not conserved. (b) Not possible since charge is not conserved. (©) Possible ()_Not possible since charge is not conserved. (©) Not possible since mass number is not conserved. @ @ im, my ~My ~My, Je (b) Jnt JO NC+ THe Q=(m, + mg — Mg — my, Jor OrQ=(m, +m, —my— my, 931.5 MeV => Q =(1.008665 +16,999131-14,000324 — 4.002603}931.5 => Q=4.54 MeV Let us see if Q-value is positive for this reaction. UO jHe+ jHe+ {He => Q=(m, —3my, 931.5 MeV => Q =(12.000000 3 x 4.002603) 931.5 => Q=-7.27 MeV Since the Q-value is negative, carbon-12 cannot deeay through this channel spontaneously, Copyright © 2014 by Nelson Education Ltd 49. (a) {Sim 2Me+ {He (6) 2Mn > 3V + }He © BFe > {Cr+ jHe SI. @ Mn fcr+e' + (b) Co RFe+e tv © BNis Score +v 53. @ 3H3iHete +r &) Q=(m, —m, —m,, — 2m, —m, 93.5 MeV == (m_—m,— Mg, +2m, —m, 931.5 MeV = = (mg —my, )931.5 MeV = Q = (3.016049 —3.016029)931.5 MeV =18.6 keV 55. Thorium-232 goes through 4 /I-decays and 6 ct-decays before converting into stable Jead-208 as shown in the decay diagram below. Thorium Thorium aa Actinium= @ 8 Radium 224 Radiam—228 Potonin Polonium— Bismuth 212 2 aS Load: Trad 208 Copyright © 2014 by Nelson Education Ltd 371 57. @) dn Bo Key Meas bn —2m,) Q=(m, + mg — mg, — Or Q=(r, my, ma, —-m,)931.5 MeV => Q = (235,043924 —89,919517 — 143,922953 — 1. 008665 }931.5 = Q=179.6 MeV () bn= BU BRb+ Cs + 2hn Q=(m, +m, my, —m,, 2m, )o* Or Q=(m, =m, —m,, ~m,)931.5 MeV > Q = (235.043924 ~ 92.922042 — 140,920046 — 1,008665}931.5 > Q-179.9 Mev 59. B(4,Z)=[2m, +(4-Z)m, B(A,Z) By (AZ)= AS (@)_B(2,1)=[1(1.007276 ) + (2-1) (1.008665) (2.014102—0.000548)] 931.5 MeV =B(21) 2MeV =B., (2,1) = 221.1 MeV/meleon 2 (6,3) = [3(1.007276) + (6-3)(1.008665)—(6.015122—3 0.000548) ]931.5 MeV = B(6,3)= 32.0 MeV = By» (6,3) = 5.3 MeV/ueleon 13(10,5) =[5(1.007276) + (10 5)(1.008665) —(10.012937-5 x0. 000548) ]931 5 McV = B(10,5)= 64.7 MeV 64.7 = B,,, (10,5)= —~ =6.5 MeV/nueleon 372 Copyright © 2014 by Nelson Education Ltd B(14,7) =[7(1.007276) + (14 -7)(1.008665) =(14,003074 ~7 x 0,000548) ]931.5 MeV = B(14,7)=104.7 MeV =B,, (14,7)= 1047 7.5 MeVinucleon 4 B(180,73) =[73 (1.007276) + (180 —73)(1.008663) —(179.947462 —73 x 0.000548) ]931.5 MeV = B(180, 73) = 1444.6 MeV =B,,, (180,73) = 448 0 Mevinucteon (>) @B(4,2)=[2(1.007276)+(4— 2)(1.008665) = (4.002603 2:0,000548)]931.5 MeV MeV =B,., (42) oe = 7.1 MeV nucleon (i) B(4,8) =[4(1.007276) + (8— 4)(1.008665) (8.005305 ~ 4 «0.000548 )]931.5 McV = B(4,8)= 56.5 MeV = =7.1 MeV/nucleon (iii) B(12, 6) = [6(1.007276)+ (12-6 (1.008665) =B,,, (48) =(12,000000 ~ 6 0.000548) ] 931.5 MeV = B(12,6)=92.1 MeV B,, (12,6)= 2427.7 Mev/nneleon e i Civ) B(16,8) = [8(1.007276) + (16— 8)(1.008665) —(15,994915—2x 0.000548)]931.5 MeV = B(16.8)=127.6 MeV 127.6 > B,.,(16,8)= = 8.0 MeViucleon Copyright © 2014 by Nelson Education Ltd 78 61 374 () B(182, 74) =[ 74 (1.007276) + (182 - 74}(1.008665) —(181,948202-74 « 0.000548) ]931.5 MeV = B(182,74) =1459.3 MeV B,,, (182,74) nee 8.0 MeV/nucleon (a) For {Be — {1c + {He we will first calculate the binding energy of beryllium using the liquid drop model. 2 4 (4-22 B(A,Z) = 1584-18343 -0.72=- 23.28 a ° = (8.4) =15.8(8)-18.3(8)! ~0.72 23.2824) ag Mey (ey Q = B(A,2)+ B(4,2)-B(8,4) => Q = 28.24 28.2—47.4=9.0 MeV (b) IfQ ~ value for the reaction 8C > {He + tHe + JHe is positive. this decay model will be possible, We know that the binding energy of carbon-12 is 92.1 MeV. Therefore the Q-value is B(4,2)+B(4,2)+B(4,2)-B(12,6) 5 MeV = Q = 28.24 28.24 28,2-92. Hence we conclude that this spontancous deeay mode of carbon-12 is not possible 2 y A= aA? = (4-22, a y B(A,2Z)= Here we have to caloulate the Q— value using the binding energies of parent and daughter nuclei. Q=S, = B(4,Z)-B(A-1Z) ‘We will calculate this term by term of the binding energy formula Ist term: a,.4-4,(4-1)= a, Copyright © 2014 by Nelson Education Ltd ~a( ~49424940(4? ) (using a Taylor series expansion) Now since is very large, the fourth and higher tems in the bracket ean be neglected. 0 (since all 4 terms are in the denominator and the Taylor series expansion will give higher orders in the denominator and the terms can be neglected since Apr wn After simplifying we ect Combining all terms we get [te Copyright © 2014 by Nelson Education Ltd 375 Chapter 34INTRODUCTION TO PARTICLE PHYSICS 1. @) False; quarks can interact with each other through any of the fundamental forces and can therefore exchange other force mediating particles as well. (b) Tue (©) False, since the strong forve is mediated by gluons, (d) False, since neutrinos are not force carriers (©) Truc, since gluons are not carriers of the strong force. 3. @ Ime (b) False, since mesons contain a quark aud an antiquark whereas baryons are made up of three quarks. (©) True (@) False, since a pion is a meson and not a baryon, (©) True (© Tre 5 @ Tne (b) True, if the electron decelerates, (e) True (@ False; a magnetic field acts in a direction perpendicular to the velocity of a moving charged particle and hence does no work on the eharged particle. (©) A constant magnetic field can change the velocity of a charged particle; if the charged particle has a component of velocity perpendicular to the direction of the magnetic field, the magnetic field exerts a force on the charged particle, changing its direetion of motion, 7. ‘The maximum uncertainty in position is Av=5x10" im a axdpet = Ap2— 2ax 376 Copyright © 2014 by Nelson Education Ltd 15 7. = Vimae > ax oar t (4) a 2m\2Ax) — 2(9.109x10° = AE 2610x103 =381 MeV eleotron, neutrino, quark photon, 2°, gluon a he . Since me* ~ J as shown in the previons question sRot Therefore if its gange bosons were massless, the range of the weak force would be infinite uds: m= 1110 GeV/e? uuus: m= 1100 GeV/e? dds; m =1120 GeV/o* uss: 1m =1300 GeV/o* ‘The u, d, and s quarks have the following quantum numbers: Charge Baryon Number Spin uo 4K 4 4% K 4 s ¥ K K This table can be used to find the charge, baryon number, and spin of the baryons. Charge Baryon Number Spin wus 41 41 % uds 0 41 XK dds -1 +1 x uss 0 +1 XK dss 1 41 XK Copyright © 2014 by Nelson Education Ltd 377 19, ‘The quarks have the following quantum numbers. Charge Baryon Number Spin ee wok RO RRR OR ROS Following the same procedure as in previous question we get the following table of quantum numbers and mass ranks Charge Baryon Number Spin Mass («C MeV/c?) Mass Rank tt, +1 +1 346200 5 tts +1 +1 342125 4 ths 0 41 175325 3 thd 0 +1 175205 2 to +2 +1 141005 I 2. mg =€ (125.04 2.5) =127.5C MeV/c", where C is the proportionality constant. 125.0 + 5.0) =130.0C MeV/c? 125.0 +125.0) = 250.00 MeVic? 2.54125.0)=127.8C MeV/c? >(5.0-+125.0) =130.0C MeVic? > (1300,0+5.0) =1305.00 MeV/c? =m, =m, conservation of baryon number violated In an clectron-positron annihilation process, both particles are momentarily at rest right before annihilation. This means that the total momentum of the system before annihilation is zero. The conservation of linear momentum then dictates that the momentum and henee energy of the photon produced must be zero, that is 1, +0, + 2m’ =F, 2 2met Hence we oan say that production of single photon in an electron-positron annihilation process is not possible based on conservation of energy and momentum processes. ‘The minimum energy of the neutrino corresponds to zero energies for all other particles Conservation of eneray then gives E,+m,c =mo?+me mn, bm, —m, JOS1.S MeV => E, =(1.008664 + 0,0005486 -1.0072765)931. 8 MeV Since the initial kinetic energies of proton and antiproton are zero, momentum conservation gives ps Be Copyright © 2014 by Nelson Education Ltd 31 Because the proton is the least massive baryon, it cannot decay into another baryon; if it decays into a meson, the law of conservation of baryon number will be violated. 33. Let us assume that the following decay is possible 35. 380 yoere Conservation of energy gives E, =2m.c*, where m, is the relativistic mass of electron or positron. Conservation of momentum gives P, =2p, cos 4, where p, is the relativistic momentum of electron or positron and @ is the angle between the direction of motion of photon and the direction of motion of electron or positron E, = pcos = E, =2m,v,ccos? But £, =2m,c? = 2m,v,c00s Sys cos 0 Syde Hence the reaction is not possible. @ roe+e, Assuming 7” to be at rest right before the decay, conservation of momentum gives OmP +P, > B= P=P Assuming that the electron-neutrino is massless, conservation of energy gives m,ct = pots me’ + pe (139.6) -(0.511) 2(139.6) = 69.8 MeVie E, = pe= 69.8 MeV EB, = pre’ + mic* = (69.8) +(0.511) = 69.8 MeV Copyright © 2014 by Nelson Education Ltd (b) 7 Su +7, Assuming 2” to be at rest right before the decay, conservation of momentum gives 0-5, +7, > P.=P.=P Assuming that the muon-neutrino is massless, conservation of energy gives mo? = [Pe + mie + po Lime —mict 1 (1396) (108.6) = 29.9 MeV/c © Im,c 2(139.6) E, = pe=29.9 MeV \p'e" +mic* = y(29.9)" +(105.6)° =109.7 MeV 37. a f 2 a 41. Letus assume that the uncertainty in the energy of the exchanged particle is on the order of its rest mass energy, that is AE= me According to Heisenberg’s uncertainty relation apart Copyright © 2014 by Nelson Education Ltd 381 382 Henoe the particle can exist for a time of ate Ime And its range is given by r=—te Ime 1,055 10 **)(3.00%10° 6 Qre ta) eae 2 (2)(135.0x1.602 x10") 1.08510“ }(3.00><10" } Gore, (1.05sx10™)(300%10") sot an 2 (2)(783.01,602%10") (1.055 x10™ )(3.00 x10") (2)(1020.0%1.602%10) =9.7x107 mn (b) If two protons could exchange an electron, the range of the strong force would be fo (1.055 x10*)(3,00 10°) & (2)(0.511x1.602x10P) 9x10" m, which is much larger than Ime a typical nuclear diameter. Therefore protons cannot exchange electrons. The total relativistic energy of a particle is given by mige Ke me’ Copyright © 2014 by Nelson Education Ltd 3.0x10° 931.5) Ly = 1 -| | = 0.109 m= 109 ©) OLD ec0x10°) (wears) —_ 1, - (35) = 0.322 m=32.2 em (2)(200%10°)¥ 100+ 981.5 (iii) L, = 30x10 (3) 0.426 m = 42.6 om (}200x10°)V 2004 981.5 (b) v=2fl, Hence for fixed tube length vat mkas ‘Therefore the frequency must be increased to achieve higher kinetiv energies. © @1,- Se 1-( 2st) =0.749 m= 74.9 om “eae \io+05T1 a man) = 0.750 m= 75.0 em (2)(200%10°)¥ 100+ 0.511 Giz, - 32 '-(seeaen) = 0.750 m= 75.0 om (2)(200x10°)Y 200+0.511 4. @ vee /t-[—™_] = (010°) (es) 2.28%10° m/s Kame 300+ 931.5 (ey ¢ = Litoumference wnference 114s 55, 16% 530m y 2.28x10" © } “vy )____931.5.602 10" 2.2810" a) (22810°F (1.602 «107 )(2) => Circumference = 27, = 11.4 m Copyright © 2014 by Nelson Education Ltd 47. a(S) ~Gonue h(t) Keme 10° +9315 => v= 2.99... «10° mvs (with at least 38 decimal places of 9's ) For the following estimation we will use a calculator that can handle up to 31 decimal places. p= (=) ve ay =) 931.5 «1.60210 B (2.99..x10F) [(rso2xt0 (3o0x10°) (3.0x10°)' = R,>1.9x10" m = Circumference = 27, > 10" m Building a synchrotron of this size is simply not possible, as itis larger than the known universe. € at Ieast 38 decimal places of 9's ) For this estimation we will use a calculator that can handle up (o 31 decimal places. (931.5)(1.602x10"") ( (2.99...<10*) } =>B> pts, (Fonz 10 (400000) Gox10°) = B>12x10" T 384 Copyright © 2014 by Nelson Education Ltd

Vous aimerez peut-être aussi